Ευκλειδης Α 88

52
ΑΠΡΙΛΙΟΣ · ΜΑΙΟΣ · IΟΥΝΙΟΣ 2013 ευρ 3,00

description

Περιοδικό Μαθηματικών

Transcript of Ευκλειδης Α 88

Page 1: Ευκλειδης Α 88

ΑΠΡΙΛΙΟΣ · ΜΑΙΟΣ · IΟΥΝΙΟΣ 2013 ευρόι 3,00

Page 2: Ευκλειδης Α 88

ΜΑΘΗΜΑτΙΚΟ ΠΕΡΙΟΔΙΚΟ ΠΛΗΡΟΦΟΡΗΣΗΣ ,. για το yuμνασιο Τεύχος 88

ευκλείδης a� Απρiλιος - Μά"ίος - Ιούνιος 2013 Τιμή Τεύχοvς Εvρώ 3,00

e-mail: [email protected], www.hms.gr

Γενικά 'Αρθρα ./ Μαθηματικά και Ζαχαροπλαστική (2), Επιμέλεια: Δ. Βαpόπουλος . . . . . . . . . . • . . . . . . • . . . . . . . 1 ./ Τα Μαθηματικά και ο Πολιτισμός, Γ. Ωpαιόπουλος ................................... 5 ./ Πολλαπλασιασμός άλα ρώσικα, Ευάγ. χ. Ζιούλας . . • . . . . . . . . . . . . . . . . . . . . . . . . . . . . . . . . . . 8

Α" Τάξη, ./ Η Ιστορία της Αριθμητικής, Γ. Ωpαιόπουλος . . . . . . . . . . . . . . . . . . . . . . . . . . . . . . . . . . . . . . . 11 v'E ' Θ '

ρωτησεις - εωρια, . . . . . . . . . . . . . . . . . . . . . . . . . . . . . . . . . . . . . . . . . . . . . . . . . . . . . . . . . . 13 ./ Επαναληπτικά θέματα Άλγεβρας, Ρ. Κιούφτη . . . . . . . . . . . . . . . . . . . . . . . . . . . . . . . . . . . . . 16 ./ Επαναληπτικά θέματα Γεωμετρίας, π. Apyύpη . . . . . . . . . . . . . . . . . . . . . . . . . . . . . . . . . . . 18

Β' Τάξη v'E ' Θ '

ρωτησεις - εωρια, . . . . . . . . . . . . . . . . . . . . . . . . . . . . . . . . . . . . . . . . . . . . . . . . . . . . . . . . . . 20 ./ Επαναληπτικά θέματα Άλγεβρας, ........................................... 23 ./ Επαναληπτικά θέματα Γεωμετρίας, .......................................... 24

Γ" Τάξη ./ Φυ'λλοqηαοίαςγιατησυWριησηψ=ar+βχ+γσιηΓΓUJΜΧΟίJυΜεχcή:ψαιkι}'vμικα.ί� π. Apyύpη, 28 v'E ' Θ '

ρωτησεις - εωρια, . . . . . . . . . . . . . . . . . . . . . . . . . . . . . . . . . . . . . . . . . . . . . . . . . . . . . . . . . . 31 ./ Επαναληπτικά θέματα, . . . . . . . . . . . . . . . . . . . . . . . . . . . . . . . . . . . . . . . . . . . . . . . . . . . . . 34

Σελίδες για όλους ./ Μαθηματικοί Διαγωνισμοί , Επιτροπή Διαywvιαμώv . . . . . . . . . . . . . . . . . . . . . . . . . . . . . . . . . . . . 38 ./ Αλληλογραφία, Συνεργασίες . . . . . . . . . . . . . . . . . . . . . . . . . . . . . . . . . . . . . . . . . . . . . . . . . . . . 44 ./ Χαίρομαι να λύνω (Θεωρία Παιγνίων), Γ. Τσομιοης . . . . . . . . . . . . . . . . . . . . . . . . . . . . . . . • . . . . . . 45 ./ Τα Μαθηματικά μας διασκεδάζουν, Σ Γεwpyίou ..................................... 49

••••••••••••••••••••••••••••••••••••••••••••••••••••••••••••••••••••••••••••••

ΕΚΔΟΣΗ ΤΗΣ ΜΑΘΗΜΑτΙΚΗΣ ΗΑIΡΕΙΑΣ

ΠΑΝΕΠΙΣΤΗΜΙΟΥ 34 • 106 79 ΑΘΗΝΑ ΤηΑ.: 210 3617784-3616532 Fax: 210 3641025 Εκδότης: Καλογερόπουλος Γρηγόριος Διευθυντής: Εμμανουήλ Κρητικός

Κωδικός ΕΛ.ΤΑ.: 2054 ISSN: 1105 - 7998 Επιμέλεια �Εκδοαης: ΑΑαφάκη Σταυρούλα Βαρόπουλος Δήμος

Η �γ�αιρη Πληρωμή της αυνδρομής - :.:

β�ηθάει ατην έκδοαη

_ του περιοδικού

Συντακτική επιτροπή Επίτιμος Πρόεδρος: Κιούφτη Ροϊδούλα Ωραιόπουλος Γεώργιος Κυράνας Παναγιώτης Πρόεδρος: Λαγός Γεώργιος Βαρόπουλος Δήμος Λυμπερόπουλος Γεώργιος

Αντιπρόεδρος Α": Μενδωνίδης Γεώργιος

Κυράνας Παναγιώτης Μορφοπούλου Μαρία

Αντιπρόεδρος ι·: Μπακάλης Αναcrτάαιος

Λυμπερόπουλος Γεώργιος Πανουαάκης Νικόλαος Γραμματεία: Παπαcrταυρίδης Σταύρος

Αλαφάκη Σταυρούλα Πουλάκη Μαρία Σίακου Μαρία Ρίζος Γεώργιος

Μέλη: Σάλαρης Κωνcrταντίνος

Αγγέλη �ννα Σίακου Μαρία Αλαφάκη Σταυρούλα Τααπακίδης Γεώργιος Αλεξανδράτου �ννα Ταικοπούλου Στάμη Αργύρη Παναγιώτα Χριcrτοδούλου Ντόρα Γεωργίου Σπύρος Χρυαοβέργης Μιχαήλ Γληνού Αικατερίνη Ωραιόπουλος Γεώργιος ••••••••••••••••••••••••••••••••••••••••••••••••••••••••••••••••••••••••••••••

ΙΔΙΟι<τΗΣΙΑ ΤΗ<; ΕΜΗΝΙΚΗΣ ΜΑΘΗΜΑΠΚΗΣ ΕrΑΙΡΕΙΑΣ

Στοιχειοθεσία - Σελιδοποίηση: ΕΛΛΗΝΙΚΗ ΜΑΘΗΜΑ ΠΚΗ ΠΑ/ΡΕ/Α

Εκτύπωση: ROTOPRINT fA. ΜΠΡΟΥΣΑΛΗ & Σ/Α ΕΕ}. τηλ.: 210 6623778-358 ·

ΥnειίΒuνος τunογpαφείοu: Δ. Παπαδόπουλος

• Τα διαφημιζόμενα βιβλία δε σημαίνει ότι προτείνονrαι από την Ε.Μ.Ε. • Οι συνεργασίες, τα άρθρα, οι προτεινόμενες ασκήσεις, οι Αύσεις ασκήσεων κτλ. πρlπει

να σrε'Άνονται έyκαιρα, σrα της Ε.Μ.Ε. με την ένδειξη "Για τον Ευκλείδη Κ". Τα χειρcSyραφα δεν σε κρίση

; Ετήσια συνδρομή (10,00+2,00 Ταχvδρομικά=εvρώ 12,00). Ετήσια συνδρομή για Σχολεία εvρώ 10,00 Το αντίτιμο yια m τεύχη που παραyyέλνονται σrε'Άνεται: 1. Με σπΑή ταχυδρομική εmταyή σε διαταγή Ε.Μ.Ε. Ταχ. Γραφείο Αθήνα 54 Τ.Θ. 30044 2. Στην ιστοσελίδα της Ε.Μ.Ε., 6που υπάρχει δυvατ6τητα τραπεζικής συvαλλαyής με την τράπεζα EUROBANK 3. Πληρώνεται σrα γραφεία της Ε.Μ.Ε. 4. Με ανnκαταβοΑή, σε εταιρεία ταχυμεταφορών σrο χώρο σας, κατά την παραλαβή.

Page 3: Ευκλειδης Α 88

Μαθηματικά και Ζαχαροπλαστική (2)

του Claude Ρ. Bruter, Πρόεδρος Ευρωπαϊκής Εταιρείας για τα Μαθηματικά και Τέχνη(ΕSΜΑ) Επιμέλεια: Βαρόπουλος Δήμος

·"· Απόδοση στα Ελληνικά: Αγγέλη Αννίτα. ��

Μία καμπύλη που ξανακλείνει πάνω στην ίδια, ένα σύρμα του οποίου τα άκρα συνενώνονται, καλείται στα μαθηματικά ένας κόμβος. Με τον ορισμό αυτό, για παράδειγμα, οι κύκλοι είναι κόμβοι, επίπεδοι πολύ απλοί φυσικά. Ο Philippe Rips υλοποίησε δύο κόμβους λιγότερο

απλούς, έναν κόμβο με τρεις λοβούς, που

ονομάζεται κόμβος του τριφυλλιού, και ένα κόμβο με πέντε λοβούς: Δύο όμορφες εικόνες που ακολουθούν, υπολογισμένες από το Jos Leys, δείχνουν έναν κόμβο τριφυλλιού από χρυσό,

γύρω από τον οποίο τυλίγονται σύρματα κόκκινα ή πράσινα.

Αν τον κάνουμε να γυρίσει πολύ γρήγορα, για παράδειγμα ,το πενταγωνικό κόμβο, γύρω από τον άξονα συμμετρίας του, τότε έχει κανείς την εντύπωση πως βλέπει την φωτεινότητα μια

σπείρας. Σε αυτή την σπείρα οι κόμβοι μπορεί να τυλίγονται τακτικά σε πενταγωνικό τριφύλι. Οι εικόνες είναι του Jos Leys.

Συνήθως

μπορούμε να φτιά­

ξουμε φύλλα, στρώ­

σεις, με αυτούς τους

κόμβους. Εδώ,ο Jos

Leys, έχει δύο εικό­

νες που δείχνουν την

έναρξη μιας ελασμα­

τοποίησης, κάνοντας

δύο ορθογωνικές ελασματοποιήσεις στην ίδια σπείρα,

που η μία είναι μέσα στην άλλη .

Μέσα στις ελασματοποιήσεις που έχουμε υπόψη μέχρι σήμερα, θα μπορούσαμε να τις ονομάσουμε τε­λειώματα των ελασματοποιήσεων, πρέπει να επιμεί­νουμε στο γεγονός ότι τα φύλλα είναι όλα απείρως λεπτά, είμαστε εδώ στην μαθηματική φαντασία.

Μέσα στην φυσική πραγματικότητα, αλλά επίσης

ΕΥΚΛΕΙΔΗΣ Β' 88 τ.4/1

Page 4: Ευκλειδης Α 88

----------- Μαθηματικά και Ζαχαροπλαστική (2) ----------­μέσα σε μια άλλη μαθηματική πραγματικότητα, τα φύλλα παίρνοντας πυκνότητα, καλούνται πα­χιά φύλλα η πλάκες( όπως τις πλάκες της σοκολάτας για να μείνουν μέσα στο ζαχαροπλαστείο, θα μπορούσαμε ακόμα να τις αποκαλέσουμε vnc,.,.c,,..

Τα σχέδια και οι εικόνες που παρουσιάζονται δίνουν παραδείγματα ελασματοποιήσεων πιο πα­χιών και οι οποίες θα μπορούσαν να δώσουν το όνομα των ελασματοποιήσεων η τις εγκοπές κα­τά πλάκας. Θα δούμε πως θα τις κατασκευάσουμε. Η κατασκευή μιας πλάκας από τον μαθηματικό είναι πολύ ακριβής. Παίρνει ένα φύλο και τε­ντώνει κάθε σημείο του φύλου σε μια κατεύθυνση δοσμένη και ένα μήκος που δίνεται επίσης. Παριστά ένα σημείο σε αυτή την εργασία με ένα βέλος που ονομάζεται φορέας, δίνει την κατεύ­θυνση και το μήκος του κινούμενου σημείου. Να τα παραδείγματα των φορέων μετατοπίσεως Κάθε φορέας μοιάζει με ένα στέλεχος σιταριού . Σε κάθε σημείο του φύλλου βρίσκεται μια τέ-τοια r ι ράβδος:

1 Όλα αυτά τα t διανύσματα I

σχηματίζουν

διανυσματικό πεδίο. Ορίζεται στο φύλλο.

Στον τοπικό κόσμο της φυσικής πολλά γεγονότα είναι αναστρέψιμα.

Εάν μπορεί να τεντώσει ή τεντωθεί το φύλλο κατά μια έννοια πηγαίνει εδώ f στην πολύ συ­

χνή περίπτωση όπου μπορεί να τεντώνει στην αντίθετη κατεύθυνση t . Η φύση επαναλαμβάνει

τις πράξεις που παρέχουν ιδιότητες σταθερότητας. Για παράδειγμα, μετά από ένα αρχικό τέντω­

μα μπορεί να επαναληφθεί ένα δεύτερο δίνοντας αφορμή για ένα τέντωμα πιο σημαντικό από το

πρώτο και το καλούμε το προκύπτον τέντωμα , ή το άθροισμα των δύο επιτυχημένων τεντωμά­των.

Με αυτές τις δύο ιδιότητες μπορούν να προστεθούν σε ένα τέντωμα (δεδομένου διανυσμα­τικού μήκους),ένα άλλο τέντωμα κατά την αντίθετη κατεύθυνση . Έχει το ίδιο μήκος με το πρώ-το. Το άθροισμα του τεντώματος είναι μηδενικό. Συμβολίζοντας Ο το μηδενικό τέντωμα ή ακό­μα την απουσία του τεντώματος

Συνοπτικά αυτή η ομάδα των αντικειμένων, αυτή η ομάδα των φορέων, χαρακτηρίζεται από τις ακόλουθες ιδιότητες:

ΕΥΚΛΕΙΔΗΣ Β' 88τ.4/2

Page 5: Ευκλειδης Α 88

----------- Μαθηματικά και Ζαχαροπλαστική (2) -----------

1) Μπορούμε να προσθέσουμε δύο διανύσματα για να αποκτήσουμε ένα τρίτο. 2) Θεωρούμε ένα μηδενικό διάνυσμα που αντιστοιχεί στην απουσία της κίνησης 3) Κάθε φορέας διανύσματος έχει ένα αντίθετο που ονομάζεται επίσης συμμετρικό, το άθροισμα αυτών των δύο διανυσμάτων είναι το Ο . 4) Μπορεί επίσης να σημειωθεί ότι ο συνδυασμός (προσθήκη)του μηδενικού τεντώματος με ένα δοσμένο τέντωμα δεν αλλάζει τίποτα σε αυτό το ίδιο (ν+ Ο= Ο+ ν= ν ). 5) Είναι το ίδιο να συνδέσει ένα τρίτο φορέα με την προσθήκη του αθροίσματος των δύο πρώ­των που συνδυάζονται με την πρώτη προσθήκη προς το άθροισμα των δύο υπολοίπων

( (ν+ ν')+ ν" = ν+ (ν'+ ν") ). Ορολογία: Εμείς λέμε ότι αυτό το σύνολο της «έκτασης , τα ταξίδια» τους φορείς διανυσμάτων, δομη­

μένο από τις ιδιότητες που έχει ορίσει, έχει τη δομή της Ομάδας. Επιστροφ1ι στη ζύμη. Ο ζαχαροπλάστης-μαθηματικός προετοιμάζει τη ζύμη που εμφανίζε­

ται για πρώτη φορά ως ένα παραλληλεπίπεδο. Τότε ισοπεδώνει: αυτός που προκάλεσε ένα τέ­ντωμα κατά μία έννοια, το ονομάζουμε αρνητικό, αντιπροσωπεύεται από ένα διάνυσμα ν .Μπορεί να επαναλάβετε αυτή την διαδικασία (νέα εξομάλυνση ν• ) και παράγει ακόμα μια ισοπέδωση ν+ ν• .

Όταν η ζύμη έγινε φύλλο, ο ζαχαροπλάστης διπλώνει και αναδιπλώνει μέχρι να αποκτηθεί το μιλφείγ του. Υπάρχει και μια άλλη λειτουργία που ονομάζεται μετασχηματισμός του αρτοποιού, όπως εφαρμόζεται από τον aρτοποιό που είναι ο μαθηματικός: είναι μετά την αναδίπλωση που

πλαταίνει την ζύμη ξανά. Μετά από μια σειρά λειτουργιών ο αρτοποιός παίρνει μια ζύμη που όπως λέγεται είναι έτοιμη να σπάσει. Μετά έναν άπειρο αριθμό εργασιών, ο μαθηματικός παίρ­νει ένα σύνολο εντελώς αποκομμένο από τα σημεία που σχηματίζονται.

Επιστρέφοντας στην κατασκευή των πλακιδίων. Λέμε ότι μια πλάκα είναι τακτική εάν ορίζε­

ται από τη δράση ενός διανυσματικού πεδίου (μετατόπισης) όλα παρόμοια:

Κοπή και διακ/ισμηση- Πλάκι:ς και γλυκά Προφανώς κόβοντας τις πλάκες και κέικ , συνδέονται με το ζήτημα της κατανομής μεταξύ

καλοφαγάδων. Για να αποφευχθούν οι εντάσεις τείνουμε να εξασφαλίσουμε ότι οι μονάδες εί­ναι περίπου ίσες. Συμβαίνει κάποιοι να θέλουν μεγάλα τμήματα .Διακρίνουμε δύο τύπους μονά­δων: Αυτές που έχουν όλες το ίδιο σχήμα χωρίς απαραίτητα να έχουν τις ίδιες διαστάσεις.

Λ:::ξιλόγιο: Σε όλες τις περιπτώσεις, ένα μέρος θα καλείται επίσης ένα μοτίβο ή μια πέτρα. Η μείωση επίσης ονομάζεται πλακόστρωση . Όταν η περικοπή γίνεται έτσι ώστε οι μονάδες

να έχουν το ίδιο σχήμα, χωρίς απαραιτήτως να έχουν τις ίδιες διαστάσεις, λέμε ότι η μείωση ή πλακόστρωση είναι τοπολογικές. Όπου οι μονάδες έχουν το ίδιο σχήμα και το ίδιο μέγεθος, το

ΕΥΚΛΕΙΔΗΣ Β' 88 τ.4/3

Page 6: Ευκλειδης Α 88

------------ Μαθηματικά και Ζαχαροπλαστική (2) -----------­κόψιμο ή επίστρωση είναι γεωμετρικά.

Μ ψιι,ι'ι πιφαδι:ίγιιατα. Στην εικόνα του Jos Leys, όλα τα πρότυπα έχουν το ίδιο σχήμα ενός επίπεδου δίσκου. Η εικόνα δείχνει την πρόσφατη λύση στο ζήτημα που έθεσε ο μεγάλος Γερμανός μαθηματικός του 1 9ου αιώνα, Felix Klein, αν υπάρχει μια τέτοια στρώση από να κατασκευάσουμε.

Σημειώνουμε την παρουσία ενός συνολικού σχεδίου, αλλά εξακολουθεί να υπάρχει μείωση σε μέγεθος σε κάθε στάδιο, και ούτω καθεξής έως το άπειρο: λέγεται ότι υφίσταται αυτοομοιό­τητα, φράκταλ διάστασης του κόσμου. Στην παρακάτω εικόνα, βλέπουμε πλακοστρώσεις από

τετράγωνα ή παραλληλόγραμμα του ίδιου τύπου του οδοστρώματος. Εδώ, όλα είναι μαθηματικά δεν προκύπτει από τον ορισμό και την εφαρμογή του σχεδίου, μια εργασία ως εκ τούτου μη τε­τριμμένη, Πεδίο Mike.

7ο ΜΑΘΗΜΑ ΤΙΚΟ ΚΑΛΟΚΑΙΡΙΝΟ ΣΧΟΛΕΙΟ "ΛΕΠΤΟΚΑΡΥΑ ΠΙΕΡΙΑΣ" 30 Ιουνίου-δ Ιουλίου 2013

Το Δ.Σ. της Ελληνικής Μαθηματικής Εταιρείας, προχωρά στη δημιουργία του Ίου Μαθηματικού Καλοκαιρινού ΣχολriοιJ (Μ. Κ. Σ.). Το Ίο Μ. Κ. Σ. θα λειτουργήσει από 30 Ιουνίου - 6 Ιουλίου 2013 στη Λετττοκαρυά Πιερίας στο Ξενοδοχείο «OLYMPIAN ΒΑΥ» 4 αστέρων. (Πληροφορίες στον ιστότοπο: www.reahotels.gr). Το ξενοδοχείο βρίσκεται 2 Km απόσταση από τον Εθνικό Αυτοκινητόδρομο Αθηνών - Θεσσαλονίκης και 1 000 μέτρα από την αντίστοιχη σιδηρο­δρομική γραμμή. Το ξενοδοχείο διαθέτει γήπεδα μπάσκετ, τένις, βόλεϊ, βιβλιοθήκη, υπερσύγχρονο γυμναστήριο και αίθουσα ηλεκτρονικών υπολογιστών. Η διαμονή θα είναι σε δίκλινα και τρίκλινα κλιματιζόμενα δωμάτια. Τα μαθήματα θα αρχίσουν τη Δευτέρα 1 Ιουλίου 9.00 το πρωί και θα τελειώσουν την Παρασκευή 5 Ιουλίου. Καθημε­ρινά τα μαθήματα θα πραγματοποιούνται σε 5 διδακτικές ώρες, από 9.00 π.μ.- 13.45μ.μ., σε υπερσύγχρονες αί­θουσες, εξοπλισμένες με όλα τα απαιτούμενα εποπτικά μέσα. Το πρόγραμμα διδασκαλίας, η ύλη και οι καθηγητές που θα διδάξουν, θα τελούν υπό την εποπτεία και επιστημονική καθοδήγηση της Επιτροπής Διαγωνισμών της Ελληνικής Μαθηματικής Εταιρείας. Όλοι οι μαθητές θα έχουν πλήρη ασφάλεια. Οι χώροι διαμονής θα φυλάσσονται όλο το 24ωρο από τις υπηρεσίες α­σφαλείας του Ξενοδοχείου, αλλά και από άτομα που θα ανήκουν στην οργανωτική επιτροπή του Μ. Κ. Σ. Η Ελληνική Μαθηματική Εταιρεία από πέρυσι συμμεριζόμενη την πρωτόγνωρη οικονομική κρίση που όλη η Ελληνική κοινωνία βιώνει, προχώρησε σε μία σ ΙJ μ βολική κίνηό"η μείωσης του ποσού συμμετοχής για κάθε μαθητή, που θα ισχύει κω φί:.ισr,. iΊJγκι:κρψtνο: Το κόστος συμμετοχής για κάθε μαθητή ανέρχεται στο ποσόν των 450€ και τη ψ!Jfψ'' υ•ψι.ιι'ιοχrJ'� ιιπορ�ί να τη βρει ο κάθε ενδιαφερόμενος στην ιστοσελίδα της Ελληνικής Μαθηματικής /Ξωψ1 !ω� (ιδιjι σr. ιιορφή pdf), στα σχολεία φοίτησης και σε γνωστούς εκπαιδευτικούς ιστότοπους. Ακόμη για τα αδέλφια ισχύει έκπτωση 20% επί του συνολικού ποσού. Για τη συμμετοχή κάθε μαθητή είναι απαραίτητη η συμπλήρωση του ΔΕΛτΙΟΥ ΣΥΜΜΕΤΟΧΗΣ ΜΑΘΗΤΗ και η κατά­θεση 1 00€ στους παρακάτω λογαριασμούς: 1. Στην Τράπεζα: ΕΘΝΙΚΗ, λογαριασμός όψεως 080/48002300 με IBAN GR87 0110 0800 0000 0804 8002 300 (προσοχή πρέπει να αναγράφονται στο έντυπο της Τράπεζας το επώνυμο, όνομα και πατρώνυμο του καταθέτη). 2. Στην Τράπεζα: ALPHA, λογαριασμός όψεως 10 100 200 20 19 988 με IBAN GR86 0140 1010 1010 0200 2019 988 (προσοχή πρέπει να αναγράφονται στο έντυπο της Τράπεζας το επώνυμο, όνομα και 3. Με ταχυδρομική επιταγή σε διαταγή, ΕΛΛΗΝΙΚΗ ΜΑΘΗΜΑτΙΚΗ ΕΤΑΙΡΕΙΑ, ΤΑΧ. ΓΡΑΦΕΙΟ ΑΘΗΝΑΣ 54, Τ.Θ. 30044 Αντίγραφο του Δελτίου Συμμετοχής μαζί με το δελτίο κατάθεσης, στέλνει ο κάθε ενδιαφερόμενος στα γραφεία της Ε. Μ. Ε. με ένα από τους παρακάτω τρόπους, Fax, ταχυδρομικά, ηλεκτρονικά. Επειδή ο αριθμός των θέσεων είναι συγκεκριμένος θα παρακαλούσαμε οι μαθητές που ενδιαφέρονται να εγγρα­φούν έγκαιρα.

ΕΥΚΛΕΙΔΗΣ Β' 88 τ.4/4

Page 7: Ευκλειδης Α 88

==================-· -····· ·· ·· Γ. Ωραιόπουλος Δ. ΤΟ ΠΑΡΟΝ ΚΑΙ ΤΟ ΜΕΛΛΟΝ ΤΟΥΣ

1 . Ο 20°ς ΑΙΩΝΑΣ Τα μεγάλα κράτη είχαν τις αποικίες τους.

Μόνο οι ΗΠΑ αντί να έχουν αποικίες μετέφε­ραν από την Αφρική χιλιάδες νέγρους να υπη­ρετούν σαν δούλοι των Αμερικάνων τα κτήμα­τα και τα εργοστάσιά τους.

Στον ανταγωνισμό για το ξαναμοίρασμα των αποικιών έγινε ο Α'Παγκόσμιος Πόλεμος ( 1 9 1 4- 1 8) με τεράστιες θυσίες. Οι εμπόλεμοι με τα Μαθηματικά και τις άλλες επιστημονι­κές - τεχνολογικές γνώσεις κατασκεύασαν κα­τάλληλα όπλα.

Το 1 9 17 επαναστάτησαν οι Ρώσοι κατά των τσάρων και ένωσαν τη χώρα με τα γύρω κράτη μέχρι τον Ειρηνικό Ωκεανό και δη­μιούργησαν τη Σοβιετική Ένωση . Ο Ρώσος Μαθηματικός Τσολκόφσκι βοήθησε στην κα­τασκευή διαστημικών πυραύλων και το 1 957 με το διαστημόπλοιο Σπούτκιν πέταξε ο πρώ­τος Σοβιετικός άνθρωπος στο διάστημα.

Ακολούθησαν οι Αμερικανοί με πλουσιό­τερο διαστημικό πρόγραμμα που προσσελή­νωσαν αστροναύτες.

Στην Ασία γίνονται μάχες των Ιαπώνων κρατηθούν στην Ινδοκίνα και Κίνα.

Ο Β ' Παγκόσμιος Πόλεμος ( 1 939- 1 945) έγινε πάλι για το μοίρασμα της Γης από τους Γερμανούς, Ιταλούς, Ιάπωνες ενάντια στη Γαλλία, Αγγλία, ΗΠΑ. Οι χιτλερικοί κατέλα­βαν σχεδόν όλη την Ευρώπη . Αργότερα επιτί­θενται στη Σοβιετική Ένωση η οποία συμμα­χεί με τους Άγγλο-αμερικάνους και όταν οι Γερμανοί προχώρησαν μέχρι το Βόλγα τους νίκησε ο Σοβιετικός στρατός και με aντεπιθέ­σεις τους καταδιώκουν μέχρι το Βερολίνο.

Ο πόλεμος αυτός έφερε μεγάλες κατα­στροφές, πολλά εκατομμύρια νεκροί και τραυματίες και τεράστιες οικονομικές απώλει­ες, πτώση του πνευματικού επιπέδου και εξα­φάνιση του πολιτισμού, γι' αυτό το ανέφερα στο άρθρο αυτό.

Ευτυχώς τα επόμενα χρόνια ήταν ειρηνικά με ανάπτυξη στα Μαθηματικά και τον Πολιτι­σμό, που θα γράψουμε στα επόμενα κεφάλαια.

2. ΠΛΗΡΟΦΟΙ'ΙΚΗ Από την αρχαιότητα οι άνθρωποι ζητού­

σαν πληpοφορίες για να ικανοποιήσουν διά­φορες ανάγκες τους. Έτσι οι μαθηματικοί με τις εργασίες τους έδωσαν σχετικές γνώσεις χρήσιμες σ' αυτούς και στους συνανθρώπους τους. Οι Ινδοί με το δεκαδικό σύστημα θέσης πρόσφεραν με τα σύμβολα 0, 1 ,2 ,3,4,5 ,6,7,8 ,9 ανάπτυξαν την Αριθμητική . Ο Αρχιμήδης με τα πειράματά του βρήκε την άνωση στερεού βυθισμένου σε υγρό, ότι είναι ίση με το βάρος του εκτοπισμένου υγρού. Τότε βγήκε γυμνός στους δρόμους φωνάζοντας «εύρηκα». Ο Πα­σκάλ με την υπολογιστική μηχανή και το τρί­γωνό του βοήθησε αυτούς που εργάζονται με

Τον 20° αιώνα ο Άγγλος μαθηματικός Τούριγκ (1912-54) έθεσε τις βάσεις μιας νέας επιστήμης της Πληροφορικής που με την Τε­χνολογία δίνουν όργανα πληροφορίας τους ηλεκτρονικούς υπολογιστές (Η/Υ) που χρησι­μοποιούν στους υπολογισμούς το δυαδικό σύ­στημα αρίθμησης (0, 1 ) .

Ο Άγγλος μαθηματικός και μηχανικός Σά­νον (1916-2001) έγραψε τη «μαθηματική θεω­ρία της επικοινωνίας». Νέος πήγε στις ΗΠΑ και έγινε Αμερικάνός. Γι' αυτόν η πληροφορία είναι φυσικό μέγεθος και έχει μονάδα μέτρησης το bite (μπίτ=κομματάκι) . Πολλοί ασχολούνται με την Πληροφορική στην Αμερική, την Ιαπω­νία, την Ευρώπη κ.α. Η Πληροφορική είναι μια επιστήμη συλλογής και ανάλυσης πληροφο­ριών και τη μετάδοσή της με καλύτερη γλώσσα τη Basic. Μεγάλοι Η/Υ χρησιμοποιούνται στο εμπόριο, τις τράπεζες, τις δημόσιες υπηρεσίες. Οι πληροφορίες, η γνώση και ιδιαίτερα η μα-

ΕΥΚΛΕΙΔΗΣ Α' 88 τ.4/i

Page 8: Ευκλειδης Α 88

------------- Μαθηματικά και Πολιτισμός -----------­

θηματική συντελούν στην οικονομική, κοινω­νική και πολιτισμική εξέλιξη. Πολύ χρήσιμοι για τους μαθητές είναι οι Η/Υ τσέπης, αλλά πρέπει να ξέρουμε ότι δεν μπορούν να αντικα­ταστήσουν το δάσκαλο άνθρωπο.

3. ΜΑΘΗΜΑΤΙΚΟΠΟΙΗΣΗ ΦΥΣΙ­

ΚΟΧΗΜΕΊΑΣ- ΠΥΡΗΝΙΚΗ ΦΥΣΙΚΗ

Ο Γάλλος Φυσικός Μπεκερέλ (1852-1900) μιλώντας για τα άτομα τα οποία ο Δη­μόκριτος τα ονόμασε έτσι επειδή δεν τέμνο­νται, τα οποία τον 20° αιώνα κόπηκαν σε πυ­ρήνα και σωματίδια τα οποία περιστρέφονται γύρω από τον πυρήνα και οι πυρήνας των α­τόμων διασπάται.

Αυτός με πειράματα στο εργαστήριό του παρατήρησε ότι ορισμένα στοιχεία χημικά φθορίζουν και εκπέμπουν ακτίνες, που προέρ­χονται από σωματίδια των ατόμων του στοι­χείου Ουράνιο. Ένα από τα σωματίδια αυτά ονομάστηκε ηλεκτρόνιο.

Το ζεύγος του Γάλλου Πιερ Κιουρί (1852-1906) και της Πολωνής Μαρίας (1867-1959) ανακάλυψαν δυο νέα στοιχεία χημικά το Πο­λώνιο και το Ράδιο τα οποία εκπέμπουν ακτίνες ραδιενέργειας από τα διασπασμένα άτομά τους. Το ζεύγος Κιουρί και ο Μπεκερέλ τιμήθηκαν με το βραβείο Νόμπελ για τις εργασίες τους.

Η θυγατέρα του Κιουρί Ιρέν σπούδασε φυσικοχημικός και όταν πέθανε ο Πιέρ η δρα­στήρια Μαρία εργάστηκε με την κόρη της, που παντρεύτηκε με τον φυσικομαθηματικό Ζολιό (1900-1958). Το νέο ζεύγος Ιρέν Ζολιό μελέτησαν την ατομική θεωρία και όταν οι Γερμανοί το 1 940 κατέλαβαν το Παρίσι έκλει­σαν το εργαστήριό τους και πολέμησαν τους κατακτητές.Ο Άγγλος πυρηνικός φυσικός Ρά­δερφορντ κτυπώντας τον πυρήνα του αζώτου (Η3) με βαρύ σωματίδιο διάσπασε τον πυρήνα του και διαπίστωσε ότι απελευθερώθηκε ένα άτομο υδρογόνου (Η) Αυτός ήταν ο πρώτος που πέτυχε τη διάσπαση του ατομικού πυρήνα. Ο Ράδερφορντ δίδαξε στο Κέμπριτζ πυρηνική φυσική .

Ο Δανός Μπορ ( 1 885- 1 960) ηλεκτρονικός φυσικός παρακολούθησε τα πειράματα του φίλου του Ράδερφορντ και μελέτησε γενικότερα τα α­τομικά σωματίδια. Εκείνος όμως που έφερε κάτι πολύ σοβαρό ήταν ο Άγγλος Paul Demi ( 1 902-84) μηχανικός ηλεκτρολόγος φυσικός με μαθη­ματική εξίσωση έδωσε τη θεωρία της ύλης με τα ατομικά σωματίδια . Γι' αυτόν το ηλεκτρόνιο εί­ναι αρνητικά ηλεκτρισμένο φορτίο, το ποζιτρόνιο θετικά όπως και το πρωτόνιο, το νετρόνιο ουδέ­τερο όπως και το φωτόνιο. Τα σωματίδια αυτά και πολλά άλλα με τις αλληλεmδράσεις τους και τη κβαντική θεωρία δημιουργούν την ύλη και την αντιύλη.

4. ΘΕΩΡΙΑ ΤΗΣ ΣΧΕΊΊΚΟΤΗΤΑΣ -KBANTIKH ΘΕΩΡΙΑ

Ο μεγάλος φυσικομαθηματικός Αλβέρτος Αϊνστάιν (1879-1955) δημιούργησε τη θεωρία της σχετικότητας που είναι τόσο πρωτότυπη και επαναστατική που χρειάζεται σοβαρή με­λέτη για την κατανόησή της. Θα δώσουμε ένα απλό παράδειγμα.

Ένας εmβάτης κινείται στην ίδια κατεύθυνση μέσα σε ένα πλοίο, όταν περνά από άλλο αγκυ­ροβολημένο πλοίο, στο οποίο ένας παρατηρητής μετρά το πόσο μήκος έτρεξε ο επιβάτης και το βρίσκει μεγαλύτερο, από όσο λέει ο εmβάτης, ε­πειδή αυτός μέτρησε την απόσταση σε σχέση με το πλοίο, ενώ ο παρατηρητής μέτρησε σχετικά με τη θάλασσα, αλλά και ο χρόνος που έγιναν οι δυο μετρήσεις είναι διαφορετικός.

Έτσι ο Αϊνστάιν μελετούσε τους νόμους κίνησης, δηλαδή της απόστασης, της ταχύτη­τας, της επιτάχυνσης, της δύναμης σε 4 δια­στάσεις του χώρου και του χρόνου ενώ ο Ευ­κλείδης και ο Νεύτωνας χρησιμοποιούσαν για όλες τις μετρήσεις το τρισδιάστατο σύστημα.

ΕΥΚΛΕΙΔΗΣ Α' 88 τ.4/6

Page 9: Ευκλειδης Α 88

Στη γενική θεωρία της Σχετικότητας ο Α­ϊνστάιν μελετά την ύλη δηλαδή τη μάζα, το βάρος, τη βαρυτική έλξη στο χώρο και στο χρόνο. Ιστορική είναι η εξίσωση E=mc2, όπου c=300000 km/s η ταχύτητα του φωτός στο κε­νό m ένα μέρος της μάζας χημικού στοιχείου του οποίου άτομα με τους πυρήνες και τα σω­ματίδια αλληλοσυγκρούονται δίνοντας ενέρ­γεια Ε. Ο Γερμανός φυσικός Μαξ Πλανκ (1858-1947) ήταν ο πρώτος που είπε ότι το φως δεν μπαίνει στα μάτια μας σαν κυματική ενέργεια κατά τον Μάξγουελ, αλλά διαδίδεται από aπειροελάχιστες διακριτές ποσότητες, τις οποίες ονόμασε ΚΒΑΝΤ Α. Πρίν από αυτόν ο Αϊνστάιν διαπίστωσε ότι ο υπομικροσκοπικός κόσμος είναι γεμάτος από σωματίδια και ότι η φύση δεν είναι συνεχής, μα θεμελιακά κυμα­τοειδής.Ο Plank και ο Αϊνστάιν ήταν φίλοι αν

Max Planck

και είχαν πολιτικές διαφορές. Και οι δυο ήταν μεγάλοι θεωρητικοί φυσικοί αλλά και μουσικοί βιολιστές.

Πολλοί φυσι­κομαθηματικοί επι­στήμονες, όπως και ο Δανός Μπορ εί­χαν εφαρμογές στη δομη' του ατόμου.

(1858·1947) Όταν το 1 920 ο Α-ϊνστάιν έφτασε στη Νέα Υόρκη και με διαλέ­ξεις μίλησε για τις θεωρίες της σχετικότητας και τα κβάντα με πολλά σωματίδια φωτόνια, ηλεκτρόνια κ.α το ακροατήριο τον χειροκρό­τησε και πολλοί αμερικανοί ασχολήθηκαν σο­βαρά με τα θέματα αυτά.

Οι θεωρίες της σχετικότητας περισσότερο από τις εξελίξεις των Μαθηματικών του 20°υ αιώνα, που διαμόρφωσαν τον πολιτισμό μας.

5. ΜΕΛΛΟΝΤΙΚΆ ΜΑΘΗΜΑ ΤΗ?Α ΚΑΙ ΠΟΛΙΤΙΣΜΟΣ

Η αναπτυξιακή πορεία του πληθυσμού άρ­χισε το 5000 π.Χ. Η καταγραφή των ανθρώ­πων της Γης ήταν το 1 μ.Χ., 250 εκατομμύρια το 1650, 500εκατ. Το 1 859, 1 ,5 δισεκ. Το 1 950, 2,4 δις. Το 1 985, 4,59 δισ. το 2000, 6,5 δις. Το άλμα στην αύξηση του πληθυσμού με τη συχνή βελτίωση της ζωής των ανθρώπων, οφείλονταν στην πρόοδο των Μαθηματικών με όλες τις επιστήμες, την τεχνολογία, τις ε­φευρέσεις. Όλα αυτά είναι πολιτισμός.

Το ανθρώπινο γένος κατάκτησε τις επιτυ­χίες αυτές, παρά τους πολέμους και τους aντα­γωνισμούς ανάμεσα στα μεγάλα κράτη και στις πλούσιες άρχουσες τάξεις με τις κρίσεις.

Πρέπει να είμαστε αισιόδοξοι για την εξέλιξη της μαθηματικής επιστήμης, με τη βοήθεια των νέων Η/Υ. Έτσι επινοήθηκαν οι φράκταλ που εί­ναι οι γραφικές παραστάσεις συναρτήσεων εξι­σώσεων που έχουν ρίζες όχι μόνο πραγματικούς αριθμούς, όπως η εξίσωση που έχει μια ρίζα χ=-2 και οι άλλες 2 ρίζες μη πραγματικοί αριθμοί με γραφική παράσταση μια καμπύλη.

Οι φυσικομαθηματικοί ανακάλυψαν τη θεωρία του Χάους για να μελετήσουν αταξία της φύσης. Ένα παράδειγμα η εξήγηση της δημιουργίας του Σύμπαντος από τη μεγάλη έκρηξη Big Bang που υπολογίζουν ότι έγινε πριν 1 3,7 δισεκατομμύρια χρόνια, με τα άπει­ρα σωματίδια φορτισμένα από αντίθετους η­λεκτρισμούς. Αμέσως άρχισε η ψύξη που με­τασχημάτισε τα σωματίδια σε παλλόμενες μι­κρές χορδές, δομικοί λίθοι παραγωγής των α­τόμων της ύλης και αντιύλης, που άρχισε η δι­αστολή και δημιουργία των ουρανίων σωμά­των. Αυτά και πολλά άλλα θα αναπτυχθούν από τους μαθηματικούς και θα διαμορφώσουν ανώτερο πολιτισμό.

ΕΥΚΛΕΙΔΗΣ Α. 88 τ.4!7

Page 10: Ευκλειδης Α 88

ΠΟΛΜΠΛΑΣΙΑΣΜΟΣ Αλλ ΡΩΣΙΚΑ (Russian Multiplication)

Ευάγγελος Χ. Ζιούλας (Καθηγητής Πληροφορικής) Ιστορικά Στοιχεία Ο πολλαπλασιασμός αΜ Ρωσικά είναι ένας ιστορικός αλγόριθμος που υπολογίζει το γινόμενο

δύο ακέραιων αριθμών χωρίς τη χρήση του πίνακα της προπαίδειας. Οι ρίζες του αλγόριθμου αυτού φτάνουν πολύ παλιά στην ιστορία και ξεκινάνε από τους γραφείς της αρχαίας Αιγύπτου.

Εναλλακτικά ο αλγόριθμος αυτός είναι γνωστός και ως «Αιθιοπικός Πολλαπλασιασμός» (Ethiopian multiplication) ή «Αιγυπτιακός Πολλαπλασιασμός» (Egyptian multiplication) ή «Πολλαπλασιασμός Χωρικών» (Peasant multiplication) . Η διαδικασία που ακολουθεί στηρίζε­ται σε μια σειρά πράξεων πολλαπλασιασμού με το 2, διαίρεσης με το 2 και πρόσθεσης.

Ο αλγόριθμος αυτός εφαρμόζεται σήμερα από τους επεξεργαστές των σύγχρονων υπολογιστών προκειμένου αυτοί να διενεργήσουν πιο γρήγορα τον πολλαπλασιασμό δύο αριθμών. Η χρήση του από τον υπολογιστή στηρίζεται στην πράξη της ολίσθησης (shift) η οποία είναι μία από τις θεμε­λιώδης πράξεις που μπορεί να εκτελέσει ένας υπολογιστής πάνω σε δυαδικές ακολουθίες.

Η πράξη της Ολίσθησης Ο ηλεκτρονικός υπολογιστής είναι μία ψηφιακή συσκευή που αναπαριστά τα δεδομένα του

σε δυαδική μορφή . Αυτό σημαίνει ότι κάθε μορφής πληροφορία που μπορεί ένας υπολογιστής να επεξεργαστεί (όπως εικόνα, ήχος, κείμενο κλπ), παριστάνεται στο εσωτερικό του ως μία ακο­λουθία δυαδικών ψηφίων που είναι ευρύτερα διαδεδομένα με το όνομα bits (binary digits ) .

Ένα δυαδικό ψηφίο μπορεί να πάρει δύο εναλλακτικές τιμές: Ο και 1. Με τις δύο αυτές δυα­δικές τιμές (καταστάσεις) μπορούμε εύκολα να συμβολίσουμε την παρουσία ή την απουσία ρεύ­ματος στα κυκλώματα του υπολογιστή και να παραστήσουμε την οποιαδήποτε πληροφορία του. Έτσι, όταν σε μία δεδομένη στιγμή περνάει ρεύμα από τα κυκλώματα του υπολογιστή, αυτή η κατάσταση μπορεί να παρασταθεί με το bit 1, ενώ όταν δεν περνάει ρεύμα η κατάσταση αυτή παριστάνεται με το bit Ο.

Για παράδειγμα θεωρούμε ότι έχουμε έναν υπολογιστή που μπορεί να παραστήσει όλες τις αριθμητικές τιμές με 8 bits (δηλαδή μέγεθος ίσο με Ι byte). Έτσι στον υπολογιστή αυτό ο αριθ­μός 23 του δεκαδικού8 συστήματος μπορεί να παρασταθεί σε δυαδική μορφή ως 00010111.

Πάνω σε κάθε δυαδική ακολουθία, ο υπολογιστής είναι εύκολο να εφαρμόσει την πράξη της ο­λίσθησης, η οποία μπορεί να γίνει προς δύο κατευθύνσεις: αριστερά και δεξιά. Πιο συγκεκριμένα: • Ολίσθηση προς τα αριστερά σημαίνει ότι χάνεται το aριστερότερο ψηφίο της δυαδικής

ακολουθίας η οποία και συμπληρώνεται από τα δεξιά με Ο. 7 6 5 4 3 2 1 ο ο ο ο 1 ο 1 1 1

• Ολίσθηση προς τα δεξιά σημαίνει ότι χάνεται το δεξιότερο ψηφίο της δυαδικής ακολουθίας η οποία και συμπληρώνεται από τα αριστερά με Ο.

7 6 5 4 3 2 1 ο

ο ο ο 1 ο

ΕΥΚΛΕΙΔΗΣ Α' 88 τ.4/8

Page 11: Ευκλειδης Α 88

------- ΠΟΛΛΑΠΛΑΣΙΑΣΜΟΣ ΑΛΑ ΡΩΣΙΚΑ (Russian Multiplication) Η ολίσθηση πάνω σε έναν αριθμό του δυαδικού συστήματος δεν έχει τυχαία αποτελέσματα

αλλά προκαλεί πάντα σε αυτόν συγκεκριμένες αλλαγές. e Η ολίσθηση προς τα αριστερά ισοδυναμεί με πολλαπλασιασμό επί 2 (* 2) ο Η ολίσθηση προς τα δεξιά ισοδυναμεί με ακέραια διαίρεση διά 2 ( div 2) ;······ ················-· '"'"-" _ ..... _.. .. .......... . ···-·�-..... .. . ............... . ·� . � . " " .. . . ! ΠΑΡΆΔΕΙΓΜΑ ' � Δεκαδικός αριθμός •

23 7 46 7 1 1 7

Δυαδικός αριθμός

000 1 0 1 1 1 00 1 0 1 1 1 0 0000 1 0 1 1

αρχικός αριθμός ολίσθηση προς τα αριστερά ολίσθηση προς τα δεξιά

Ο αλγόριθμος που ακολουθεί ο υπολογιστής για να υπολογίσει το γινόμενο δύο αριθμών, είναι γνωστός ως Πολλαπλασιασμός αλά Ρωσικά.

Σύμφωνα με τον αλγόριθμο αυτό, θεωρούμε τον ένα αριθμό (πολλαπλασιαστέος) ως Α και τον δεύτερο αριθμό (πολλαπλασιαστής) ως Β . Ο υπολογιστής προκειμένου να εκτελέσει τον πολλαπλασιασμό μεταξύ δύο αριθμών Α και Β, βασίζεται στην πράξη της ολίσθησης υποβάλλο­ντας τον ένα αριθμό (έστω Α) σε συνεχείς διπλασιασμούς (αριστερή ολίσθηση) και τον άλλο α­ριθμό (έστω Β) σε συνεχείς ακέραιους υποδιπλασιασμούς (δεξιά ολίσθηση).

Όταν μετά από τους επαναλαμβανόμενους υποδιπλασιασμούς ο αριθμός Β γίνει ίσος με 1 , η διαδικασία των ολισθήσεων σταματάει.

Στο τελευταίο στάδιο της διαδικασίας του αλγορίθμου, ελέγχουμε ποιες από τις τιμές που πήρε ο αριθμός Β είναι περιττές και υπολογίζουμε το άθροισμα των αντίστοιχων τιμών του α­ριθμού Α. Το αποτέλεσμα που θα βρούμε από το άθροισμα των αριθμών είναι το τελικό γινόμε­νο του πολλαπλασιασμού.

Ι ! • !

.

� . .

Έστω ότι θέλουμε να υπολογίσουμε το γινόμενο των αριθμών 35 και 18 Θεωρούμε τον αριθμό 35 ως Α και τον αριθμό 1 8 ως Β (ή αντίστροφα). Διπλασιάζουμε συνεχώς τον αριθμό Α και υποδιπλασιάζουμε ακέραια τον αριθμό Β . Η διαδικασία συνεχίζ εται μέχρι ο αριθμός Β να γίνει 1 . Ελέγχουμε την στήλη του Β και όσοι αριθμοί είναι περιττοί προσθέτουμε τους αντίστοιχους α­ριθμούς από τη στήλη του Α. Το άθροισμα των αριθμών είναι και το τελικό αποτέλεσμα (γινόμενο Α *Β).

Α Β 35 18 70 9 70

140 4 280 2 560 1 +560

Αθροισμα 630

Θεωρούμε Α = 35 και Β = 1 8

.:. '\ '�- ; ...

Α Β 18 35 18 36 17 36 72 8 144 4 288 2

I 576 1 +576 Αθροισμα 630 I

Θεωρούμε Α = 1 8 και Β = 35

Ο αλγόριθμος του Πολλαπλασιασμού αλά Ρωσικά χρησιμοποιείται πρακτικά στους υπολο­γιστές γιατί ακολουθεί μια πιο απλή υλοποίηση σε σχέση με τον παραδοσιακό χειρωνακτικό τρόπο που στηρίζεται στο πίνακα προπαίδειας.

Πιο συγκεκριμένα, ακολουθεί την πράξη της ολίσθησης (πολλαπλασιασμό επί 2, διαίρεση με το 2) και πρόσθεση σε αντίθεση με τον παραδοσιακό τρόπο που απαιτεί πολλαπλασιασμό με οποιοδήποτε ακέραιο και πρόσθεση . Τα κυκλώματα του υπολογιστή μπορούν ταχύτατα να εκτε-

ΕΥΚΛΕΙΔΗΣ Α' 88 τ.4/9

Page 12: Ευκλειδης Α 88

------- ΠΟΛΛΑΠΛΑΣΙΑΣΜΟΣ ΑΛΛ ΡΩΣΙΚΑ (Russian Multiplication) λέσουν την εντολή της ολίσθησης σε αντίθεση με τον πολλαπλασιασμό με οποιοδήποτε αριθμό που θεωρητικά αποτελεί πιο χρονοβόρα διαδικασία.

Για το λόγο αυτό, ο αλγόριθμος του Πολλαπλασιασμού αλά Ρωσικά θεωρείται συμφερότε­ρος στην εύρεση γινομένων δύο ακεραίων αριθμών από τον υπολογιστή .

Υλοποίηση αλγορίθμου Ο αλγόριθμος του Πολλαπλασιασμού αλά Ρωσικά μπορεί να υλοποιηθεί σε όλες τις γνωστές

γλώσσες προγραμματισμού. Παρακάτω, παρουσιάζεται ο αλγόριθμος με τους εξής τρόπους: • Διάγραμμα Ροής Δεδομένων (Flow Chart) • Ψευδογλώσσα • Γλώσσα προγραμματισμού (σε περιβάλλον Γ ΛΩΣΣΟΜΑΘΕΙΑ) • Γλώσσα προγραμματισμού Pascal (σε περιβάλλον IPCute)

ΔΙΆΓΡΑΜΜΑ ΡΟΗΣ ΔΕΔΟΜΕΝΩΝ ΕΝΤΟΛΕΣ ΨΕΥ ΔΟΓ ΛΩΣΣΑΣ

Αλyόριθμος Πολλαπλασιασμός_Ρωσικά Δεδομένα I I Α, Β I I s�o Όσο Β >= 1 επανάλαβε

Αν Β mod 2 = 1 τότε S�S+A

Τέλος_ Αν Α�Α* 2 Β� Β div 2

Τέλος_ επανάληψης Αποτελέσματα I I 5 I I Τέλος Πολλαπλασιαμός_Ρωσικά

ΠΕΡΙΒΑΛΛΟΝ Γ ΛΩΣΣΟΜΑΘΕΙΑ

1 ΠΡΟΓΡΑΜΜΑ Πολλαπλασιασμός αλα Ρωσικά 2 ΜΕΤΑΒΛΗΤΕΣ

- -ΑΚΕΡΑΙΕΣ:Α,Β,S

' ΑΡΧΗ

:ο

ΓΡΑΨΕ 'Δώσε τους αριθμούς Α,Β' ΔΙΆΒΑΣΕ Α, Β s <-- ο ΟΣΟ Β >= 1 ΕΠΑΝΆΛΑΒΕ

ΑΝ Β mod 2 = 1 ΤΟΤΕ

S <-- S + Α ΤΕΛΟΣ ΑΝ Α <-- Α * 2 Β <-- Β div 2

ΤΕΛΟΣ ΕΠΑΝΑΛΗΨΗΣ " ΓΡΑΨΕ 'r ι νόμενο = ' , S 16 ΤΕΛΟΣ ΠΡΟΓΡΑΜΜΑΤΟΣ

Εκτέλεση προγράμματος

ΠΕΡΙΒΑΛΛΟΝ IPCute (Pascal) �------------------------------� ! ι,,''""'''-'

r.mι:· Ull>J}ώJii:1 r;;, ._-;ς.ο·ιr. .:f_,-.-::κ φrogrcιm Rω,5ian_Multiplication; tΊJ;se:J crt; � Vδ.ι� � a,b,S:integer; •beqin c clr�c.r;

writeln('Give the two numbers'); • rea ln (a, b); • S:-0; " while Β >- 1 do 11 beqin

if Β mod 2 z 1 then S:= S + Α;

Α:� Α * 2; Β:• Β div 2

'' end; · a writeln('Pro uct = ',S)

'•end. ΕΥΚΛΕΙΔΗΣ Α' 88 τ.4/10

Εκτέλεση προγράμματος

Page 13: Ευκλειδης Α 88

Η Ιστορία της Αριθμητικής

4. Ακέραιοι αριθμοί Μερικά ποσά μπορούν να μετρηθούν με

δυο αντίθετες φορές όπως κέρδος και ζημία, ένας δρόμος Αθήνα - Λαμία και ο αντίθετος Λαμία - Αθήνα, η θερμοκρασία ενός τόπου πάνω από το Ο και κάτω από το Ο.

Για να μετρηθούν τα ποσά του είδους αυ­τού δεν αρκεί μια μονάδα μέτρησης δηλαδή με έναν αριθμό αλλά με ένα σημείο που να δεί­χνει τον προσανατολισμό του ποσού.

Έτσι οι Ινδοί πρώτοι ονόμασαν το κέρδος θετικό αριθμό +500 και τη ζημιά με αρνητικό αριθμό -500 δηλαδή δημιούργησαν τους θετι­κούς και αρνητικούς αριθμούς που με ένα όνομα λέγονται ακέραιοι αριθμοί. Το μηδέν δεν είναι σχετικός αριθμός αλλά κάθε θετικός αριθμός είναι μεγαλύτερος από το Ο + 1 ,5>0 και κάθε αρνητικός αριθμός είναι μικρότερος από το Ο, - 1 00<0. Κάθε θετικός αριθμός είναι

1 μεγαλύτερος από κάθε αρνητικό + 1 > -1-

2 και κάθε αρνητικός είναι μικρότερος από κάθε

θετικό -5 < +_!_ . 5

Οι αριθμοί που χρησιμοποιούμε μέχρι τώ-

ρα χωρίς πρόσημα +ή- όπως 5, 1 ,5, 2 ήταν 8

θετικοί. Ομόσημοι λέγονται δυο ή περισσότεροι

αριθμοί με το ίδιο πρόσημο και ετερόσημοι όταν δεν έχουν το ίδιο πρόσημο.

Απόλυτη τιμή ενός σχετικού αριθμού είναι ο ίδιος αριθμός χωρίς το σημείο του

l-51 =5, 17±1=7: . Αντίθετοι αριθμοί λέγονται αυτοί που έ­

χουν την ίδια απόλυτη τιμή και διαφορετικά πρόσημα +8 και -8 τότε -8+( +8)=0. χ' I I I I I ι I ι I I I Χ

Β -3 -2 -1,9 -1 Ο 1/2 1 2 3 Α Πάνω σε μια προσανατολισμένη ευθεία,

που λέγεται και άξονας θέτουμε όλους τους ακέραιους αριθμούς αρχίζοντας αριστερά από

Γεώργιος Ωραιόπουλος τους μικρότερους αρνητικούς και προχωρώ­ντας αυξάνουμε φτάνουμε στο Ο που θεωρεί­ται αρχή του άξονα και προχωρούμε σε μεγα­λύτερους. Κάθε ακέραιος αριθμός Α μπορεί να παρασταθεί γεωμετρικά με ευθύγραμμο τμήμα ΟΑ και αντίστροφα κάθε σημείο Β του άξονα παριστάνει το ακέραιο αριθμό ΟΒ.

Ορισμός: Ρητός αριθμός λέγεται κάθε αριθμός που μπορεί να πάρει μορφή κλά­σματος με αριθμητή ακέραιο και παρονο­μαστή μη μηδενικό ακέραιο.

Q � {; με α, β ε Ζ και β ,. ο}. Δ, , α γ , δ β

Π -3 3 υο ρητοι - =- οταν α = γ . .χ. - = -

β δ 5 -5 Κάθε ρητός μπορεί να μετατραπεί σε ίσο

ρητό με θετικό παρονομαστή αλλάζοντας τα πρόσημα αν ο παρονομαστής είναι αρνητικός.

Πράξεις στο σύνολο Q. Το άθροισμα ρητών αριθμών δεν αλλάζει

αν αλλάξουμε τη θέση τους. Ούτε αν αντικα­ταστήσουμε μερικούς με το άθροισμά τους, ούτε αν προσθέσουμε χωριστά τους ομόση­μους. Για να προσθέσουμε 2 ετερόσημους α­ριθμούς τους αφαιρούμε και βάζουμε το πρό­σημο του μεγαλύτερου.

( +ιΞ)+( -s�)+( -%)+(+ι,s)+(+2)+(+o, os)+( - ι ��)+(-0,666 ... )=

( ι ) ( 3 3 ιο ) +1-+1,5+2+0,05 + -5----1-- 0,666 ... =(+1,5+1,5+2+0,05)+ 2 8 4 50

(-s�-�-�-�)=s os+(-s- 405 _ 432 _ 240)=s os+(-s- ιο77)= 8 8 5 9

, 360 360 360

, 360

5 1077 180 1077 897 100 360 360 360 360 Αφαίρεση. Για να αφαιρέσουμε δυο ρητούς αριθμούς προσθέτουμε στο μειωτέο τον αντί­θετο αφαιρετέου. Επειδή η πρόσθεση δυο ρη­τών είναι πάντοτε δυνατή και η αφαίρεση εί­ναι πάντοτε πράξη δυνατή . Π. χ. 5 - 8 = -3 Πολλαπλασιασμός. Για κάθε ρητό έχουμε -l.α=-α:Ετσι - 1 ( +3) = -3 .

Αν α,β ομόσημοι το γινόμενο α.β είναι θετκός. Αν α,β ετερόσημοι το γινόμενο α. β αρνητικός.

ΕΥΚΛΕΙΔΗΣ Α' 88 τ.4/1 1

Page 14: Ευκλειδης Α 88

------------- Ιστορία της Αριθμητικής -------------

Το γινόμενο πολλών ρητών ισούται με το Άρρητοι ή aσύμμετροι αριθμοί γινόμενο των απόλυτων τιμών όλων των πα- Ο Αρχαίος έλληνας μαθηματικός Πυθαγό-ραγόντων με πρόσημο + αν ο αριθμός των αρ- ρας που ανακάλυψε και απέδειξε το Πυθαγό­νητικών παραγόντων είναι άρτιος και - αν εί- ρειο Θεώρημα παρατήρησε ότι στο ορθογώνιο ναι περιττός. Π.χ. και ισοσκελές τρίγωνο με κάθετες πλευρές 1 , 1

( - 1 )( +2 )( -3 )( -5) ( -0, 1) = + 1 2 η υποτείνουσα θ α έχει τετράγωνο 1 2+ 1 2=2,

αλλά το 2 είναι τετράγωνο της .fi = 1, 4 1 4 ... 1�( -2%)( +3 �)( -4�)( -s�) =-468

Αν α>β και γ>Ο τότε (α+β) .γ>Ο ,αγ+βγ>Ο αγ>-βγ ,α>-β.Αν πολλαπλασιάσουμε τα 2 μέλη ανισότητας επί αρνητικό αριθμό η φορά της

ανισότητας αντιστρέφεται. Π. χ. � > -0, 2 τότε 5

3 -- < 0, 2 .

5 1\.lH Αν δοθούν δυο ρητοί αριθμοί

α,β:;t:Ο τότε υπάρχει ένας μόνο ρητός χ τέτοιος ώστε βχ=α ,τότε χ = α: β

Επειδή α : β = α· _!_ , η διαίρεση είναι ο β

πολλαπλασιασμός του διαιρετέου επί τον α­ντίστροφο του διαιρέτη .

Στις αριθμητικές παραστάσεις πρώτα γίνο­νται οι πολλαπλασιασμοί και οι διαιρέσεις στη σειρά που είναι σημειωμένες και ακολουθούν οι προσθέσεις και οι αφαιρέσεις.

-�+� : ( +η-(�-3 }2 =-� +� ·%-(� ·2-3·2 ) =4� Δυνάμεις. Το γινόμενο 5 . 5 . 5 .5=625 λέγεται δύναμη του 5 και γράφεται 54• Ο 5 λέγεται βάση και ο 4 εκθέτης. Η τρίτη δύναμη ενός αριθμού λέγεται κύβος ( -2)( -2)( -2)=( -2)3 =-8 Η δεύτερη δύναμη λέγεται και τετράγωνο Η δύναμη αρνητικού αριθμού είναι θετικός α­ριθμός αν ο εκθέτης είναι άρτιος( - 1 0)4 = 1 0000 και αρνητικός αν ο εκθέτης είναι περιττός ( -0, 1 )3=( -0, 1 )( -0, 1 )( -0, 1 )=-0,00 1

Οι μεγάλοι και μικροί αριθμοί γράφονται με δυνάμεις του 1 0. Η επιφάνεια της Γης είναι 1 50 .000.000km2= 1 5 . 1 07 km2• Η μάζα ενός ηλεκτρονίου είναι 9, 1 09χ1 0-31χλ Ιδιότητες δυνάμεων αμ ·αν=αμ+ν, α: =αμ-ν (α·β)"=αν·β", (αμ )ν=αμν α Π.χ. �=�i: =(-2)' =-s, [HJ']' =(-ι)' =ι

1

I "' Αυτός με τους μαθητές του προχώρησαν

στην εύρεση της αλλά ο αριθμός δεν τελείωνε ούτε ήταν δεκαδικός αριθμός και τον ονόμα­σαν ασύμμετρο. Αργότερα ο μαθηματικός

Θεαίτητος βρήκε ότι και οι J3, J5, J7, J8, .. . είναι ασύμμετροι ,δηλαδή είναι δεκαδικοί με άπειρα δεκαδικά ψηφία μη περιοδικά. Οι τε­τράγωνοι αριθμοί 1 ,4,9, 1 6,25 ,36 . . . έχουν τε­τραγωνική ρίζα ακέραια. Αυτό συμβαίνει και

με άλλες ρίζες ifi, �, ... Αν συμβολίσουμε με Α τους άρρητους α­

ριθμούς, το σύνολο των αριθμών που είναι η ένωση όλων των ρητών Q και όλων των άρρη­των αριθμών Α είναι οι πραγματικοί αριθμοί που συμβολίζονται με το R.

Ο γερμανός μαθηματικός Ντεντεκιν ( 1 831 -1 9 1 6) όρισε τους πραγματικούς αριθμούς με τη θεωρία της τομής.

-ι I ι ιοο I ,-

. Ι:-l 2

1 .. � -�-:- ;: , . " "' ν2

Ο Ντέντεκιντ ήταν μαθητής στο Πανεπι­στήμιο του Γκέντιγκεν του μεγάλου μαθημα­τικού Γκάους που είπε:

«Τα Μαθηματικά είναι η βασίλισσα των Επιστημών. Η Αριθμητική είναι η βασίλισσα των Μαθηματικών».

ΕΥΚΛΕΙΔΗΣ Α' 87 τ.3/12

Page 15: Ευκλειδης Α 88

Α , ΓΥΜΝΑΣΙΟΥ ΟΛΗ Η θΕΩΡΙΑ ΣΕ 1 1 5 ΕΡΩΤΗΣΕΙΣ!

ΑΡΙΘΜΗΤΙΚΗ • ΑλfΕΒΡΑ

Α. 1 . 2 1 . Ποιες είναι οι ιδιότητες της πρόσθεσης των φυσικών; ( σελ. 15) 2. Πως ορίζεται η πράξη της αφαίρεσης στους φυσικούς και πότε αυτή μπορεί να εκτελεστεί; (σελ. 15) 3. Ποιες είναι οι ιδιότητες του πολλαπλασια­σμού των φυσικών; (σελ. 15) 4. Τι λέει η επιμεριστική ιδιότητα του πολλα­πλασιασμού ως προς την πρόσθεση και τι ως προς την αφαίρεση ; (σελ. 15) Α. 1 . 3 5. Τι ονομάζεται νιοστή δύναμη ενός φυσικού αριθμού α, πως συμβολίζεται και πως ονομά­ζονται τα μέρη της; (σελ.20) 6. Πως αλλιώς διαβάζονται η δεύτερη και η τρίτη δύναμη ενός φυσικού αριθμού α και με τι είναι ίσα το α1 και το Ι ν; (σελ.20) 7. Τι ονομάζεται αριθμητική παράσταση και τι τιμή αριθμητικής παράστασης; (σελ.21) Α. 1 . 4 8. Τι ονομάζεται Ευκλείδεια διαίρεση; (σελ.25) 9. Πότε η Ευκλείδεια διαίρεση λέγεται τέ­λεια και ποιες είναι οι ιδιότητες της; (σελ. 25) λ. 1 . 5 10. Τι ονομάζονται πολλαπλάσια ενός φυσι­κού αριθμού; (σελ. 2 7) 1 1 . Ποιες ιδιότητες ισχύουν για τα πολλα­πλάσια ενός φυσικού αριθμού; (σελ. 2 7) 12. τι ονομάζεται ελάχιστο κοινό πολλαπλά­σιο (ΕΚΠ) δύο η περισσοτέρων αριθμών δια­φορετικών του μηδενός; (σελ. 2 7) 13. Ποιοι ονομάζονται διαιρέτες ενός φυσι-κού αριθμού; (σελ.27) 14. Ποιοι αριθμοί ονομάζονται πρώτοι και ποιοι σύνθετοι; (σελ.27) 15. τι ονομάζεται μέγιστος κοινός διαιρέτης δύο φυσικών αριθμών; ΜΚΔ(α,β); (σελ. 2 7) 16. Πότε δύο φυσικοί αριθμοί ονομάζονται πρώτοι μεταξύ τους; (σελ. 2 7) 17. Ποια είναι τα κριτήρια της διαιρετότητας;

Κλάσματα Α. 2 . I 18. Τι ονομάζεται κλασματική μονάδα; (σελ. 35) 19. 1 8 .Τι ονομάζεται κλάσμα ή κλασματικός αριθμός και τι διακρίνουμε σ' αυτό; (σελ. 35) 20. Τι παριστάνει ένα κλάσμα; (σελ. 35) 21 . Μπορεί ένας φυσικός αριθμός να γραφεί σαν κλάσμα; (σελ. 35) Α. 2 . 2 22. Πότε · δύο κλάσματα λέγονται ισοδύναμα ή ίσα; (σλ. 38) 23. Ποιες είναι οι ιδιότητες των ισοδυνάμων κλασμάτων; (σελ. 38) 24. Πότε δύο ή περισσότερα κλάσματα λέγο­νται ομώνυμα και πότε ετερώνυμα; (σελ. 38) Α . 2 . 3 25. Πως συγκρίνουμε δύο κλάσματα; (σελ. 41) Α. 2 . 4 26. Τι ονομάζεται μικτός αριθμός; (σελ. 45) Α. 2 . 5 27. Πότε δύο κλάσματα λέγονται αντίστροφα; (σελ. 48) Α . 2 . 6

ένα κλάσμα λέγεται σύνθετο;

Δεκαδικοί α ρ ιb μ ο i Α . 3 . 1 29. Πότε ένα κλάσμα λέγεται δεκαδικό; (σελ. 56) 30. Πως κάθε δεκαδικό κλάσμα γράφεται ως δεκαδικός αr2ιθμός; (σελ.56)

ΕξισώσΕις κα ι προμλa) μιηα Α. 4. 1 ---· 31. Τι ονομάζεται, εξίσωση, τι λύση (ή ρίζα) μιας εξίσωσης και τι επίλυση μιας εξίσωσης; (σελ. 73) 32. Πότε μια εξίσωση λέγεται αδύνατη και πό-

73)

Α. 5 . ] 33. Τι ονομάζεται ποσοστό επί τοις εκατό ή απλά ποσοστό και τι ποσοστό επί τοις χιλίοις; (σελ. Β Ο)

ΕΥΚΛΕΙΔΗΣ Α' 88 τ.4/13

Page 16: Ευκλειδης Α 88

--------- Α ' ΓΥΜΝΑΣΙΟΥ ΟΛΗ Η ΘΕΩΡΙΑ Σ Ε 1 1 5 ΕΡΩΤΗΣΕΙΣ! ---------

Κεφάλα-ιο 6°: Ανάλογα ποσά & αντιστρό­φως ανάλογα ποσά Α. 6. 1 34. Τι ονομάζεται ορθοκανονικό σύστημα ημι­αξόνων και τι συντεταγμένες ( τετμημένη, τε­ταγμένη) σημείου; (σελ. 88) 35. Τι γνωρίζετε για τις συντεταγμένες των σημείων των ημιαξόνων Οχ και Oy σ ' ένα ορ­θοκανονικό σύστημα; (σελ. 88) Α. 6. 2 36. τι ονομάζεται λόγος δύο ομοειδών μεγε­θών που μετρήθηκαν με την ίδια μονάδα με­τρησης; (σελ. 91) 37. Τι ονομάζεται αναλογία και ποια η βασική της ιδιότητα; (σελ. 91) 38. τι ονομάζεται κλίμακα; (σελ. 91) 39. Πότε δύο σχήματα λέγονται όμοια; (σελ. 91) Α. 6. 3

40. Πότε δύο ποσά λέγονται ανάλογα; (σελ. 96) 41. Πότε δύο ποσά είναι ανάλογα; (σελ. 96) 42. Ποιες είναι οι ιδιότητες δύο ανάλογων πο­σών; (σελ. 96) Α. 6. 4 43. Που βρίσκονται τα σημεία που παριστά­νουν τα ζεύγη τιμών (x,y) δύο αναλόγων πο­σών; (σελ. 99) Α. 6. 5 44. Πως εξετάζουμε αν δύο ποσά είναι ανάλο­γα; (σελ. 102) Α. 6. 6

45. Πότε δύο ποσά λέγονται αντιστρόφως α­νάλογα; (σελ. 1 07) 46. Πότε δύο ποσά είναι αντιστρόφως ανά­λογα; (σελ. 107) .Κε:giίiλnlo;;?:% Θετικοί & Αρνητικοί αριθμοί Α. 7. 1 47. Τι είναι τα πρόσημα και πως χαρακτηρίζο­νται οι αριθμοί από αυτά; (σελ. 115) 48. Πότε δύο ή περισσότεροι αριθμοί λέγονται ο­μόσημοι και πότε ετερόσημοι; (σελ. 115) 49. Ποιοι είναι οι ακέραιοι και ποιοι οι ρητοί αριθ­μο� (σελ. 115) Α. 7. 2 50. τι εκφράζει η απόλυτη τιμή ενός ρητού αριθ­μού α και πως συμβολίζεται; ( σελ. 118) 51. Πότε δύο ρητοί αριθμοί λέγονται αντίθετοι; (σελ. 118) 52. Ποιος είναι ο αντίθετος του αριθμού χ; (σελ.118)

53. Πως ορίζεται η απόλυτη τιμή ενός ρητού αριθ­μού; (σελ. 118) Α. 7. 3

54. Πως προσθέτουμε δύο ρητούς αριθμούς; (σελ. 122) 55. Ποιες είναι οι ιδιότητες της πρόσθεσης των ρητών; (σελ. 123) Α. 7. 4 56. Πως αφαφσύμε δύο ρητούς αριθμούς; (σελ. 126) Α. 7. 5 57. Πως πολλαπλασιάζουμε 2 ρητούς αριθμούς; (σελ. 130) 58. Ποιες είναι οι ιδιότητες του �­σμσύ των ρητών; (σελ. 130) 59. Πότε δύο ρητοί αριθμοί λέγονται aντί­στροφοι; (σελ. 130) Α. 7. 6

60. Πως διαφσύμε δύο ρητούς αριθμούς; (σελ. 133) �

�φά�αιο 0; Βασικές Γεωμετρικές έννοιες Β. 1 . 1 61 Τι ονομάζεται ευθεία και ποιες προτάσεις αναφέρονται σ' αυτή ; (σελ. 149) 62. Τι ονομάζεται ημιευθεία; (σελ. 149) 63. Ποιες ημιευθείες ονομάζονται αντικείμε­νες; (σελ. 149) 64. τι είναι το επίπεδο και ποιες προτάσεις α­ναφέρονται σ' αυτό; (σελ. 150) 65. Τι ονομάζεται ημιεπίπεδο; (σελ. 150) 8. 1 . 2 66. Τι ονομάζεται γωνία, κυρτή γωνία, μη κυρ­τή γωνία; (σελ. 153) 67. Ποια γραμμή ονομάζεται τεθλασμένη ; (σελ. 154) 68. Πότε μια τεθλασμένη γραμμή ονομάζεται κυρτή και πότε μη κυρτή ; (σελ. 154) 69. Τι ονομάζεται ευθύγραμμο σχήμα; (σελ. 154) 70. Πότε δύο ευθύγραμμα σχήματα λέγονται ίσα; (σελ. 155) 71. Ποια είναι τα αντίστοιχα στοιχεία σε δύο ίσα ευθύγραμμα σχήματα; (σελ. 155) Β. 1 . 3

72. Τι ονομάζεται απόσταση δύο σημείων; (σελ. 159) 73. Τι ονομάζεται μέσο ευθυγράμμου τμήμα­τος; (σελ. 1 60) 8. 1 .5 74. Τι ονομάζεται μέτρο γωνίας; (σελ. 1 65)

ΕΥΚΛΕΙΔΗΣ Α' 88 τ.4/14

Page 17: Ευκλειδης Α 88

--------- Α ' ΓΥΜΝΆΣΙΟΥ ΟΛΗ Η ΘΕΩΡΙΑ Σ Ε 1 1 5 ΕΡΩΤΗΣΕΙΣ! ---------

75. Ποια είναι η μονάδα μέτρησης των γω­νιών; (σελ. 165) 76. Τι ονομάζεται διχοτόμος μιας γωνίας; (σελ. 1 67) Β. 1 . 6 77. Ποια γωνία ονομάζεται: i ) ορθή, ii) οξεία, iii) αμβλεία, iν) ευθεία, ν) μηδενική, νi) πλή­mις; (σελ. 1 70) Β. 1 .7 78. Πότε δύο γωνίες ονομάζονται εφεξής; (σελ. 1 73)

95. Τι ονομάζεται μεσοκάθετος ευθύγραμμου τμή­ματος και ποιες είναι οι ιδιότητες της; (σελ.206) Β. 2. 6 96. Ποιες είναι οι ιδιότητες δύο παραλλήλων ευ­θειών που τέμνονται από μια τρίτη ευθεία; (σελ.214)

Β. 1 .8 97. Ποιο τρίγωνο ονομάζεται i) οξυγώνιο, 79. Πότε δύο γωνίες ονομάζονται παραπλη- ii) ορθογώνιο, iii) αμβλυγώνιο; (σελ.218) ρωματικές; ( σελ. 1 7 6) 98. Ποιο τρίγωνο ονομάζεται: i) σκαληνό, 80. Πότε δύο γωνίες ονομάζονται συμπλη- ii) ισοσκελές, iii) ισόπλευρο ; (σελ.218) ρωματικές; (σελ. 1 76) 99. Τι ονομάζεται διάμεσος ενός τριγώνου; 81 . Πότε δύο γωνίες ονομάζονται κατακορυ- (σελ.219) φήν; (σελ. 1 76) 100.Τι ονομάζεται ύψος ενός τριγώνου; Β. 1 .9 (σελ.219) 82. Πότε δύο ευθείες του επιπέδου ονομά- 101 .Τι ονομάζεται διχοτόμος μιας γωνίας; ζονται παράλληλες; (σελ. 180) (σελ.219) 83. Πως συμβολίζεται η παραλληλία δύο ευ- Β. 3. 2 θειών ε 1 , ε2; (σελ. 180) 102.Να αποδείξετε ότι το άθροισμα των γω-84. Πότε δύο ευθύγραμμα τμήματα λέμε ότι νιών ενός τριγώνου ΑΒΓ είναι 1 80° . (σελ.222) είναι παράλληλα; (σελ. 180) 103.Ποιες είναι οι ιδιότητες του ισοσκελούς 85. Πότε δύο ευθείες του επιπέδου ονομά- τριγώνου; (σελ.221) ζονται τεμνόμενες; (σελ. 180) 104.Ποιες είναι οι ιδιότητες του ισοπλεύρου Β. 1 . 1 0 τριγώνου; (σελ.221) 86. Τι ονομάζεται απόσταση σημείου από ευ- Β. 3. 3 θεία; (σελ. 184) 105.Τι ονομάζεται παραλληλόγραμμο και ποια 87. Τι ονομάζεται απόσταση δύο παραλλή-λων είναι τα στοιχεία του; (σελ.225) ευθειών; (σελ. 184) 106.Ποιες είναι οι ιδιότητες του παραλληλο-Β. 1 . 1 1 γράμμου; (σελ.229) 88. Τι ονομάζεται κύκλος με κέντρο Ο και α- 107.Τι ονομάζεται ορθογώνιο παραλληλό-κτίνα ρ; (σελ. 188) γραμμο; (σελ. 226) 89. Τι ονομάζεται: i) Χορδή, ii) Διάμετρος 108.Ποιες είναι οι ιδιότητες του ορθογωνίου; iii) Τόξο ενός κύκλου; (σελ. 188) (σελ.229) 90. τι ονομάζεται κυκλικός δίσκος με κέντρο 109.Τι ονομάζεται ρόμβος; (σελ.226) Ο και ακτίνα ρ; (σελ. 188) 1 10.Ποιες είναι οι ιδιότητες του ρόμβου; Β. 1 . 13 1 1 1 .Τι ονομάζεται τετράγωνο; (σελ.230) 91. Πότε μια ευθεία λέμε ότι είναι εξωτερική 1 12.Ποιες είναι οι ιδιότητες του τετραγώνου; ενός κύκλου; (σελ. 193) (σελ.230) 92. Πότε μια ευθεία λέγεται εφαπτόμενη ενός 1 13. τι ονομάζεται τραπέζιο και ποια είναι τα κύκλου; (σελ. 193) στοιχεία του; (σελ.226) 93. Πότε μια ευθεία λέγεται τέμνουσα ενός 1 14. Τι ονομάζεται ισοσκελές τραπέζιο; κύκλου; (σελ. 193) (σελ.226) 94. Ποιες οι σχετικές θέσεις μιας ευθείας ε και 1 15.Ποιες είναι οι ιδιότητες του ισοσκελούς ενός κύκλου (Ο,ρ); (σελ. 193) τραπεζίου; (σελ.230) " Οι παραπομπές αφορούν σελίδες του σχολικού βιβλίου «Μαθηματικά Α ' Γυμνασίου»,

Ι.Βανδουλάκης, ΧΚαλλιγάς, Ν.Μαρκάκης, Σ. Φερεντίνος, ΟΕΔΒ, 2007.

ΕΥΚΛΕΙΔΗΣ Α ' 88 τ.4/15

Page 18: Ευκλειδης Α 88

=====Κιούφτη Ροϊδούλα

Δίνονται οι παραστάσεις: Α =4 2 • [ ( 3 2 - 7) + ( 3 · 23 - ι 02 : 5) J - 62 : 9 · 2 Β = (72 - 22 - ι2 )8 + ( 2 · 33 - ι 53 : 3) · ( 43 - 2 · 5 2 ) - 4 · ( 2 · 1 3 - 42 ) Να υπολογίσετε την τιμή τους.

(2Α)3 ·Β 2 Δίνεται το κλάσμα Κ = 3 • Να εξετάσετε αν το κλάσμα είναι ανάγωγο και (Α + ι) · (Β + ι) αν όχι να μετατραπεί σε ισοδύναμο ανάγωγο κλάσμα.

Αν χ=2 � { � . � - �) -% { 3 � : ι: _ 2 � . �) y = 2{(2 -Hi - 4{� - 4 ·H)J -HH- � , �) +%]

Να υπολογίσετε την τιμή τους. Να χαρακτηρίσετε τις προτάσεις ως σωστές ή λάθος: i . Ο αριθμός χ στον άξονα των πραγματικών βρίσκεται αριστερά του y.

ii. Ο αριθμός χ στον άξονα των πραγματικών βρίσκεται πλησιέστερα στον αριθμό 2 από ότι ο y.

Δίνεται η παράσταση : Α= (30:6 · 2)2 - (ι 5 - 5 :3+22 · 6) - 3 · (25 - 33 + 2' ) Να υπολογίσετε την τιμή της. Να βρεθούν οι διαιρέτες του αριθμού Α. Να αναλύσετε τον αριθμό Α σε γινόμενο πρώτων παραγόντων.

Δίνονται οι παραστάσεις: Α = [(�- ι} 1� } [(f - 2 }(�+;ι )] Β = ! + [(� - �HJH- �) - :� Να υπολογίσετε την τιμή τους. Να χαρακτηρίσετε τις παρακάτω σχέσεις ως σωστές ή λάθος:

ί. Α>Β ίί. Α<Β ίίί. Α> ι 7 iv. Β= -(l -�) . (l_ +!) . _!_

Δ, , Α 4 5 . 10 2 . 20 ινονται οι παραστασεις: = ....,--'--....,..-'-�--,----:'-----(1 +�} (� -�} 3 ·:;6 '

Να βρεθούν οι τιμές των Α,Β. Ν λ , , Γ Α ι

α υπο ογισετε την παρασταση =Ι : Β . 2

35 ι -! 3 - (� -!)

Β=-2 - 6 2 �+ ι ιο 3

Να μετατρέψετε τον αριθμό Γ σε μεικτό και να γράψετε τον aντίστροφό του.

ΕΥΚΛΕΙΔΗΣ Α ' 88 τ.4/16

Page 19: Ευκλειδης Α 88

-------- Επαναληπτικές ασκήσεις Α ' Γυμνασίου στην Άλγεβρα --------6. Στην Α ' γυμνασίου ενός σχολείου φοιτούν 1 50 μαθητές και αποφάσισαν να διοργανώσουν

μαθητικούς αγώνες σε τρια αθλήματα: ποδόσφαιρο, μπάσκετ και βόλεϊ. Το ..!.. των μαθητών 2

θα πάρει μέρος στο ποδόσφαιρο, τα � στο μπάσκετ και οι υπόλοιποι στο βόλεϊ. Να βρεθεί: 5

α) Πόσοι μαθητές θα πάρουν μέρος σε κάθε άθλημα. β) Τι ποσοστό των μαθητών θα πάρει μέρος σε κάθε άθλημα.

7. Στις εξετάσεις Ιουνίου από την Α ' τάξη τα 2 των μαθητών προβιβάστηκαν στη Β ' τάξη, το 9

.!.. έμεινε για επανεξέταση το Σεπτέμβριο και οι υπόλοιποι έμειναν στην ίδια τάξη . Αν οι 5 μαθητές που έμειναν στην ίδια τάξη είναι 8 να βρεθούν: α) Πόσοι μαθητές προβιβάστηκαν. β) Πόσοι μαθητές θα επανεξεταστούν το Σεπτέμβρη .

Αν το Σεπτέμβρη προβιβάστηκαν στη Β ' τάξη τα � αυτών που εξετάστηκαν, να βρείτε: 1 5

γ) Τι ποσοστό των μαθητών της Α ' γυμνασίου προβιβάστηκε τον Ιούνιο. δ) Τι ποσοστό των μαθητών της Α ' γυμνασίου προβιβάστηκε τον Σεπτέμβριο. ε) Τι ποσοστό των μαθητών της Α ' γυμνασίου προβιβάστηκε στη Β ' γυμνασίου.

8. Το εισητήριο απλής διαδρομής στις αστικές συγκοινωνίες της Αθήνας το έτος 2000 ήταν 0,50€, το 2008 αυξήθηκε κατά 60% και το 20 1 0 αυξήθηκε και πάλι κατά 25%. Να βρείτε: α) Την τιμή του εισητηρίου το 2008. β) Την τιμή του εισητηρίου το 20 1 0. γ) Τη συνολική αύξηση της τιμής του εισητηρίου από το 2000. ε) Το ποσοστό αύξησης της τιμής του εισητηρίου από το 2000.

9. Ένα κατάστημα ηλεκτρικών συσκευών αυτή την εβδομάδα κάνει προσφορά σε όλα του τα είδη και τα πουλά 30% φθηνότερα. Να βρείτε: α) Πόσο θα αγοράσουμε αυτή την εβδομάδα μια τηλεόραση που η τιμή της είναι 540€. β) Ποιά ήταν η τιμή ενός ψυγείου που με την προσφορά το αγοράσαμε 480€. γ) Πόσο θα πληρώσουμε για μια ηλεκτρική κουζίνα της οποίας η αρχική τιμή είναι 7 50€ αν γι ' αυτή , λόγω ελαττώματος, μας κάνουν επιπλέον έκπτωση 1 5%.

1 Ο. Μια συνταγή γλυκού περιέχει: 4 αυγά 500gr φαρίνα 300gr βούτυρο 2 φλιτζάνια ζάχαρη 1 ,5 φλιτζάνια γάλα. Η κ.Κατερίνα διαθέτει μεγαλύτερο ταψί γι ' αυτό θα φτιάξει μεγαλύτερη δόση χρησιμοποιώντας 1 Ο αυγά. τι ποσότητα πρέπει να βάλει για καθένα από τα υπόλοιπα υλικά.

11 . Καταθέτουμε στην τράπεζα ένα κεφάλαιο 1 8 .000€ με επιτόκιο 3%. Αν οι τόκοι κεφαλαιοποιούνται, δηλαδή προστίθενται στο κεφάλαιο στο τέλος κάθε χρόνου και ξανατοκίζονται με το ίδιο επιτόκιο, να βρείτε τους τόκους που θα πάρουμε μετά από: α) 1 έτος. , β) 2 έτη . , γ) 2 έτη και 6 μήνες.

12. 'Ενα κεφάλαιο 1 0.000€ τοκίζεται με επιτόκιο 8%. Αν ο τόκος φορολογείται με 1 2% , να βρείτε πόσο τόκο θα πάρουμε τελικά στο τέλος του 1 ου έτους.

ΕΥΚΛΕΙΔΗΣ Α' 88 τ.4/1 7

Page 20: Ευκλειδης Α 88

Επαναληπτ ι κά Θέματα Γεωμετρίας ========================= Αργύρη Παναγιώτα

Θεμα Ι ο Δίνεται το ισοσκελές τρίγωνο ΑΒΓ του

παρακάτω σχήματος με ΑΒ=ΑΓ και η γωνία

Α , , , 2 θή του τριγωνου ειναι ιση με τα - της ορ ς. 5

ω

Β Γ Α) Να αποδείξετε ότι η γωνία Α=3 6°

ε

Β) Ν α υπολογίσετε τις γωνίες Β και Γ του τριγώνου ΑΒΓ Γ) Απο το Α φέρνουμε ευθεία ε παράλληλη προς την βάση ΒΓ του τριγώνου ΑΒΓ, να υπολογίσετε με πλήρη αιτιολόγηση την γωνία ω . Απάντηση

Α) Α = 3. · 90° = 1 80 = 36° 5 5

Β) Είναι Α + Β + Γ = 1 80° . Όμως το τρίγωνο ΑΒΓ είναι ισοσκελές, άρα Β=Γ. Οπότε: Α + 2iJ = 1 80° <=> 36° + 2Β = 1 80° <=> <=> 2Β = 1 80° - 36° = 1 44° <=> Β = t = 72° Γ) Είναι ω = Β = 72° ,ως εντός εναλλάξ μεταξύ των παραλλήλων ε,ΒΓ που τέμνονται απο την ΑΒ. Θεμα 2ο Στο παρακάτω σχήμα οι χχ ' , ψ ψ είναι ευθείες και η Οδ είναι διχοτόμος της (μικρότερης) γωνίας χΟψ.

χ· χ

ψ•

Απάντηση Αφού Οδ είναι διχοτόμος : β = 50° . Επίσης α+50° +β= 1 80°,ως παραπληρωματικές. Άρα : α= 1 80° -50° -50°=80° , δ= i)ψ=S(f +S(f =1αf,

ως κατακορυφήν, r = 1 80° - 1 00° = 80° , ως παραπληρωματικές Θεμα 3ο Α) Σε ένα ορθογώνιο τρίγωνο ΑΒΓ (ΑΓ<ΑΒ), η γωνία Γ είναι διπλάσια της γωνίας Β Να υπολογιστούν οι γωνίες Β και Γ. Β) Φέρνουμε την μεσοκάθετο της πλεράς ΒΓ, η οποία τέμνει την κάθετη πλευρά ΑΒ στο Δ . Τι είδους τρίγωνο είναι ΒΜΔ; Να υπολογιστούν οι γωνίες του. Απάντηση Α) Α + Β + t = 1 80° <=> 90° + Β + 2Β = 1 80 <=>

<=> 3Β = 90° <=> Β = 30° , t = 60° Β) Κάθε σημείο της μεσοκαθέτου ισαπέχει απο τα άκρα του ευθυγράμμου τμήματος. Άρα ΒΔ=ΜΔ ,άρα ΜΒΔ ισοσκελές με Β=30°. Είναι : ΒΔΜ = 30° , ΜΔΒ = 1 80° -30° -30° = 120° Θεμα 4ο Οι ευθείες ει και εz είναι παράλληλες. Με βάση το παρακάτω σχήμα να υπολογίσετε:

Α) το χ Β) την γωνία ω Γ) τις γωνίες του τριγώνου ΔΓΕ που σχηματίζεται. Απάντηση Α)Είναι χ+8χ= 1 80° ,ως παραπληρωματικές. Άρα χ=20°. Β) Στο τρίγωνο ΑΒΓ: 30° + χ+ω= 1 80° <:::::> χ = 180° -20° -30° = 1 30° . Γ) Δf'Ε = 1 50° , ως κατακορυφήν, Γ ΔΕ = 30° , ως εντός εναλλάξ της ΓΑΒ μεταξύ των παραλλήλων ΑΒ και ΔΕ που τέμνονται από την ΑΕ, ΓΕΔ = 20 °, ως εντός εναλλάξ της ΓΒΑ μεταξύ των παραλλήλων ΑΒ και ΔΕ που τέμνονται απο την ΔΒ. Θεμα 5ο Σε ένα ρόμβο ΑΒΓΔ η διαγώνιο� ΑΓ σχηματίζει με την πλευρά ΑΒ γωνία 55 • Α) Να υπολογίσετε τις γωνίες του ρόμβου.

ΕΥΚΛΕΙΔΗΣ Α' 88 τ.4/18

Page 21: Ευκλειδης Α 88

-------- Επαναληπτικές ασκήσεις Α ' Γυμνασίου στην Γεωμετρία -------­

Β) Να υπολογίσετε τη γωνία που σχηματίζει η διαγώνιος ΒΔ με την πλευρά ΒΑ του ρόμβου.

Γ

Β

Α) Οι απέναντι γωνίες του ρόμβου είναι ίσες και οι διαγώνιοι του διχοτομούν τις γωνίες Λ 0 ο ο 0 00 του . . Αρα ΒΑΓ = 55 , Β=1 80 -55 -55 =7 , Δ=70°, W = ΑfΔ = 55° . Β) Οι διαγώνιοι του ρόμβου τέμνονται κάθετα άρα ΑΒΔ = 1 80° - 90° - 55° = 35° .

Δίνεται τετράγωνο ΑΒΓΔ και η διαγώνιος του ΑΓ. Α Α Α) Να υπολογίσετε τις γωνίες ΒΑΓ και ΒΓ Α . Β) Να αιτιολογήσετε γιατί η ΒΔ είναι μεσοκάθετος της ΑΓ. Γ) Αν Ο είναι το σημείο τομής των ΑΓ και ΒΔ, να εξηγήσετε γιατί το Ο είναι το κέντρο συμμετρίας του τετραγώνου ΑΒΓΔ. Δ) Απο τα μέσα Κ,Λ,Μ,Ν των πλευρών ΑΒ, ΒΓ, ΓΔ και ΔΑ αντίστοιχα σχηματιζούμε τετράπλευρο ΚΛΜΝ. Να αιτιολογήσετε τι είδους τετράπλευρο είναι το ΚΛΜΝ ;

Γ

Λ

Α Κ Β Α) Οι διαγώνιοι του τετραγώνου διχοτομούν Λ Λ ο τις γωνίες του. Άρα : ΒΑΓ = ΒΓ Α = 45 . Β) Οι διαγώνιοι του ρόμβου τέμνονται κάθετα και διχοτομούνται . Άρα Ο μέσο του ΑΓ και ΒΟ κάθετη στην ΑΓ, οπότε ΒΔ διχοτόμος της ΑΓ. Γ) Ο μέσο της ΑΓ και ΒΔ. ΟΑ=ΟΒ=ΟΓ=ΟΔ. Άρα Ο κέντρο συμμετρίας.

1. Να υπολογίσετε τις άγνωστες γωνίες στα παρακάτω σχήματα.

Α

φ 1 20

Σχήμα 1

1 40

εΙ/η

Σχήμα 3

ΒΓ // ΔΕ Σχήμα 2

ε

η

Γ

2. Το τρίγωνο ΑΒΓ είναι ορθογώνιο και ισχύει ΑΒ=ΑΓ=ΓΔ. Α) Να υπολογίσετε τις γωνίες του τριγώνου ΑΒΔ.

�. Β Γ Δ

Β) Φέρνουμε τη διάμεσο ΓΜ του τριγώνου ΑΓΔ. i) Να αιτιολογήσετε οτι η ΓΜ είναι κάθετη στην AΔ.ii) Να υπολογίστε τη γωνία ΑΓΜ Γ) τι είδους τετράπλευρο είναι το ΑΒΓΜ; Να αιτιολογήσετε την απάντηση σας. 3. Δίνεται τραπέζιο ΑΒΓΔ με Γ=50° και Δ=70°. Να σχεδιάσετε τα ύψη ΑΕ και ΒΗ του τραπεζίου. Α) Τι είδους τετράπλευρο είναι το ΑΒΗΕ ; Να υπολογίσετε τις γωνίες ΔΑΕ και ΗΒΓ .

Δ Ε Η Γ Στο παρακάτω σχήμα το ΑΒΓΔ είναι

Λ ο ορθογώνιο και BK=3cm, ΔΑΚ = 65 . Α) Να βρείτε τα μήκη ΒΔ, ΑΓ, ΚΔ, ΚΑ Β) Να υπολογίσετε τις γωνίες ω και φ.

Α Β

Δ Γ

ΕΥΚΛΕΙΔΗΣ Α' 88 τ.4/19

Page 22: Ευκλειδης Α 88

IB" /I1MNIADOY OMil Η OEORIA ΕΕ 75 f!POlHΣIEW

Α. 1 1 1 . Τι ονομάζεται Αριθμητική και τι

Αλγεβρική παράσταση; (σελ. 1 1) 2. Ti ονομάζουμε όρους μιας αλγεβρικής

παράστασης και τι αναγωγή ομοίων όρων της; σελ. 1 1, 12

Α. 1 2

αριθμού και ποιες οι ιδιότητες της; (σελ. 41 & 42))

Α. 2 2 12. Ποιοι αριθμοί ονομάζονται ρητοί, άρρητοι,

πραγματικοί; (σελ. 45 & 46) 13. Πότε μια ευθεία ονομάζεται άξονας των

πραγματικών αριθμών; (σελ. 46)

3. Ποιες είναι οι τρεις πιθανές σχέσεις που --...ι;.ι Συναρτήσεις συνδέουν δύο αριθμούς α, β ; (σελ. 15) Α. 3 1

4. Ποιοι κανόνες ισχύουν για την ισότητα δύο αριθμών; (σελ. 15 & 1 6)

ι. Τι ονομάζουμε: ίί. εξίσωση; (σελ. 1 7) iii. γνωστούς και άγνωστους όρους μιας

εξίσωσης; ( σελ. 1 7) ίν. λύση ( ή ρίζα) μιας εξίσωσης; (σελ. 1 7) ν. επίλυση μιας εξίσωσης; (σελ. 1 7) 6. Πότε μια εξίσωση λέγεται αδύνατη και

πότε αόριστη(ή ταυτότητα); (σελ. 19)

Α. 1 5

7. τί εννοούμε όταν γράφουμε α ::::; β , και

πως το διαβάζουμε; (σελ. 31) 8. τι συμπέρασμα βγάζετε αν σας πουν ότι

14. τι ονομάζεται συνάρτηση και τι πίνακας τιμών της; (σελ. 55)

Α. 3 2 15. Τι ονομάζεται ορθοκανονικό σύστημα

αξόνων (σύστημα ορθογωνίων αξόνων) και τι συντεταγμένες ( τετμημένη, τεταγμένη) σημείου; (σελ. 59 & 60)

16. Τι ονομάζουμε τεταρτημόρια; (σελ. 60) 17. Τι ονομάζουμε γραφική παράσταση μιας

συνάρτησης; (σελ. 62) 18. Τι γνωρίζετε για τις συντεταγμένες των

σημείων των αξόνων χ 'χ και y 'y σ' ένα ορθοκανονικό σύστημα; (σελ. 62)

Α. 3 3

ισχύουν συγχρόνως οι σχέσεις: α:::; β και 19. Πότε δύο ποσά λέγονται ανάλογα;(σελ. 67)

α2:: β;

9. Να διατυπώσετε τις ιδιότητες των ανισοτήτων. (σελ. 31 & 32)

10. τι ονομάζουμε ανίσωση και τι λύσεις της ανίσωσης; (σελ. 33)

Πραγματικοί αριθμοί Α. 2 1

20. Τι γραμμή είναι η γραφική παράσταση της συνάρτησης y=αχ και από που διέρχεται; (σελ. 68)

21. τι εννοούμε όταν λέμε η ευθεία με εξίσωση y=αχ ή πιο απλά η ευθεία y=αχ; (σελ. 68)

22. Ποια είναι η εξίσωση του άξονα χ ' χ; (σελ. 68)

1 1 . τι ονομάζεται τετραγωνική ρίζα θετικού ΕΥΚΛΕΙΔΗΣ Α' 88 τ.4/20

Page 23: Ευκλειδης Α 88

--------- Β' ΓΥΜΝΑΣΙΟΥ ΟΛΗ Η ΘΕΩΡΙΑ Σ Ε 7 5 ΕΡΩΤΗΣΕΙΣ! ----------

23. Τι ονομάζεται κλίση της ευθείας y=αχ; (σελ. 68)

Α. 3 4 24. Τι γραμμή είναι η γραφική παράσταση της

συνάρτησης y=αχ+β και από που διέρχε­ται; (σελ. 73)

25. Τι εννοούμε όταν λέμε η ευθεία με εξίσωση y=αχ+β ή απλούστερα η ευθεία y=αχ+β; (σελ. 73)

26. τι ονομάζεται κλίση της ευθείας y=αχ+β; (σελ. 73)

27. τι παριστάνει μια εξίσωση της μορφής

μεταβλητής; (σελ. 95) 39. Τι ονομάζεται σχετική συχνότητα μιας

τιμής της μεταβλητής και πως εκφράζεται συνήθως; (σελ. 96)

40. Τι ονομάζεται μέση τιμή μιας μεταβλητής και πως συμβολίζεται; (σελ. 104)

41. Τι ονομάζεται διάμεσος; (σελ. 105)

ΜΕΡΟΕ u � = ΓΕΩΜΕΤΡIΑ Εμβαδά επιπέδων σχημάτων

αχ+βy+γ=Ο με α7=0 και β7=0; (σελ. 74) Β. ι . ι

28. Τι παριστάνει μια εξίσωση τηςμορφή:;: i . αχ+βy=γ ( α7=0 ή β7=0); i i . y=κ; i i i . χ=λ;

iν. χ=Ο; ν. y=O; (σελ. 74) 29. Ποια είναι τα σημεία τομής της ευθείας

αχ+βy=γ με α7=0 και β7=0 με τους άξονες χ 'χ και y 'y. (σελ. 74)

Α. 3 5 30. Πότε δύο ποσά λέγονται αντιστρόφως

ανάλογα; (σελ. 79) 31 . Πότε δύο ποσά είναι αντιστρόφως ανάλογα

και τι προκύπτει απ' αυτό; (σελ. 79) 32. Πως λέγεται η γραφική της συνάρτησης

α y = - με α 7: Ο; (σελ. 80) χ

33. Ποιες είναι οι ιδιότητες της υπερβολής; (σελ. 80)

�f{Ui�rnn� Περιγραφική Στατιστική

42. Τι ονομάζεται εμβαδόν μιας επίπεδης επιφάνειας και από τι εξαρτάται; ( σελ. 1 14)

Β. ι . 2 43. Ποιες είναι οι μονάδες μέτρησης εμβαδού

και ποια η σχέση που τις συνδέει; ( σελ. 1 16)

Β. ι . 3

44. Με τι ισούται το εμβαδόν τετραγώνου, ορθογωνίου, παραλληλογράμμου, τριγώνου, ορθογωνίου τριγώνου, τραπεζίου; ( σελ. 1 19 & 120)

Β. ι . 4

45. Τι λέει το Πυθαγόρειο θεώρημα και τι το αντίστροφο του; (σελ. 128)

Β. 2. ι 46. τι ονομάζεται εφαπτομένη οξείας γωνίας

ορθογωνίου τριγώνου; (σελ. 1 37) 34. τι ονομάζεται πληθυσμός και

μεταβλητή ; (σελ. 86) τι 47. Με τι ισούται η κλίση α της ευθείας με

35. τι ονομάζεται δείγμα δείγματος; (σελ. 86)

και τι μέγεθος

36. Πως γίνεται η συλλογή των στατιστικών δεδομένων; (σελ. 86)

37. Ποια ήδη διαγραμμάτων υπάρχουν; (σελ. 90 & 91)

38. Τι ονομάζεται συχνότητα μιας τιμής της

εξίσωση y=αχ; ( σελ. 13 7)

Β. 2. 2 48. Τι ονομάζεται ημίτονο οξείας γωνίας

ορθογωνίου τριγώνου; (σελ. 142) 49. Τι ονομάζεται συνημίτονο οξείας γωνίας

ορθογωνίου τριγώνου; (σελ. 143)

ΕΥΚΛΕΙΔΗΣ Α ' 88 τ.4/21

Page 24: Ευκλειδης Α 88

---------- 8 ' ΓΥΜΝΆΣΙΟΥ ΟΛΗ Η θΕΩΡΙΑ Σ Ε 7 5 ΕΡΩΤΗΣΕΙΣ! ----------

50. Πως υπολογίζουμε τους τριγωνομετρικούς αριθμούς των 30°,45°,60°;(σελ. 152 & 153)

. ; _1

51 . Τι ονομάζεται εγγεγραμμένη γωνία και τι αντίστοιχο τόξο της; (σελ. 1 75)

60. Πότε μια ευθεία είναι κάθετη σε επίπεδο; (σελ.203)

61. τι ονομάζεται απόσταση σημείου από επίπεδο; (σελ. 203)

62. Τι ονομάζεται απόσταση δύο παραλλήλων επιπέδων; (σελ. 203)

52. Ποιες προτάσεις ισχύουν για εγγεγραμμένες γωνίες; ( σελ. 1 7 6)

τις 63. Ποιο είναι το εμβαδόν της παράπλευρης

53. Τι ονομάζεται: ι. κανονικό πολύγωνο; (σελ. 180) ί ί . περιγεγραμμένος κύκλος κανονικού

πολυγώνου; (σελ. 181) ι ι ι . κεντρική γωνία κανονικού πολυγώνου;

(σελ. 182) ιν. απόστημα κανονικού πολυγώνου; (σελ. 182) 54. Ποια σχέση συνδέει τη γωνία φ και την

κεντρική γωνία ω ενός κανονικού πολυγώνου (ν-γώνου ); (Αιτιολόγηση) (σελ. 182)

55. Ποιοι οι τύποι που μας δίνουν το μήκος (L)

του κύκλου (0, ρ). (σελ. 187)

56. Ποιοι οι τύποι για το εμβαδόν (Ε) του κυκλικού δίσκου (0, ρ); (σελ. 193)

! : ; � -

επιφάνειας Επ και το ολικό εμβαδόν Εολ ενός πρίσματος; (σελ.207)

64. Ποιο είναι το εμβαδόν της παράπλευρης επιφάνειας Επ και το ολικό εμβαδόν Εολ ενός κυλίνδρου; (σελ.208)

65. Τι καλείται όγκος ενός στερεού σώματος; (σελ.212)

66. Ποιες είναι οι μονάδες όγκου και πως συνδέονται μεταξύ τους; (σελ.212)

67. Ποιες μονάδες χρησιμοποιούμε για τη μέτρηση του όγκου των υγρών; (σελ.212)

68. Με τι ισούται ο όγκος ενός πρίσματος; (σελ.213)

69. Με τι ισούται ο όγκος ενός κυλίνδρου; (σελ.213)

70. τι ονομάζεται πυραμίδα και ποια είναι τα στοιχεία της; (σελ.216)

71. Πως ονομάζεται μια πυραμίδα; (σελ.216 & 21 7)

72. Ποια πυραμίδα ονομάζεται κανονική και ποιες είναι οι ιδιότητες της;

73. Πως βρίσκουμε το εμβαδόν της ολικής επιφάνειας μιας πυραμίδας; (σελ.21 7)

74. Ποιο είναι το εμβαδόν της παράπλευρης 57. Ποιες είναι οι δυνατές θέσεις

διαφορετικών επιπέδων; (σελ.202) δύο και ποιο το εμβαδόν της ολικής επιφάνειας

μιας κανονικής πυραμίδας; (σελ.218) 58. Ποιες είναι οι δυνατές θέσεις

διαφορετικών ευθειών; (σελ.202) δύο 75. Με τι ισούται ο όγκος μιας πυραμίδας;

59. Ποιες είναι οι δυνατές θέσεις μιας ευθείας και ενός επιπέδου; (σελ.203)

(σελ.219)

ΕΥΚΛΕΙΔΗΣ Α' 88 τ.4/22

Page 25: Ευκλειδης Α 88

Επαναληπτ ι κές ασκήσε ι ς Άλγεβρας Από τον Μαθηματ ι κό όμ ιλο Εκπα ιδευτηρίων •• Εκπα ιδευτ ι κή Αναγέννηση•• 1. Να λυθούν τις εξισώσεις: Αν Ο η αρχή των αξόνων, Α το σημείο τομής με

χ + 2 + 5 = 3χ - 1 + � τον άξονα χχ ' και Β το σημείο τομής με τον άξονα ί. 3 2 3 yy', να βρεθεί το εμβαδόν του τριγώνου ΑΟΒ. ( ή χ - 1 4 _ 2χ 9 Να υπολογιστεί το εμβαδόν του χωρίου που ορί-

ίί. 5 -2 = -4-+ 2 ζουν η γραφική παράσταση και οι άξονες χχ' ,yy') 6) Μια εταιρία έκανε μείωση μισθών κατά

ίίί. 3χ - χ - 1 + 2χ - 1 _ 1 = 3 (χ - 1 ) + χ - 1 1 5%. Να εκφράσετε τους νέους μισθούς των 3 2 2 6 υπαλλήλων ως συνάρτηση των παλιών. Να 3χ + 1 6χ - 4 σχεδιάσετε τη γραφική παράσταση . Ποιες τι-ίv. -2-- χ = -7- μές δεν έχουν νόημα; 2. .:!:ρεθούν οι κοινές λύσεις των ανισώ-

7) Να uπολσyιστεi Α � ( ( -ητ ( ( - �� }' )'

ί. 4 (χ + 4) + χ + 1 > 2 (4χ + 5) . . . . . και . . . . . -6 (χ - 1 1 ) � 4 (χ - 2) - 3 (χ + 1 ) ' χ + 4 χ - 2 χ - 2 ίί. -- + 2 > --+-- και 6 4 6

χ - 3 χ χ - 4 χ -- � ----2 2 4 χ + 5 χ χ - 3 ίίί. -- -- ;;:: - και 4 3 6 7 - 3χ +� - 2χ < 5(5 - 2χ) - 4 1 2 4 6 4χ - 9 χ - 9 2χ + 6 2χ - 1 5 iv. ----- > και 5 6 3 9 4 - 2χ χ 3 ---- > -(2 - χ) - 1

5 6 4 3. α) Να λυθεί η εξίσωση :

1 3χ + 5 χ + 4 4χ + 1 2χ + 5 -+ -- ---+-- = --12 3 2 6 4 β) Να βρεθεί αν η λύση της εξίσωσης

ανήκει στις λύσεις της παρακάτω α-, 5χ - 4 4χ + 2 νισωσης: -3- > -5-

4. Να λυθεί η εξίσωση: l 2x + 11 = lx + 31 .

Από τη συνάδελφο Μορφοπούλου Μαρία 5) Μ θ ' ' κλ' 1 ' ' ια ευ εια εχει ιση 2" και τεμνει τον α-

ξονα yy ' στο 3 . Να συμπληρωθεί ο πίνακας και να γίνει η γραφική παράσταση

χ -2 1 Υ ο 3

8) Αν Α = ((2ι s .2s( •223 )2ο ι 3

[ 6-14 ·36-4 ],[3 Β = - 2_22 .τ22 να λυθούν οι εξισώσεις

και

α) Αχ = Β β) (Α+Β)χ=Β γ) (Α-Β)χ = 2Α+Β 9) Να λυθούν οι ανισώσεις :

χ 1 - 2χ 4 - χ 2χ + 3 α) -- -- < 2 --- β) -3 < -- < 5 1 0 5 2 4 Από το Μαθηματικό εργαστή ρι 4ου Γυμνα­σίου Ν. Σμύρνης 10) Δίνεται η παράσταση: Α=4(2χ-�-�3y-Ίχ}-12χ α) Να aπλοποιήσετε την παράσταση β) Να υπολογίσετε την τιμή της παράστασης

1 αν x - y = --

5 γ) Να υπολογίσετε την παράσταση :

Β = (-α -�) : 20 1 2, α * Ο α -α

δ) για τις τιμές των Α και Β να λύσετε την ε­ξίσωση Α(χ - 4) = Β - 2χ . 1 1) α) Να λύσετε τις εξισώσεις -4 - [2 - (χ - 5)] = 0 και 6 + [y - (3 - 2)] = -5 β) Για τα χ και y που βρήκατε να υπολογίσετε την τιμή της παράστασης:

Α = (χ + y) 20 1 2 + (-χ - y) 20 1 3 γ) Για Α=- 1 Να λύσετε την εξίσωση :

3 _ 5 - ω = 2 - 3 (ω - 1) Α - 9 Α + 1 1

ΕΥΚΛΕΙΔΗΣ Α' 88 τ.4/23

Page 26: Ευκλειδης Α 88

Επαναληπτ ι κές '

ασκησε ι ς Γεωμετρίας ======================= Τουρναβίτης Στέργιος

1 . Στο παρακάτω σχήμα υπάρχουν δύο ορθογώνια τρίγωνα ΑΟΒ, ΑΒΔ. Θα μπορούσαν να μας ζητήσουν απλά να βρούμε τα μήκη των πλευρών ΑΒ, ΑΔ.

Α

6

Προσπαθήστε να λύσετε την άσκηση με βήματα, συμπληρώνοντας τα κενά. Παρατηρούμε ότι η υποτείνουσα του ορθογωνίου τριγώνου . . . . . . . . . είναι και κάθετη πλευρά για το ορθογώνιο τρίγωνο . . . . . . . . . . . . Εφαρμόζουμε το . . . . . . . . . . . . . . . . . . . . θεώρημα, στο ορθογώνιο

ΑΟΒ, κι έχουμε: 2_ ο - . . . . . . . . . . . . . . . . . . . . . . . . . . . . . . . . . . . . . . . .

ο2= . . . . . . . . . . . . . . . . . . . . . . . . . . . . . . . . . . . . . . . . 2_ ο - . . . . . . . . . . . . . . .

o = ..J Ο υπολογισμός του μήκους της ΑΒ «μεταφέρεται» στο 2° ορθογώνιο, αυτή τη φορά ως μήκος κάθετης

πλευράς. Εδώ βρίσκεται η κίνηση που λέγαμε προηγουμένως . . .

Πάλι με την βοήθεια του . . . . . . . . . . . . . . . . . . . . . . . . . . . . . . . . . . Θεωρήματος, έχουμε:

β2 = . . . . . . . . . . . . . . . . . . . . . . . . . . . . . . . . . . , β2 = . . . . . . . . . . . . . . . , β2 = . . . . . . . . . . . . . . . και β = ...[ 2. Μία φάρμα βρίσκεται σ' ένα αγροτεμάχιο ασυνήθιστου

σχήματος, χωρισμένο σε δύο μικρότερα ορθογώνια τρίγωνα, όπως δείχνει η διπλανή εικόνα.

Μπορείτε να βοηθήσετε τον κτηνοτρόφο, να υπολογίσει το συνολικό μήκος του συρματοπλέγματος του φράκτη (όχι του ενδιάμεσου) που περιβάλλει την φάρμα;

(Απάντηση: μήκος περιμέτρου = 99, 7m ) 3. Μία διατομή ενός σωλήνα νερού έχει το σχήμα του

ημικυκλίου ακτίνας 4cm, όπως φαίνεται στο διπλανό σχήμα. Η επιφάνεια του νερού μέσα στον σωλήνα «προβάλλεται» στο ευθύγραμμο τμήμα ΑΒ του οποίου το μήκος είναι 5cm. Να βρείτε το μέγιστο ύψος h, του νερού στον σωλήνα. (Απάντηση: h = 0, 9cm )

Ένας ελαιοχρωματιστής θέλει να «φθάσει» τη στέγη ενός σπιτιού που έχει 1 5m ύψος. Διαθέτει μία σκάλα μήκους 20m. Πόσο μακριά από το σπίτι θα απέχει η βάση της σκάλας; (Απάντηση : χ ::: 1 3, 2m )

5. Η Ελένη «πετούσε» τον αετό της μ' ένα σχοινί μήκους 55 m. Αυτή έχει ύψος 1 ,5m και στεκόταν σε απόσταση 50m από ένα δέντρο. Για μεγάλη της ατυχία ο αετός «μπλέχτηκε» στην κορυφή του δέντρου. Μπορείτε να την βοηθήσετε να υπολογίσει το ύψος του; (Απάντηση : δ = 24, 5m ) 6. Ένα πλοίο ξεκινάει από το λιμάνι 0(0,0) και κατευθύνεται 7km

ΕΥΚΛΕΙΔΗΣ Α' 88 τ.4/24

Page 27: Ευκλειδης Α 88

------------- Ασκήσεις Μαθηματικού Ομίλου-----------­

δυτικά και 24km νότια όπως δείχνει το διπλανό σχήμα. Να βρείτε την απόσταση (ΟΑ) από το λιμάνι.

(Απάντηση : (OA)=25km) 7. Μία εταιρεία τηλεφωνίας προκειμένου να βρεί τη

συντομότερη απόσταση (ΒΓ) μεταξύ δύο υποκα­

, !

της, i u · ταστημάτων κατασκεύασε το � Ι διπλανό σχήμα. Ποιο είναι το μήκος αυτού του συντομότερου δρόμου ΒΓ;

(Απάντηση : (BΓ)=1 3km) 8. Η είσοδος μιας σκηνής

είναι ένα ισοσκελές τρίγωνο. Το κοντάρι που στηρίζει την σκηνή έχει ύψος 1 ,2m και η απόσταση από την βάση του κονταριού μέχρι το

I �� I

κάλυμμα είναι 56cm. Βρείτε: �1 α) το μήκος του καλύμματος από την κορυφή του κονταριού μέχρι το έδαφος.

= (6 , 1 3)

β) Το εμβαδόν της «πόρτας» εισόδου της σκηνής. (Απάντηση: χ = 1, 32m, Ε = Ο , 672m2 ) 9. Ένα μοντέρνο γλυπτό από γυαλί είναι όρθιο, στο

σχήμα δύο αιχμηρών βουνών. Για τον υπολογισμό της συνολικής επιφάνειας του γυαλιού, το σχεδιάσαμε στο επίπεδο ενός ορθοκανονικού συστήματος συντεταγμένων με την μονάδα μέτρησης και στους δύο άξονες να αντιστοιχεί στο 1 m. Επιπλέον το χωρίσαμε σε δύο τρίγωνα ΤΙ ' τ2 και δύο τραπέζια Τρl ' Τρ2 αντίστοιχα. Μπορείτε να συμπληρώσετε τα παρακάτω κενά; α) Εμβαδόν τριγώνου Τ1 = . . . . . . . . . . . . . . . . . . . . . . . . . . . . . . . . . . . . . . . . . . . . . . . . β) Εμβαδόν τριγώνου Τ2 = . . . . . . . . . . . . . . . . . . . . . . . . . . . . . . . . . . . . . . . . . . . . . . . . γ) Εμβαδόν τραπεζίου Τρ1 = . . . . . . . . . . . . . . . . . . . . . . . . . . . . . . . . . . . . . . . . . . . . . . . . δ) Εμβαδόν τραπεζίου Τρ2 = . . . . . . . . . . . . . . . . . . . . . . . . . . . . . . . . . . . .

ε) Εμβαδόν συνολικής επιφάνειας γλυπτού Ε= . . . . . . . . . . . . . . . . . . . . . . . . . . . . . . . . . . . . . . . . . . . . . . . . 1 0. Ένα εξάρτημα από ένα παιδικό παιχνίδι είναι φτιαγμένο

από πλαστικό σε σχήμα οκταγώνου με ένα τετράγωνο στο κέντρο να «λείπει» όπως δείχνει το διπλανό σχήμα. Μπορείτε να βοηθήσετε τον κατασκευαστή να υπολογίσει την επιφάνεια του πλαστικού που θα χρειαστεί για την κατασκευή τεσσάρων από αυτά τα εξαρτήματα;

α) Μετρώντας τα τετραγωνάκια πλευράς 1 cm. β) Με οποιονδήποτε άλλον τρόπο.

ΕΥΚΛΕΙΔΗΣ Α ' 88 τ.4/25

Page 28: Ευκλειδης Α 88

------------ Ασκήσεις Μαθηματικού Ομίλου------------

Από τον Μαθηματικό όμιλο Εκπαιδευτηρίων 1 . Να υπολογίσετε την τιμή των παραστάσεων:

" Εκπαιδευτική Αναγέννηση "

Α = εφ45 . .J9 · 2220 1 3 3 + ημ60 - συν30

Β = ( 3ημ60 - 2συν30) · JJ9 2. Στο παρακάτω σχήμα δίνεται κύκλος κέντρου Α και Β, Γ

δυο σημεία του κύκλου, έτσι ώστε το τρίγωνο ΑΒΓ να είναι ορθογώνιο, με ΑΓ = 1 0 cm. Να βρείτε:

ί . τις γωνίες Β, Γ , ίί. την περίμετρο του τριγώνου ΑΒΓ,

ίίί. το εμβαδό της γραμμοσκιασμένης επιφάνειας.

Β

Α Δ ...--�-------

3. Ένας μαθητής θέλει να γεμίσει με αυτοκόλλητα την επιφάνεια του ορθογώνιου πλαισίου ΑΒΓ Δ. Έχει ήδη τοποθετήσει αυτοκόλλητα στα τεταρτημόρια που φαίνονται στο σχήμα. Αν η πλευρά ΒΓ έχει μήκος 1 5cm να βρεθεί το εμβαδόν της ακάλυπτης επιφάνειας.

4. Στο τραπέζιο ΑΒΓ Δ, οι γωνίες Β και Γ (Β = Γ = 90°) είναι ορθές � και το ΑΕ ύψος. Επίσης ισχύει για τις γωνίες ΕΑΓ = 45° και � � Δ = 2ΔΑΕ . α) Να βρείτε τις πλευρές ΑΕ, ΕΓ, αν ΑΓ= J72 cm. β) Να βρείτε το εμβαδό του τραπεζίου ΑΒΓΔ και του τριγώνου ΑΔΓ.

Α Β

Δ Ε r

Β

5. α) Να λυθούν οι εξισώσεις: i) 2� · χ = 2 - 3χ , ii) 4y + J4y2 - 7 = - l Oy - 5

Γ

β) Αν ΑΒΓ ορθογώνιο τρίγωνο (Α = 90°) και οι πλευρές ΑΒ, ΑΓ έχουν αντίστοιχα μέτρο ΑΒ = χ2 + 1 , ΑΓ = y3 + 1 , όπου χ και y οι λύσεις των παραπάνω εξισώσεων, να βρεθεί το μέτρο της πλευράς ΒΓ.

6. Δίνεται ορθογώνιο τρίγωνο ΑΒΓ (Α = 90°) με ΑΒ = 4 cm και Γ = 30° . Να βρεθούν τα μήκη των πλευρών ΑΓ και ΒΓ και το εμβαδόν του τριγώνου.

Β

Α Γ

ΕΥΚΛΕΙΔΗΣ Α ' 88 τ.4/26

Page 29: Ευκλειδης Α 88

------------ Ασκήσεις Μαθηματικού Ομίλου------------

7 . Δίνεται ορθογώνιο τρίγωνο ΑΒΓ (Α=90°) με

( 2 -F2) · ( 2 +F2) ΑΓ = . Να βρεθούν τα μήκη των πλευρών και τα μέτρα των γωνιών.

8

και

8 . Δίνεται ορθογώνιο τρίγωνο ΑΒΓ (Α = 90°) με ΑΓ = 5J3 cm και υποτείνουσα 1 Ocm. Να βρεθεί η πλευρά ΑΒ και οι γωνίες Β και Γ.

9 . Να αποδείξετε το Πυθαγόρειο θεώρημα για τις πλευρές α, β και γ, χρησιμοποιώντας εμβαδά των επίπεδων σχημάτων του παρακάτω σχήματος.

I Ο. Σε τετράπλευρο ΑΒΓ Δ οι διαγώνιες ΑΓ και ΒΔ τέμνονται στο σημείο Ο και είναι κάθετες μεταξύ τους. Να αποδείξετε ότι: α) ΟΑ 2 + ΟΒ2 + ΟΔ2 + ΟΓ2 = ΑΒ2 + ΓΔ2 β) ΑΒ2 + ΓΔ2 = ΑΔ2 + ΒΓ2

Ι ι . Μια πινακίδα είναι ένα ισοσκελές τρίγωνο ΑΒΓ με πλευρά βάσης 40 cm και η γωνία απέναντι από τη βάση, έχει μέτρο 90 = . Α ν η κορυφή της πινακίδας απέχει από το έδαφος 1m, να υπολογίσετε την απόσταση από την κορυφή του τριγώνου έως το έδαφος .

1 2 . Δίνεται ευθεία (ε ι ) : y = 2χ + 4. Να βρεθούν:

β β

i.H ευθεία (εz) η οποία είναι παράλληλη στην (ε ι ) ((εz) // (ει ) ) και η οποία διέρχεται από την αρχή των αξόνων.

i i .To σημείο τομής Α της ευθείας (ε ι ) με τον άξονα y'y. i i i.H ευθεία (ε3) η οποία είναι παράλληλη στον χ 'χ ((ε3) // χ 'χ) και διέρχεται από το

σημείο Α. iv .To σημείο τομής Β των ευθειών (ε2) και (ε3) . ν Να σχεδιαστεί κύκλος με κέντρο το Β και ακτίνα την ΒΑ. v i .To εμβαδόν της περιοχής που περικλείετε από τα σημεία Ο, Α, Γ.

1 3 . Δίνονται οι ευθείες (ε ι ) : y = (6ω - 2)χ + κ και (εz) : y = 2Ν · χ . Να βρεθούν: i.H τιμή του θετικού ρητού αριθμού ω, ώστε η ευθεία (ε2) να είναι παράλληλη στην (ει ) ((εz) I/ (ε ι) ) .

ί ίΝα βρεθεί ο πραγματικός αριθμός κ ώστε η ευθεία (ε ι ) να διέρχεται από το σημείο Α(2, 8) .

ί ί ί . Να βρεθεί η εξίσωση της ευθείας (ε3), η οποία είναι κάθετη στην (ε2) και διέρχεται από το σημείο Β(2 , 2).

ί ' 'Να βρεθεί το εμβαδόν του πολυγώνου που δημιουργείται από τα σημεία ΟΑΓΔ, όπου Γ το σημείο τομής της (ε ι ) με τον χ 'χ και Δ το σημείο τομής της (ει ) με την ευθεία (ε3) .

1 4 . Δίνεται ημικύκλιο κέντρου Ο, διαμέτρου ΒΓ = 2 α.Jl8 cm και ισοσκελές τρίγωνο ΑΒΓ, με το Α να είναι σημείο του κύκλου. Να υπολογιστούν:

ί . Οι πλευρές του τριγώνου συναρτήσει του α. ii. Το εμβαδόν του τριγώνου συναρτήσει του α.

ί ί ί . Το εμβαδόν του ημικυκλίου συναρτήσει του α. iv. Αν το εμβαδόν της καμπυλόγραμμης επιφάνειας που σχηματίζεται από τις πλευρές του

τριγώνου και το ημικύκλιο είναι Ε = 8(2π - 1 ) cm2 να βρεθεί το μέτρο του μήκους της ακτίνας.

ΕΥΚΛΕΙΔΗΣ Α' 88 τ.4/27

Page 30: Ευκλειδης Α 88

Τάξη

Φύλλο εργασίας yια τη συνάρτηση ψ = αχ2 + βχ + y

Με χρήση του λογισμικού Geogebra ========================= Αργύρη Παναγιώτα

Να ανοίξετε το λογισμικό Geogebra και να κατασκευάσετε 3 δρομείς α, β, γ με τιμές από ....:.5 ως 5 ο καθένας και αύξηση 0. 1

Δραστηριότητα 1 η • Να εισάγετε την συνάρτηση α*χ2

• Να εισάγετε την συνάρτηση α*χ2 +β • Να μεταβάλλεται τους δρομείς α και b

Ερώτηση : Τι παρατηρείτε ; Η β λ , - * Λ2 'ζ

, 'δ , ' ξ παρα ο :η ψ- α χ μετατοπι εται κατα . . . . • . • . β μονα ες . . . . . . . . . . . . . . . . προς τα πανω στον α ονα

ψψ', αν . . . . β>Ο • • • • • • και κατά . . . . . . . . . . . . . β μονάδες . . . . . . . . . . . . προς τα κάτω αν . . . . . . . . . . . . . β<Ο .... . . . . . . . . . .

Ερώτηση : Τι παρατηρείτε ; Ποια τα νέα χαρακτηριστικά της παραβολής ( κορυφή, μέγιστη ή ελάχιστη τιμή , άξονας συμμετρίας ) ; . . . . . . άξονας. συμμετρίας . . : ψψ ' . . . . . . . . . . . . . . . . . . . . . . . . . . . . . . . . . . . . . . . . . . . . . . . . . . . . . . . . . . . . . . . . . . . . . . . . . . . . . . . . . . . . . . . . . . . . . . . . . . . . . . . . . . . . . . . . . . . . . . . . κοpυφή . . . . . . Λ (Ο,β) . . . . . . . . . . . . . . . . . . . . . . . . . . . . . . . . . . . . . . . . . . . . . . . . . . . . . . . . . . . . . . . . . . . . . . . . . . . . . . . . . . . . . . . . . . . . . . . . . . . . . . . . . . . . . . . . . . . . • . . . . . • . . . • . . . παίρνει .. μέγιστη.τιμή.στη θέση για.χ=Ο την ψ=β αν α<Ο .... . παίρνει..ελάχιστη. τιμή.στη θέση για.χ=Ο την ψ=β αν α>Ο Δραστηριότητα 2η

• Να εισάγετε την συνάρτηση α*(χ+γ)Λ2 • Να μεταβάλλεται τους δρομείς α και γ

Ερώτηση : Τι παρατηρείτε ; • Η παραβολή ψ= α*χΛ2 μετατοπίζεται κατά . . . . . . γ μονάδες . . . . . . . . . . προς τα δεξιά στον άξονα

χχ' • • • • • • • • • • . αν γ<Ο . . . • . . . . και κατά . • • . . γ μονάδες . . . • . . . . . . προς τα αριστερά στον άξονα , n αν. . . ,>Ο .... . . . . . . . . . . . .

• Ερώτηση : Τι παρατηρείτε ; Ποια τα νέα χαρακτηριστικά της παραβολής (κορυφή, μέγιστη ή ελάχιστη τιμή , άξονας συμμετρίας ) ;

. . . . . . . . Άξονα ς. συμμετpίας. η.ευθεία. χ=γ • • • . . . . . . . . . . . . • . • . . . . . • . . . . • . . • . . . . . . . . . . . . . . . . . . . . . . . . . . . . . • . . • . . . • . • . . . . . . . . . . • . • . • . • . . . . .

. . • . . . κορυφή .. Α(γ,Ο) . . . . . . . • • • • • παίρνει .. μέγιστη.τιμή.στη θέση για.χ=γ την ψ=Ο αν α<Ο

.... . παίρνει .. ελάχιστη. τιμή.στη θέση για.χ=γ την ψ=Ο αν α>Ο

ΕΥΚΛΕΙΔΗΣ Α' 88 τ.4/28

Page 31: Ευκλειδης Α 88

----------- Φύλλο εργασίας για τη συνάρτηση φ--αi+βχ+γ ---------­

(Για την αναγνώριση και κυρίως για την μαθηματική αποτύπωση των χαρακτηριστικών της παραβολής προτείνεται στους μαθητές : Να βρείτε το σημείο τομής Α της γραφικής παράστασης με τον άξονα χχ ' και να φέρετε κάθετη ευθεία στον άξονα χχ ' στο σημείο Α. Να μετακινήσετε το δρομέα c. Τι παρατηρείτε ;)

Δραστηριότητα 3η • Να εισάγετε την συνάρτηση α*(χ+γ)Λ2 +β Ερώτηση : Τι παρατηρείτε ; Ποια είναι η θέση της νέας γραφικής παράστασης σε σχέση με την f, την g και την h. ( Θυμηθείτε Η συνάρτηση α*(χ+γ)Λ2 +β μετά την εκτέλεση των πράξεων παίρνει την μορφή ενός τριωνύμου δευτέρου βαθμού ) Ορισμός Κάθε συνάρτηση της μορφής ψ= α*χΛ2 +β*χ + γ , ονομάζεται τετραγωνική. • Σε ένα νέο φύλλο εργασίας Geogebra, να κατασκευάσετε 3 δρομείς α, β, γ με τιμές από -5

ως 5 ο καθένας και αύξηση Ο. l και να εισάγετε την συνάρτηση α*χΛ2 +β*χ + γ ,

; § -,v ,. :ι & •

f

ΕΥΚΛΕΙΔΗΣ Α' 88 τ.4/29

Page 32: Ευκλειδης Α 88

---------- Φύλλο εργασίας για τη συνάρτηση ψ=αχ2+βχ+γ ----------

Μελέτη των χαρακτηριστικών της παραβολής α*χΛ2 +β*χ + γ:

• Επιλέξετε κάποιες τιμές για τα α, β και γ και να εισάγετε και να υπολογίσετε την τιμή d=βΛ2-4*α*γ

• Επιλέξετε κάποιες τιμές για τα α, β και γ και να εισάγετε και να υπολογίζεται την τιμή -β/(2*α) που ονομάζεται c

• Να εισάγετε την ευθεία χ= c και το σημείο ( c, -d I (4*α))

Ερώτηση : Να βρείτε το σημείο τομής της ευθείας x=c και της παραβολής.

Ερώτηση : Τι παρατηρείτε για το σημείο τομής;

. . . . . . . . Είναι η κορυφή της . . παραβολής με συντεταγμένες ( c, -d I (4 *a)) . . . . . . . . . . . . . . . . .

Να μετακινήσετε τους δρομείς α, b, c για τις διάφορες τιμές που θα επιλέξετε και να επιβεβαιώσετε την παρατήρηση σας.

Συμπέρασμα : Η παραβολή ψ= α*χΛ2 +b*x + c έχει άξονα συμμετρίας την ευθεία . . . . . . χ=c=-β12α . . . . . . . . . . . . . . .

• Έχει κορυφή το σημείο . . . . . ( c, -d I (4 *a)). = (-β12 α, ( βΛ2-4 *α *γ)14 '�α )

Αν . . . . . . . α<Ο . . . . . . . παίρνει γιαχ= . . . . . . . . . . . -β12α . . . . . . . . . . . . . . . . . . . μέγιστη τιμή την ψ= . . . . -dl4 *a . . . . . . . . . . . . . . . . . Αν . . . . . . . α<Ο . . . . . . . παίρνει για χ= . . . . . . . . . . . -β12α . . . . . . . . . . . . . . . . . . ελάχιστη τιμή την ψ= . . . . -dl4 *lι . . . . . . . . . . . . . . . . .

Ερώτηση : Απο την γραφική παράσταση της συνάρτησης ψ= α*χΛ2 +β*χ + γ μπορώ να λύσω την εξίσωση 2ou βαθμού α*χΛ2 +β*χ + γ=Ο ;

.Σ , , ' ): , . . . . . . . . . . . . . . . . . . . . ημεια τομης με τον α.,.ονα χχ . . . . . . . . . . . . . . . . . . . . . . . . . . . . . . . . . . . . . . .

ΑΣΚΗΣΕΙΣ ΓΙΑ ΛΥΣΗ 1 . Η παραβολή ψ = χ2 - 4χ + λ διέρχεται απο το σημείο Α(3 , -8) .

Α) Να βρείτε τον αριθμό λ

Β) Να βρείτε την κορυφή και τον άξονα συμμετρίας

Γ) Να βρείτε τα σημεία τομής της παραβολής με τους άξονες

Δ) Να συμπληρώσετε τον πίνακα τιμών της παραβολής

2. Η παραβολή ψ = 4χ2 + β χ + r παίρνει ελάχιστη τιμή ψ=-9 όταν χ=-1

Α) Να βρείτε τους αριθμούς β,γ

Β) Τα σημεία τομής της παραβολής με τους άξονες.

3 . Η παραβολή ψ = χ2 + β χ - 7 έχει άξονα συμμετρίας την ευθεία χ = -3 . Να βρείτε:

Α) Τον αριθμό β

Β) Τα σημεία τομής Α,Β και Γ της παραβολής με τους άξονες.

Γ) Το εμβαδόν του τριγώνου ΑΒΓ.

ΕΥΚΛΕΙΔΗΣ Α' 88 τ.4/30

Page 33: Ευκλειδης Α 88

Γ' ΓΥΜΝΑΣΙΟΥ ΟΛΗ Η ΘΕΩΡΙΑ ΣΕ 82 ΕΡΩ ΤΗΣΕΙΣ!

ΜΕΡΟΣ Α� ΑΛfΕΒΡΑ

ΕΦ�ΛΑΙΟ 0 Αλγεβρικές Παραστάσεις A. l . l

1. Τι ονομάζετε δύναμη αν με βάση τον πραγματικό α και εκθέτη το φυσικό ν> 1 ; (σελ. 1 7)

2. Ποιες είναι οι ιδιότητές των δυνάμεων με βάση πραγματικό και εκθέτη ακέραιο; (σελ. 1 7)

3. Τι ονομάζεται τετραγωνική ρίζα θετικού αριθμού α; (σελ.20)

4. Ποιες είναι οι ιδιότητές των ριζών; (σελ.20)

5. Α ν α ;;:: Ο και β ;;:: Ο να αποδείξετε ότι,

J;. · .jβ = ra:P (σελ. 21)

6. Αν α ;, Ο και β > Ο να αποδείξετε ότι. � � J* (σελ.21)

A. l . 2

7. Τι ονομάζεται αλγεβρική παράσταση; (σελ.25)

8. Τι ονομάζεται αριθμητική τιμή αλγεβρικής παράστασης; (σελ.25)

9. Πότε μια αλγεβρική παράσταση ονομάζε­

ται ακέραια; (σελ.25) 10. Τι ονομάζεται μονώνυμο και ποια τά μέρη

από τα οποία αποτελείται; (σελ.26) 1 1 . Ποια μονώνυμα ονομάζονται όμcjια;

(σελ.26) 12. Ποια μονώνυμα ονομάζονται ίσα και ποια

αντίθετα; (σελ.26) 13. Τι ονομάζεται βαθμός μονωνύμου ως προς

μία μεταβλητή του; (σελ.26) 14. Τι ονομάζουμε σταθερό και τι μηδενικό

μονώνυμο και πο�ος ο βαθμός τους; (σελ.26)

15. Πως ορίζεται το άθροισμα ομοίων μονω­νύμων; (σελ. 30)

16. Τι ονομάζεται αναγωγή ομοίων όρων; (σελ. 34)

17. Πως ορίζεται το γινόμενο μονωνύμων; (σελ.30)

A. l . 3 18. Τι ονομάζεται πολυώνυμο; (σελ.33) 19. Τι ονομάζεται βαθμός ενός πολυωνύμου

ως προς μία μεταβλητή του; (σελ. 33) 20. Τι ονομάζουμε σταθερό και τι μηδενικό

πολυώνυμο και ποιος ο βαθμός τους; (σελ. 33)

A. l . 4

21 . Πως πολλαπλασιάζουμε: α. Μονώνυμο με πολυώνυμο; β. Πολυώνυμο με πολυώνυμο; (σελ. 38)

A. l . 5 22. Τι ονομάζεται ταυτότητα; (σελ. 42) 23. Να αποδείξετε τις ταυτότητες: ί. (α +β)2 = α2 + 2αβ + β2

ίί. (α -β)2 = α2 - 2αβ + β2

ίίί. (α + β)3 = α3 + 3α2β + 3αβ2 + β3

iv. (α -β)3 = α3 - 3α2β + 3αβ2 - β3

ν. (α -β)( α + β) = α2 - β2 (σελ. 43 & 44)

A. l . 6

24. τι ονομάζεται παραγοντοποίηση; (σελ. 53) 25. Ποιες είναι οι χαρακτηριστικές περιπτώ­

σεις παραγοντοποίησης; (σελ.54,55, 56, 57) Α. 1 . 8 26. τι ονομάζεται Ελάχιστο Κοινό Πολλα­

πλάσιο (Ε.ΚΠ.) και τι Μέγιστος Κοινός Δι­αφέτης (Μ.ΚΔ.) δύο ή περισσοτέρων αλ γε-

ΕΥΚΛΕΙΔΗΣ Α' 88 τ.4/31

Page 34: Ευκλειδης Α 88

----------Γ ΓΥΜΝΑΣΙΟΥ ΟΛΗ Η ΘΕΩΡΙΑ ΣΕ 82 ΕΡΩΤΗΣΕΙΣ! ----------

βρικών παραστάσεων που έχουν αναλυθεί σε γινόμενο πρώτων παραγόντων; (σ'ελ. 68)

Α. 1 . 9

27. Πότε μια αλγεβρική παράσταση ονομάζε­ται ρητή ; (σελ. 71)

28. Πότε μια αλγεβρική παράσταση ορίζεται;

(σελ. 71) 29. Πότε μια ρητή αλγεβρική παράσταση μπο­

ρεί να απλοποιηθεί; (σελ. 71)

Α. 1 . 1 0

30. Πως κάνουμε πράξεις με ρητές αλγεβρικές

παραστάσεις; (σελ. 75, 76, 78)

Εξισώσεις Α νισώσεις Α. 2. 2 3 1 . Τι ονομάζεται εξίσωση2ου βαθμού, με έ­

ναν άγνωστο; (σελ.90) 32. Να αποδείξετε τον τύπο που δίνει την λύ­

ση της δευτεροβάθμιας εξίσωσης

αχ2+βχ+γ=Ο με α, β, γ πραγματικούς αριθ­μούς και α :;tO. (σελ. 94)

33. Πότε μία εξίσωση δευτέρου βαθμού: α. έχει δύο άνισες ρίζες; β. έχει μια διπλή ρίζα; γ. δεν έχει ρίζες; (σελ.94)

34. Πως παραγοντοποιείται το τριώνυμο

αχ2+βχ+γ όταν η εξίσωση αχ2+βχ+γ=Ο με

α � Ο έχει λύσεις τις ρ ι , ρ2; (σελ.96) Α. 2. 4 35. Τι ονομάζεται κλασματική εξίσωση και

πότε ορίζεται αυτή ; (σελ. 103) Α. 2. 5

36. Πως συγκρίνουμε (διατάσσουμε) δύο πραγματικούς αριθμούς; (σελ. 1 10)

37. Τι ονομάζεται ανισότητα και ποια τα χα-

ρακτηριστικά της; 38. Ποιες είναι οι ιδιότητες της διάταξης;

1 1 1 & 112)

ξισώσεων Α. 3. 1

Συστήματα Γραμμικών Ε-

39. Τι ονομάζεται γραμμική εξίσωση με δύο αγνώστους και τι λύση της; ( σελ. 12 2)

40. Πως παριστάνεται γραφικά κάθε εξίσωση

της μορφής αχ+βy=γ με α�ο ή β�Ο και τι

ισχύει γι' αυτή ; ( σελ. 12 3) 41. Τι παριστάνουν οι εξισώσεις;

α. y = k με k � O β. y = Ο (σελ. 123)

γ. χ = k με k � Ο δ. χ = Ο;

42. Πως βρίσκουμε τις τομές μιας ευθείας

αχ+βy=γ με α�ο και β�Ο με τους άξονες

χ'χκm y'y; (σελ. 125)

Α. 3. 2 43. Τι ονομάζεται:

α. Γραμμικό σύστημα δύο εξισώσεων με δύο αγνώστους χ και y; (σελ. 128) β. Λύση γραμμικού συστήματος δύο εξι­σώσεων με δύο αγνώστους χ και y; (σελ. 128) γ. Επίλυση γραμμικού συστήματος δύο εξισώσεων με δύο αγνώστους χ και y; (σελ. 128)

44. Πως γίνεται η γραφική επίλυση γραμμικού συστήματος δύο εξισώσεων με δύο αγνώ­στους χ και y και πότε αυτό έχει μία λύση, είναι αδύνατο, είναι αόριστο; (σελ. 129)

Α. 4. 1 'ζ

. 2 45. Τι γνωρι εται για την συναρτηση y = αχ

με α > Ο; (σελ. 145) 46. Τι γνωρίζεται για την συνάρτηση y = αχ2

με α < Ο; (σελ. 145)

Α. 4. 2 47. Ποια συνάρτηση ονομάζεται τετραγωνική;

(σελ. 150) 48. Τι γνωρίζεται για τη συνάρτηση

�- ...... !-{�""'"""' με α :;t Ο ; (σελ. 151) Πιθανότητες

49. Τι είναι το σύνολο; (σελ. 160) 50. Πως μπορεί παρασταθεί ένα σύνολο;

(σελ. 160 & 161) 51. Πότε δύο σύνολα λέγονται ίσα; (σελ. 1 61) 52. Πότε ένα σύνολο Α ονομάζεται υποσύνολο

ΕΥΚΛΕΙΔΗΣ Α' 88 τ.4/32

Page 35: Ευκλειδης Α 88

----------Γ ΓΥΜΝΑΣΙΟΥ ΟΛΗ Η ΘΕΩΡΙΑ ΣΕ 82 ΕΡΩΤΗΣΕΙΣ! ----------

ενός συνόλου Β; ( σελ. 161) 53. Τι ονομάζεται κενό σύνολο και πως συμ­

βολίζεται; ( σελ. 162) 54. Τι ονομάζεται πείραμα τύχης; (σελ. 167) 55. Τι ονομάζεται δειγματικός χώρος ενός πει­

ράματος τύχης και πως συμβολίζεται; (σελ. 167)

56. Τι ονομάζεται ενδεχόμενο ενός πειράματος τύχης και πότε αυτό πραγματοποιείται; (σελ. 169)

57. Ποιο ενδεχόμενο ονομάζεται βέβαιο και ποιο αδύνατο σε ένα πειράματος τύχης; (σελ. 169)

58. Πότε δύο .ενδεχόμενα Α και Β ενός πειρά­ματος τύχης ονομάζονται ασυμβίβαστα; (σελ. 1 70)

59. Τι ονομάζεται πιθανότητα Ρ(Α) ενός ενδε"" χόμενου Α σε ένα πείραμα τύχης με ισοπί­θανα αποτελέσματα και ποιες οι ιδιότητες της; (σελ. 1 74 & 1 75)

ΓΕΩΜ ΕΤΡΙΑ - ΤΡΙ ΓΩΝΟΜ ΕΤΡΙΑ ΚΕΦΑΛΑ Ι Ο ι ο Γεωμετρία Β. ι . ι 60. Ποια τα κύρια στοιχεία του τριγώνου;

τμημάτων και με τι ισούται; (σελ.200) 69. Πότε τα ευθύγραμμα τμήματα α, γ εΜχι σ:νάJ.JJγα

προς τα ευθύγραμμα τμήματα β, δ; (σελ.201) 70. Ποιες είναι οι σημανnκότερες ιδιότητες των

αναλογιών; (σελ.201) 71. Να αποδείξετε ότι αν από το μέσο μιας

πλευράς ενός τριγώνου φέρουμε παράλ­ληλη προς μία άλλη πλευρά του, αυτή διέρχεται και από το μέσο της τρίτης πλευράς. (σελ.202)

Β. 1 . 5 72. Πότε δύο πολύγωνα λέγονται όμοια;

(σελ.215) 73. Ποιες προτάσεις προκύπτουν από τον ορι­

σμό της ομοιότητα δύο πολυγώνων; (σελ.216)

74. Πότε δύο τρίγωνα λέγονται όμοια; (σελ.220)

75. Πότε δύο τρίγωνα είναι όμοια; (Κριτήριο ομοιότητας τριγώνων) (σελ.220)

Β. 1 . 6 76. Με τι ισούται ο λόγος των εμβαδών δύο

ομοίων σχημάτων; (σελ.226)

Τριγωνομετρία (σελ. 186) Β. 2 . ι

61. Ποια είναι τα είδη των τριγώνων ως προς

τις πλευρές, και ως προς τις γωνίες τους; (σελ. 186 & 187)

62. Τι ονομάζεται διάμεσος, διχοτόμος, ύψος, τριγώνου; (σελ. 187)

63. Πότε δύο τρίγωνα λέγονται ίσα; (σελ. 187) 64. Πότε δύο τρίγωνα είναι ίσα; (Κριτήρια ι­

σότητας τριγώνων) (σελ. 188 & 189) 65. Πότε δύο ορθογώνια τρίγωνα είναι ίσα;

(Κριτήρια ισότητας ορθογωνίων τριγώ­νων) (σελ. 190)

66. Ποια είναι η χαρακτηριστική ιδιότητα των σημείων της μεσοκαθέτου ευθυγράμμου τμήματος; (σελ. 192)

67. Ποια είναι η χαρακτηριστική ιδιότητα των σημείων της διχοτόμου μιας γωνίας; (σελ. 192)

Β. ι . 2 68. Τι ονομάζεται λόγος δύο ευθυγράμμων

77. Πως ορίζονται οι τριγωνομετρικοί αριθμοί μιας οποιασδήποτε γωνίας; (σελ 233).

78. Ποιοι οι τριγωνομετρικοί αριθμοί μιας γω­νίας ω=0° ή ω=90° ή ω=1 80°; (σελ.233)

Β. 2 . 2 79. Ποιες σχέσεις συνδέουν τους τριγωνομε­

τρικούς αριθμούς δύο παραπληρωματικών γωνιών; (σελ.23 7)

Β. 2 . 3 80. Να αποδείξετε ότι για μια οποιαδήποτε

γωνία ω ισχύουν οι τύποι: ημ2ω+συν2ω=l

και εφω = ημω . (σελ.240)

συν ω Β. 2 . 4 81. Να διατυπώσετε και να αποδείξετε τον νό­

μο των ημιτόνων. (σελ.244) 82. Να διατυπώσετε και να αποδείξετε τον νό­

μο των συνημιτόνων. (σελ.245)

ΕΥΚΛΕΙΔΗΣ Α' 88 τ.4/33

Page 36: Ευκλειδης Α 88

ΑΣΚΗΣΕΙΣ ΕΠΑΝΑΛΗ ΠΤΙΚΕΣ Γ 'ΓΥΜΝΑΣΙΟΥ ========== Γιώργος Λυμπερόπουλος - Τάσος Μπακάλης - Μαρία Σίσκου 1 . Σημειώστε σωστό (Σ) ή λάθος (Λ) στις παρακάτω προτάσεις:

α. χ2+ 1 > 1 για κάθε πραγματικό αριθμό χ. β. (χ-2)2> Ο , για κάθε πραγματικό αριθμό χ. γ. Αν χ2-7χ+ Ι Ο =Ο τότε χ=2 .

δ. Α ν αχ> β και α:#:Ο , τότε χ> � α

ε. ( α+β)2 = α2+β2 2. Ομοίως και για τις προτάσεις:

( . . . . . ) ( . . . . . ) ( . . . . . )

( . . . . . )

( . . . . . )

α. (α+r)3= α3+β3+3αβ(α+β) ( . . . . . ) β. 4χ -9ψ2= (2χ-3ψ). (2χ+3ψ) ( . . . . . ) γ. Η ευθεία ε: (λ-2)χ+λψ=7 , είναι παράλληλη στην ψ'ψ αν λ=2 ( . . . . . )

3 δ. Α ν α>- η εξίσωση χ2+(2α-1 )χ - α2 + 1 =Ο έχει 2 λύσεις πραγματικές και άνισες( . . . . . )

4 ε. Η ελάχιστη τιμή του τριωνύμου: χ2-3χ+6 , είναι 6 ( . . . . . )

3. Να αποδείξετε, ότι : Αν α+β+γ =Ο , τότε α3+β3+γ3=3αβγ. Αντίστροφα αν α3+β3+γ3=3αβγ , τότε α+β+γ =Ο ,ή α=β=γ.

4 . Στις προτάσεις της στήλης Α , αντιστοιχείστε τη σωστή απάντηση από τη στήλη Β :

ΣΤΗΛΗ Α α. Αν 3χ+7>2χ+9 τότε χ>; β. Αν χ2-5χ+6=0 τότε χ=; γ. Η παραβολή ψ=2χ2-27 περνά από το σημείο . . . .

δ. Οι ακέραιοι αριθμοί που ικανοποιούν τις ανισώσεις 3χ-7>2 και 2χ<7 είναι . . . ε. Το σύστημα 3χ-7ψ=5

χ+ψ=5 έχει λύση το ζεύγος . . . στ. Αν -1<χ.<3 τότε , (χ.-1γ<; ζ. Στο ορθογώνιο τρίγωνο ΑΒΓ αν α=6 και β=3 , τότε Β=;

η. Στο ορθογώνιο τρίγωνο ΑΒΓ αν α= 1 Ο και γ=5 τότε Β=;

θ . Σε τρίγωνο ΑΒΓ τα Δ και Ε που ανήκουν στις ΑΒ και ΑΓ αντίστοιχα ΑΔ ΑΕ 1 ΔΕ είναι τέτοια ώστε - = - = - τότε- =·

ΒΓ ' ΑΒ ΑΓ 3

Ο ' ΑΔ ΑΕ 2 , ΔΕ ι. μοιως αν - = - = - τοτε - =· ΑΒ ΑΓ 3 ΒΓ '

5. Αν η εξίσωση : χ2- 2χ +3α+4=0 , έχει μία λύση τότε: 3 α. α= 4 . β. α=1 γ. α=- δ. α=Ο ε. α= -2 7

6. Το πολυώνυμο 3χ2+2χψ-6χ-4ψ είναι ίσο με :

ΣΤΗΛΗ Β 1 . χ;=-2 και χ=3 2 . 60°

3 . 1 -3

4. (χ=4 ' ψ=5)

5 . 2

6 . χ.=2 ή χ.=3

7 . 2 -3

8 . 30u

9 . (χ=4, ψ= 1 )

1 0. 4

α.( χ2-1 ) . (χ-2ψ) β . (χ+3ψ). (χ-2) γ. (3χ-2ψ). (χ+2) δ .(3χ+2ψ).(χ-2ψ) ε. (χ2+ 1 )(2ψ-3) 7. Η εξίσωση λ2 .(χ-1 )-λ-3=0 για να επαληθεύει το σύστημα: 2χ-ψ=5

χ+2ψ=5

ΕΥΚΛΕΙΔΗΣ Α' 88 τ.4/34

Page 37: Ευκλειδης Α 88

----------- Ασκήσεις Επαναληπτικές Γ Γυμνασίου ----------

Πρέπει : α. λ= 1 β.λ=3 ή λ=-- 1 γ. λ= � ή λ=-- 1 δ. λ=4 ε. λ=2 ή λ= -_!_ 2 2

8. α. Να εκτελέσετε τις διαιρέσεις: 1 . (3χ3+20χ2+ 1 6χ-20) : (χ2+7Χ+1 0) 2 . (2χ4+9χ3+9χ2-8χ-12) : (2χ2+χ-3)

β. Να παραγοντοποιήσετε τα πολυώνυμα: 1 . 3χ3+20χ2+ 1 6χ-20 2. 2χ4+9χ3+9χ2-8χ-12 ' ' 1 1 1 1 1 1 1 20 1 2 9. Να αποδειξετε οτι : α .

( ) = - - -- β . - + - + - + . . . + =

χ χ + 1 χ χ + 1 1 .2 2 . 3 3 .4 20 1 2.20 1 3 20 1 3

10 Ν δ , Α Β , 1 Α Β β 1 Α Β . α προσ ιορισετε τα και ωστε: α. (χ + 1) .(χ + 2)

= χ + 1

+ χ + 2 · χ2 - Sχ + 6

= χ - 3

+ χ - 2

1 1 . Αν χψω=1 , να αποδείξετε ότι: Χ + Ψ + 1

= 1 χψ + χ + 1 ψω + ψ + 1 ωχ + ω + 1

12 . Να αποδείξετε ότι : α. Από όλα τα ορθογώνια παραλληλόγραμμα που έχουν την ίδια περίμετρο, μεγαλύτερο

εμβαδό έχει το τετράγωνο. β. Από όλα τα ορθογώνια που έχουν το ίδιο εμβαδό, μικρότερη περίμετρο έχει το τετράγωνο.

13 . Να λύσετε την εξίσωση : (χ+ 1 0)2+(χ+8)2+ . . . . +(χ+2)2= (χ+9)2+(χ+7)2+ . . . . +(χ+ 1 )2 14. Ρίχνουμε τα ζάρια στο τάβλι 2 φορές , βρείτε τη πιθανότητα:

α. Να φέρουμε και τις δύο φορές εξάρες. β. Να φέρουμε τη μια φορά εξάρες και μια άσσους . γ. Να φέρουμε τη πρώτη φορά εξάρες και τη δεύτερη φορά άσσους. δ. Να φέρουμε μια φορά τουλάχιστον άθροισμα 7 .

1 5. Σε ένα Γυμνάσιο φοιτούν 250 μαθητές εκ των οποίων οι 1 1 5 είναι αγόρια. Η τάξη Β' έχει 1 0 μαθητές περισσότερους από τη Γ ' και Α ' , 5 μαθητές περισσότερους από τη Β ' . Η Α' τάξη έχει 2 κορίτσια περισσότερα από τα αγόρια και κάθε μια από τις τάξεις Β ' και Γ ' ένα κορίτσι περισσότερο από τα αγόρια. Να βρεθεί η πιθανότητα:

α. Να πάρουμε τυχαία μια μαθήτρια και να είναι μαθήτρια της Α'Τάξης. β. Να πάρουμε τυχαία έναν μαθητή και να είναι μαθητής της Β ' Τάξης γ. Να πάρουμε τυχαία ένα μαθητή και μια μαθήτρια και να είναι μαθητές της Γ ' Τάξης. δ. Να διαλέξουμε τυχαία από τον αλφαβητικό κατάλογο του σχολείου ένα όνομα και να αντιστοιχεί σε μαθήτρια της Α' Τάξης .

1 6. Σε ένα παραλληλόγραμμο ΑΒΓ Δ οι ΔΕ και ΒΖ είναι κάθετες στη διαγώνιο ΑΓ και τέμνουν τις ΑΒ και ΓΔ στα σημεία Κ,Λ αντίστοιχα. Αν ΑΒ = I Ocm, ΑΓ= 9cm και EZ=3cm. Να υπολογίσετε το εμβαδό του παραλληλογράμμου. 1 7. Σε ένα τραπέζιο ΑΒΓΔ(ΑΒ//ΓΔ), αν ΕΖ//ΑΒ//ΓΔ, (Ε ανήκει στην ΑΔ και Ζ ανήκει στην ΒΓ). Να υπολογιστεί το ευθ. τμήμα ΒΖ , αν ΑΕ =9cm , ΕΔ=3 cm και BΓ=1 6cm.

1 8. Σε ένα ισόπλευρο τρίγωνο ΑΒΓ ενώνουμε τα μέσα των πλευρών του και σχηματίζουμε ένα νέο τρίγωνο ΚΛΜ. Να αποδείξετε ότι το τρίγωνο αυτό είναι ισόπλευρο και το εμβαδόν του είναι 4 φορές μικρότερο από το εμβαδό του αρχικού τριγώνου ΑΒΓ.

1 9. Να αποδείξετε ότι: α. συν ( 1 50°-χ)=-συν(30°+χ) β. ημ( 1 50°-χ)= ημ(30°+χ) γ. ημ(Α+Β)=ημΓ, ημ(Β+Γ)=ημΑ και ημ(Γ+Α)=ημΒ, όπου Α,Β,Γ γωνίες τυχαίου τριγώνου ΑΒΓ.

20. Αν οι κορυφές τετραπλεύρου, ΑΒΓΔ είναι σημεία ενός κύκλου, τότε : συν Α +συνΒ+συνΓ +συν Δ=Ο

2 1 . Να αποδείξετε ότι : α. (ημω+συνω)2= 1+2ημω.συνω β. ημ2χ.συν2ψ+συν2χ.συν2ψ+ημ2ψ= 1 δ. συνχ

+ η μχ

= -1-

η μχ 1 + συνχ η μχ

ΕΥΚΛΕΙΔΗΣ Α ' 88 τ.4/35

Page 38: Ευκλειδης Α 88

----------- Ασκήσεις Επαναληπτικές Γ Γυμνασίου ---------­

Από τον Μαθη ματικό ό μιλο Εκπαιδευτη ρίων " Εκπαιδευτική Αναγέννηση " 1 . Να παραγοντοποιήσετε τις παρακάτω παραστάσεις:

Α=χ2-5f+4 Β=χ2+6χψ+5ψ2 Γ=α2-3α+2 Δ=α+2β+2αβ+4β2 Ε=α2-γ +2αβ+β2 Ζ=α3+2αβ+8β+4α2 Η=χ2+ψ2+2χ+ψ(2ψ+ 1 )+3ψ-2 Θ=χ2-3χψ-4ψ2

2 . Να υπολογίσετε τις παρακάτω παραστάσεις χωρίς χρήση υπολογιστή τσέπης:

Α= 200 12 - 4002 + 1 Β=(992+ 1 99)2 1 0002

3 . Πέντε διαφορετικές κάλπες περιέχουν βόλους. Η δεύτερη κάλπη έχει διπλάσιο αριθμό βόλων από την πρώτη . Η τρίτη έχει τριπλάσιο αριθμό από την τέταρτη και η πέμπτη έχει 2 βόλους. Αν όλες οι κάλπες περιέχουν 36 βόλους παραπάνω από το άθροισμα των βόλων της τέταρτης και της πέμπτης κάλπης μαζί, να βρείτε το άθροισμα των βόλων της πρώτης και της τέταρτης κάλπης

4. Να αποδείξετε την παρακάτω ισότητα: (α2+2αβ+β2)2-2(α2-β2)+(α2-2αβ+β2)2=1 6α2β2 5 . Να βρείτε τα αναπτύγματα στις παρακάτω ισότητες:

Α� ( s,Ji + 7.J3)' Β� ( 3α -;β)' Γ�(2κ2+3β)3 Δ� ( .fiX -iβψ)' Ε�8 Ι κ4-1 6λ4

6. Να προσδιορίσετε τα χ,ψ,z αν χ+ψ+z=3 και χ2+ψ2+z2=χψ+ψz+zχ. 3 1 7. Να λύσετε την παρακάτω εξίσωση : χ2+χ+ 1 = χ -

χ2 - 6χ + 5 2 2 α2 25 8 . Να βρείτε το πρόσημο της παρακάτω παράστασης: Α=2χ -1 Οαψ-7χ+ 1 +4ψ + 25 - + -

9 . Να λύσετε τις παρακάτω εξισώσεις: I. (χ2+3χ+2)2+2(χ2+3χ+4)=7 π. (χ2+4χ+6)2+(χ2-1 2χ+36)2=0

1 Ο. Α.Να βρεθούν δυο πολυώνυμα Ρ( χ) και Q(x) πρώτου βαθμού αν I. Οι συντελεστές του χ είναι θετικοί με Ε.Κ.Π=2

Π. Οι σταθεροί όροι είναι θετικοί πρώτοι με Ε.Κ.Π=6 ΠΙ. P(-1 )>Q(-1 )

4 4

' ' 2χ - 3 χ - 6 Β. Να λυσετε την εξισωση : Ρ (χ)

+ Q (χ)

= 1 όπου P(x),Q(x) τα πολυώνυμα του Α

ΕρωηΊματος. 1 1 . Να λύσετε τις παρακάτω εξισώσεις:

I. 2(χ2+χ+ 1 )-3χ(χ-1 )=(χ+2)2 Π. Χ2+3αχ-4α2 =Ο ΠΙ. 2χ2+ 1 3χ=6-2(χ-2)(3χ-4)-9 IV. χ2 - (2J3 + J5)x + 2Ji5

χ2 +2-(2χ-3) -4χ Χ2 + 2χ + 3 (2χ - χ2 ) 4χ3 + 2χ2 - 2χ 1 2 . Δίνονται οι παραστάσεις: Α= , Β= , Γ= . χ-1 2χ χ

I. Να βρείτε τις τιμές του χ για τις οποίες ορίζονται οι παραστάσεις Α,Β και Γ Π. Να aπλοποιήσετε τις παραπάνω παραστάσεις

ΠΙ. Να βρείτε τις τιμές του χ αν ισχύει Α+Β= Γ 2

1 3 . Δίνεται η παράσταση Α=χψ-2χ-3ψ+6 όπου χ,ψ θετικοί ακέραιοι. Να λύσετε την εξίσωση Α=6. 1 4 . Να λύσετε τις παρακάτω κλασματικές εξισώσεις:

I. 3 + 2χ + � - 1 = 3 - 2χ π. χ - 5 + 2χ + 6 = ;χ χ - 2 χ - 4 χ + 2 χ + 1 χ - 1 χ - 1

IV. Χ - 2 - 5 - Χ = -iL V 3 + Χ - Χ + 1 = 2χ 2

χ + 3 χ - 3 χ - 9 .

χ - _!_ χ - 1 2χ2 - 3χ + 1 2

ΕΥΚΛΕΙΔΗΣ Α ' 88 τ.4/36

ΠΙ. 5 + χ _ χ - 3 = -5-χ + 1 χ - 2 χ - 1

Page 39: Ευκλειδης Α 88

----------- Ασκήσεις Επαναληπτικές Γ' Γυμνασίου -----------

Ι 5. α) Να βρεθεί η εξίσωση της ευθείας (ε) με την γραφική παράσταση του σχήματος. β) Να βρεθεί το εμβαδόν που περικλείεται μεταξύ του άξονα χ ' χ, της (ε) και των ευθειών χ= - 1 και χ = 1 . γ) Να βρεθεί το σημείο τομής Β(χο,Υο) της (ε) με τη γραφική παράσταση της y = - 2χ και το εμβαδόν του τριγώνου ΑΒΟ, όπου Α το σημείο τομής της (ε) με τον χ 'χ.

1 6 . Έστω οι ευθείες: (ε ι ) : y = 2αχ+β, (ε2) : y = (α - 1 )χ+5β. Να βρεθούν τα α και β, ώστε οι ευθείες (ε ι ) και (ε2) : α) να είναι παράλληλες, β) να ταυτίζονται, γ) να τέμνονται στο Α( 1 , 1 ) .

1 7 . i . Να υπολογίσετε την τιμή του χ στο παρακάτω σύστημα : { χ6 - 4χ4 - 1 6χ3+ 32χ2= ο

3 - 2χ2 + χ > ο

·3

ii . Για την τιμή το{u �::��::::ρωτήματος να λύσετε το σύσtημα

α - ψ = (ψ - 1 )2 1 8 . Να υπολογίσετε την τιμή του χ στα παρακάτω συστήματα :

i . { χ4 - 1 0χ3 + 2χ2 + 5χ -8 = ο χ2 + 4χ + 3 = ο

ii . χ2+ χ = ο { χ4 + χ3 - 4χ3 + 4χ = ο χ3 + 8χ +4 + 5χ2 + 7χ + 7 = ο

1 9. Να υπολογίσετε την τιμή του χ . α2 χ 2 - .J[3 αχ + 1 = Ο 20. Να υπολογίσετε την τιμή του χ στα παρακάτω συστήματα .

ί . { χψ = 3 "' 4χ + 2ψ = 1 1 2 2 2 1 ί ί . { χ - χ - ψ = -

χ + ψ = 3 ί ί ί . { χ - ψ = 6

-2χ + 4ψ = -8 iv. { χ - ψ = 6

-2χ + 4ψ = 8 ν. { χ2 + ψ2 = 8

(χψ)2 = 42

ΕΥΚΛΕΙΔΗΣ Α' 88 τ.4/37

· 1

Page 40: Ευκλειδης Α 88

Επιμέλεια: Επιτροπή Διαγωνισμών

30η Ελληνική Μαθηματική Ολυμπιάδα

"Ο Αρχιμήδης" 23 Φεβρουαρίου 2013

Λύσεις προβλημάτων μικρών τάξεων ΠΡΟΒΛΗΜΑ 1 (α) Να γράψετε την παράσταση A = k4 + 4 , όπου k θετικός ακέραιος, ως γινόμενο δύο παραγόντων που ο καθένας τους να είναι άθροισμα δύο τετραγώνων ακεραίων αριθμών. (β) Να aπλοποιήσετε την παράσταση

Κ = ( 2' +υ( 4' +υ( 6' +Η · · ·{(2n)' +υ (ι4 + l)(34 + l)(s4 + l} . . . {(2n - I)4 + l)

και να τη γράψετε ως άθροισμα τετραγώνων δύο διαδοχικών θετικών ακέραιων. Λύση (α) Έχουμε

k4 +4 = ( k2 )2 +4k2 + 22 -4k2 = ( k2 +2)2 -(2k)2 = ( k2 +2-2k) ( k2 +2+2k) = [( k -1)2 + 12 ][( k + 1)2 + 12 ] . (β) Πολλαπλασιάζουμε και τους δύο όρους του κλάσματος επί ( 24 )n , οπότε έχουμε:

κ = ( 2' +Η 4' +Η 6' +n · - [(2n)' +�] ( 4' + 4) (s' + 4) ( ι2' + 4) . . . [(4n)' + 4] (ι4 +� )(34 +�)(s4 +�) . . . [(2n - 1)4 +�] ( 24 + 4 ) ( 64 + 4) ( 1 04 +4) · · · [( 4n -2)4 + 4] (32 + ι) ( 52 + 1 ) ( 72 + ι) (92 + ι) ( l l2 + ι) . . . [( 4n -3)2 + ι][( 4n - ι)2 + ι][( 4n + ι)2 + ι]

- ( 12 + ι) ( 32 + ι) ( 52 + 1 ) ( 72 + 1 ) (92 + 1 ) ( 1 ι2 + 1 ) ( 1 32 + 1) · · · [ ( 4n -3 )2 + 1 ] [( 4n - 1)2 + ι J ( 4n + ι )2 + ι 2 2

= 2 = 8n2 + 4n + 1 = 4n2 + 4n2 + 4n + ι = ( 2n) + ( 2n + ι) . ι + 1

Παρατήρηση. Για το ερώτημα (β) μπορούμε να χρησιμοποιήσουμε και την παραγοντοποίηση

k' +�+' +H -k' = (k' +Ξ-k)(k' +Ξ++[(k -H +H(k +H +Η Για την απλοποίηση του κλάσματος εργαζόμαστε όπως προηγουμένως.

ΠΡΟΒΛΗΜΑ 2 Δίνεται οξυγώνιο τρίγωνο ΑΒΓ , με ΑΒ < ΑΓ . Έστω Μ το

μέσο της πλευράς ΒΓ. Στην πλευρά ΑΒ θεωρούμε σημείο Δ τέτοιο ώστε, αν το ευθύγραμμο τμήμα Γ Δ τέμνει τη διάμεσο ΑΜ στο σημείο Ε, τότε ισχύει ότι ΑΔ = ΔΕ . Ν α αποδείξετε ότι ΑΒ = ΓΕ .

ΕΥΚΛΕΙΔΗΣ Α ' 88 τ.4/38

Α

Page 41: Ευκλειδης Α 88

------------- Μαθηματικοί Διαγωνισμοί ------------­

Σχήμα 1 2°ς τρόπος

Λύση (1 ος τρόπος) Προεκτείνουμε τη διάμεσο ΑΜ κατά τμήμα ΜΘ = ΑΜ . Επειδή οι διαγώνιες του τετραπλεύρου ΑΒΘΓ διχοτομούνται το τετράπλευρο αυτό είναι παραλληλόγραμμο. Άρα είναι ΑΒ 1 1 ΓΘ και Αι = Θι , (εντός εναλλάξ γωνίες) . Όμως από την ισότητα ΑΔ = ΔΕ της υπόθεσης έπεται ότι Αι = Ει και επιπλέον Ει = Ε2 , ως

κατά κορυφή . Άρα είναι και Θι = Ε 2 , οπότε το τρίγωνο ΕΓΘ είναι ισοσκελές με ΓΕ = ΓΘ . Όμως από το παραλληλόγραμμο ΑΒΘΓ έχουμε ότι ΑΒ = ΓΘ , οπότε από τις δύο τελευταίες ισότητες προκύπτει το ζητούμενο ΑΒ = ΓΕ .

Από το μέσο Μ της πλευράς ΒΓ φέρουμε ευθεία δ παράλληλη προς την πλευρά ΑΒ, άρα και προς την πλευρά ΒΔ του τριγώνου ΒΓΔ, η οποία τέμνει το ευθύγραμμο τμήμα ΓΔ, έστω στο σημείο Ζ. Τότε το Ζ θα είναι το μέσο της πλευράς ΓΔ, δηλαδή ΓΖ = ΖΔ ( 1 ) και επιπλέον ισχύει ότι ΒΔ = 2 · ΜΖ . (2)

Σχήμα 2 Επίσης έχουμε Αι = Μι , (εντός εναλλάξ γωνίες) . Όμως από την ισότητα ΑΔ = ΔΕ της

υπόθεσης έπεται ότι Αι = Ει και επιπλέον Ει = Ε2 , ως κατά κορυφή . Άρα είναι και Μι = Ε2 , οπότε το τρίγωνο ΕΜΖ είναι ισοσκελές με ΖΜ = ΕΖ . (3) Από τις παραπάνω ισότητες έχουμε:

ΓΕ = ΓΖ + ΖΕ = ΔΖ + ΖΕ + ΖΕ (λόγω της (1)) = ΔΕ + 2 · ΖΜ (λόγω της (3)) = ΑΔ + ΔΒ = ΑΒ. (λόγω της υπόθεσης και της (2))

ΠΡΟΒΛΗΜΑ 3 Έστω Α = abcd = a · 1 03 + b · 1 02 + c · 1 Ο + d τετραψήφιος θετικός ακέραιος με ψηφία

τέτοια ώστε να ισχύουν: a � 7 και a > b > c > d > Ο' . Θεωρούμε και τον θετικό ακέραιο Β = dcba = d · 1 03 + c · 1 02 + b · 1 Ο + a , που προκύπτει από τον Α με αντίστροφη γραφή των ψηφίων του. Αν δίνεται ότι ο αριθμός Α+Β έχει όλα τα ψηφία του περιττούς ακέραιους, να προσδιορίσετε όλες τις δυνατές τιμές του αριθμού Α.

Λύση Έχουμε ότι: Α + Β = ( α + d) · 1 03 + (b + c ) · 1 02 + (b + c) · 1 0 + ( α + d) . Από την υπόθεση, όλα τα ψηφία του ακέραιου Α + Β είναι περιττοί ακέραιοι. Όμως για την

εύρεση των ψηφίων του ακεραίου Α + Β πρέπει να ξέρουμε αν οι ακέραιοι α + d και b + c είναι μικρότεροι του 1 Ο. Έτσι διακρίνουμε τις περιπτώσεις: (α) Έστω α + d � 1 0 και b + c � 1 0 . Τότε, επειδή α > b > c > d > Ο , θα έχουμε:

α + d = 1 0 + k, k = 0, 1, 2, . . . , 5 , b + c = 1 0 + R, f = 0, 1, 2, . . . , 5 .

ΕΥΚΛΕΙΔΗΣ Α' 88 τ.4/39

Page 42: Ευκλειδης Α 88

------------ Μαθηματικοί Διαγωνισμοί -----------­Έτσι ο αριθμός Α + Β γράφεται στη μορφή

A+B=(10+k) · 1<f +(10+f} · 1cY +(10+f} · 10+(10+k)= 104 +( k+ 1) · 1<f +( f+ 1) · 1cY +( f+ 1) · 10+k, δηλαδή έχει ψηφία 1, k + 1, f + 1, f + 1, k , τα οποία πρέπει να είναι περιττοί ακέραιοι, που είναι

άτοπο, λόγω της ύπαρξης των διαδοχικών ακέραιων k και k + 1 . (β) Έστω α + d ;;::: 1 0 και b + c < 1 0 . Τότε, επειδή α > b > c > d > Ο , θα έχουμε: α + d = 1 0 + k, k = 0, 1, 2, . . . , 5 και ο αριθμός Α + Β γράφεται

Α+ B = (1 0+k) · 103 + (b + c) · 102 + (b + c) · 1 0+ ( 10 + k)= 104 + k · 1 03 + (b + c) · 102 + (b + c+ 1) · 1 0 +k, οπότε έχουμε τις περιπτώσεις:

• Αν b + c = 9 , τότε ο Α + Β έχει ψηφίο δεκάδων το Ο, που είναι άρτιος, άτοπο. • Α ν b + c < 9 , τότε ο Α + Β έχει ψηφία τους ακέραιους b + c και b + c + 1 που δεν είναι

δυνατόν να είναι και οι δύο περιττοί. (γ) Έστω α + d < 1 0 και b + c ;;::: 1 0 . Τότε, επειδή α > b > c > d > Ο , θα έχουμε: b + c = 1 0 + f, f = 0, 1 , 2, . . . , 5 και ο αριθμός Α + Β γράφεται

Α+ Β = ( α+d) · 103 + (1 0+ f} · 102 + ( 10 + f} · 1 0 + ( α+d) = ( α+d + 1 ) · 1 03 + ( f + 1) · 1 02 + f · 1 0 + ( α+d) , οπότε οι ακέραιοι f και f + 1 είναι ψηφία του Α + Β , άτοπο. (δ) Έστω α + d < 1 Ο και b + c < 1 Ο . Τότε τα ψηφία του αριθμού Α + Β είναι οι ακέραιοι α + d και b + c , οι οποίοι πρέπει να είναι περιττοί. Λόγω των περιορισμών α > b > c > d > Ο και α ;;::: 7 , έπεται ότι α + d = 9 και επίσης 5 ;;::: b ;;::: 2, 6 ;;::: c ;;::: 3 , οπότε 1 0 > b + c ;;::: 5 , δηλαδή

b + c ε { 5, 7, 9} Επομένως, έχουμε τις παρακάτω περιπτώσεις:

• α+ d = 9 με α = 8, d = 1 και b + c = 9 με b = 7, c = 2 ή b = 6, c = 3 ή b = 5, c = 4 Επομένως, προκύπτουν οι αριθμοί: Α = 872ι , Α = 863ι , Α = 854ι .

• α + d = 9 με α = 7, d = 2 και b + c = 9 με b = 6, c = 3 ή b = 5, c = 4. Στη περίπτωση αυτή προκύπτουν οι αριθμοί: Α = 7632 , Α = 7542

• α + d = 9 με α = 8, d = 1 και b + c = 7 με b = 5, c = 2 ή b = 4, c = 3 .

Στη περίπτωση αυτή προκύπτουν οι αριθμοί: Α = 852ι , Α = 843ι . • α + d = 9 με α = 7, d = 2 και b + c = 7 με b = 4, c = 3 .

Στη περίπτωση αυτή προκύπτει ο αριθμός: Α = 7432 . • α + d = 9 με α = 8, d = 1 και b + c = 5 με b = 3, c = 2 .

Στη περίπτωση αυτή προκύπτει ο αριθμός: Α = 832ι . ΠΡΟ ΒΛ Η Μ Α 4 Να βρείτε όλες τις τριάδες (x, y, z) θετικών ακέραιων αριθμών που είναι λύσεις της

ξ, ι 2 4 ι ε ισωσης: - + - - - = χ Υ z

Λύση

Α ' 3 3 ' θ ' 1 2 4 1 2 4 1 4

1 ' ξ' ν ειναι χ ;;::: και y ;;::: , τοτε α εχουμε: - + - - - � - + - -- = - - < , οποτε η ε ισωση

x y z 3 3 z z δεν επαληθεύεται. Επομένως θα είναι: χ � 2 ή y � 2 , οπότε πρέπει να ισχύει ένα από τα

επόμενα: χ = 1 ή χ = 2 ή y = 1 ή y = 2 . Στη συνέχεια διακρίνουμε τις περιπτώσεις:

2 4 • Για χ = 1 η εξίσωση γίνεται: - - - = Ο <:::::> z = 2y <:::::> y = k, z = 2k, όπου k θετικός Υ z

ακέραιος, οπότε έχουμε τις λύσεις (x, y, z ) = ( 1, k, 2k) , k ε Ζ θετικός.

2 4 1 2 8+z 4z 4(z+8)-32 32 • Για χ = 2 η εξίσωση γίνεται: - -- =- <=>- =- <=>y=- <=>y <:::::>y=4--.

y z 2 y 2z z+8 z+8 z+8

ΕΥΚΛΕΙΔΗΣ Α' 88 τ.4/40

Page 43: Ευκλειδης Α 88

------------- Μαθηματικοί Διαγωνισμοί ------------­Επειδή ο y πρέπει να είναι ακέραιος, έπεται ότι ο z + 8 πρέπει να είναι θετικός διαιρέτης του 32 και μεγαλύτερος του 8 . Άρα οι δυνατές τιμές του z + 8 είναι 1 6 ή 32, οπότε z = 8 ή z = 24. Για z = 8 λαμβάνουμε y = 2 , ενώ για z = 24 λαμβάνουμε y = 3 . Άρα στην περίπτωση αυτή έχουμε τις λύσεις (x, y, z ) = (2, 2, 8) ή (x, y, z ) = (2, 3, 24) .

Γ 1 ξ, , 1 4

1 4 1 + χ 4χ

4 4 Ε δ , . , ια y = η ε ισωση γινεται: - - - = - <=> - = -- <=> z = -- = ---. πει η πρεπει

χ z z χ l + x l + x ο χ να είναι ακέραιος, πρέπει ο 1 + χ να είναι θετικός διαιρέτης του 4 και μεγαλύτερος του 1 , δηλαδή πρέπει l + x = 2 ή l + x = 4 <:::> x = l ή χ = 3 Για χ = 1 λαμβάνουμε z = 2, ενώ

για χ = 3 λαμβάνουμε z = 3 . Άρα στην περίπτωση αυτή έχουμε τις λύσεις (x, y, z ) = (l , l , 2) ή (x, y, z) = (3, 1, 3 ) .

• Για y = 2 η εξίσωση γίνεται: _!_ _ _± = Ο <=> z = 4χ <=> χ = .e, z = 4.f, όπου .e θετικός χ z

ακέραιος. Άρα, στην περίπτωση αυτή έχουμε τις λύσεις (χ, y, z) = ( .e, 2, 4.f) , όπου .e θετικός ακέραιος. Συνολικά, λαμβάνοντας υπόψη και τις επικαλύψεις των λύσεων που βρήκαμε, έχουμε τις

λύσεις: (χ, y, z ) = ( 1, k, 2k ) , όπου k θετικός ακέραιος, (χ, y, z) = (.e, 2, 4.e) , όπου .e θετικός

ακέραιος, (x, y, z ) = (3, 1, 3 ) και (x, y, z ) = (2, 3, 24) .

Λύση ασκήσεων τεύχους 87 Al S. Δίνεται ότι η εξίσωση mx2 - 1 0χ + 3 = Ο έχει δύο ρίζες χ1 , χ2 τέτοιες ώστε χ2 = � χ1 • Να

3 βρείτε την τιμή της παραμέτρου m και τις ρίζες της εξίσωσης. Λύση

Αν m = Ο , η εξίσωση έχει μόνο μία ρίζα χ = �. Άρα πρέπει m * Ο . Από τους τύπους του Vieta 1 0

έχουμε:

Άρα έχουμε

2 1 0 2 2 3 5 1 0 2 2 3 6 2 9 χ + -χ = - -χ = - <=> -χ = - -χ = - <=> χ = - χ = -1 3 1 m ' 3 1 m 3 1 m ' 3 1 m 1 m ' 1 2m ( 6 )2 9 36 9 2 - = - <=> -2 = - <=> 9m = 72m <:::> 9m (m - 8) = 0 <=> m = 8 ,

m 2m m 2m

' ο ' λ β ' 3 1 αφου m * , οποτε αμ ανουμε χ1 = - και χ2 = - . 4 2

Α16. Βρείτε όλα τα ζεύγη (χ, y) μη αρνητικών ακέραιων που είναι λύσεις της εξίσωσης : χ - Υ = χ2 + xy + / . Λύση Για (χ, y) = (Ο, Ο) η εξίσωση αληθεύει. Έστω ότι (χ, y) * (Ο, Ο) . Τότε, θα είναι

χ' + xy + y' = (χ + Ω' +

3 �2

> Ο , οπότε από την ισότητα χ - y = χ' + η> + y' προκύπτει ότι:

χ - y > Ο και χ > y � Ο . Τότε θα είναι χ � 1 και χ2 + .xy + / � χ2 � χ � χ - y , οι οποίες πρέπει να ισχύουν παντού ως ισότητες, δηλαδή μόνον όταν χ = 1 και y = Ο. Επομένως μία ακόμη λύση είναι το ζευγάρι (χ, y) = ( 1, Ο) . ΓΙΟ. Σε οξυγώνιο τρίγωνο ABC, θεωρούμε το ύψος CH, Η ε ΑΒ και τη διάμεσο ΒΚ, Κ

ΕΥΚΛΕΙΔΗΣ Α' 88 τ.4/41

Page 44: Ευκλειδης Α 88

------------- Μαθηματικοί Διαγωνισμοί ------------­

μέσον της AC. Δίνεται ότιΒΚ=CΗ και KBC = HCB . Να αποδείξετε ότι το τρίγωνο ABC είναι ισόπλευρο.

Λύση Επειδή KBC = HCB , το τρίγωνο EBC είναι ισοσκελές με ΕΒ = EC . Από την υπόθεση έχουμε ΒΚ = CH , οπότε με αφαίρεση κατά μέλη των δύο ισοτήτων λαμβάνουμε ΒΚ -ΕΒ = CH -EC => ΕΚ =ΕΗ. Επομένως τα τρίγωνα ΒΕΗ και CEK έχουν δύο πλευρές ίσες και τις περιεχόμενες των πλευρών αυτών γωνίες ίσες (ως κατά κορυφή). Άρα τα τρίγωνα είναι ίσα, οπότε θα έχουν και

EKC = ΕΗΒ => BKC = CHB = 90° . Β �---------� 0 Επομένως, η διάμεσος ΒΚ είναι και ύψος του

τριγώνου ABC , οπότε ΑΒ = BC . Σχήμα 1 Επιπλέον τα ύψη ΒΚ και CH είναι από υπόθεση ίσα,

οπότε και τα ορθογώνια τρίγωνα ΑΒΚ και Α CH είναι ίσα με συνέπεια την ισότητα ΑΒ = Α C . Γ Ι I . Τα σημεία A, B, C και D βρίσκονται πάνω σε ευθεία ε με τη σειρά που δίνονται. Οι κύκλοι με διάμετρο τις A C και BD τέμνονται στα σημεία S και Τ . Αν είναι DAS = 20° και ADS = 25° , να αποδείξετε ότι οι ευθείες SB και SC χωρίζουν τη γωνία

Λ ASD σε τρεις ίσες γωνίες. Λύση

Α Β c D Σχήμα 2

Από το τρίγωνο ADS έχουμε ASD = 1 80° - { ADS + DAS) = 1 35° . Επειδή το σημείο S βρίσκεται στον κύκλο διαμέτρου BD θα είναι BSD = 90° , οπότε ASB = 1 35° - 90° = 45° .

Λ Λ Λ Ομοίως είναι ASC = 90° , οπότε CSD = 1 35° - 90° = 45° και BSC = 45°.

Ν 1 1 . Στο σύνολο των θετικών ακέραιων, να λύσετε την εξίσωση χ2 + y2

+ z2 = 6383

Λύση Παρατηρούμε ότι το πρώτο μέλος της εξίσωσης είναι άθροισμα τριών τετραγώνων θετικών

ακέραιων. Γνωρίζουμε ότι για α = 2k Ε Ζ ισχύει ότι α2 = 4k2 = Ο ( mod 4) , ενώ για

α = 2k + 1 Ε Ζ , ισχύει ότι α2 = 4k 2 + 4k + 1 = 1 ( mod 4) . Επιπλέον, παρατηρούμε ότι

6383 = 4 · 1 595 + 3 = 3 ( mod4) .

Επειδή το άθροισμα των τετραγώνων χ2 + y2

+ z2 γίνεται ισοϋπόλοιπο tου 3 mod 4 , μόνον όταν

και οι τρεις θετικοί ακέραιοι x, y, z είναι περιττοί, έπεται ότι πρέπει να είναι

χ = 2k + 1, y = 2m + 1, z = 2n + 1, k, m, n Ε Ζ θετικοί οπότε η εξίσωση χ2 + / + z2 = 6383 γίνεται

ΕΥΚΛΕΙΔΗΣ Α. 88 τ.4/42

Page 45: Ευκλειδης Α 88

------------- Μαθηματικοί Διαγωνισμοί -------------

4 ( k2 + m2 + n2 ) + 4 (k + m + n ) + 3 = 6383

<::::> k2 + m2 + n2 + k + m + n = 1 595 <=> k ( k + 1) + m ( m + 1 ) + n( n + Ι ) = 1 595 .

Το πρώτο μέλος της παραπάνω εξίσωσης είναι άρτιος ακέραιος, ως άθροισμα τριών άρτιων ακέραιων, αφού είναι γινόμενα διαδοχικών ακέραιων, ενώ το δεύτερο μέλος είναι περιττός ακέραιος, με συνέπεια η εξίσωση να είναι αδύνατη . Άρα η δεδομένη εξίσωση είναι αδύνατη στους θετικούς ακέραιους.

Δ 7" Δίνεται ορθογώνιο πλέγμα αποτελούμενο από 50 χ 2 σημεία (τελείες). Ν α βρείτε τον αριθμό των ισοσκελών τριγώνων που έχουν τις κορυφές τους σε σημεία του πλέγματος. Λύση Υποθέτουμε ότι τα διαδοχικά σημεία του πλέγματος απέχουν απόσταση 1 οριζόντια και κατακόρυφα. Υπάρχουν δύο κατηγορίες ισοσκελών τριγώνων που μπορούν να έχουν τις τρεις κορυφές τους σημεία του πλέγματος. Η πρώτη κατηγορία αποτελείται από ορθογώνια ισοσκελή τρίγωνα με τις κάθετες πλευρές τους ίσες με 1 και την υποτείνουσα ίση με J2 . Για κάθε μία από τις 48 στήλες του πλέγματος με αριθμό 2,3 , . . . ,49 ορίζονται συνολικά 4 τέτοια τρίγωνα, ενώ για τις δύο ακραίες στήλες ορίζοντα από δύο τέτοια τρίγωνα. Έτσι συνολικά ορίζονται 48 · 4 + 2 · 2 = 1 96 ισοσκελή τρίγωνα σε αυτή την περίπτωση . Η δεύτερη κατηγορία αφορά τρίγωνα με δύο κορυφές στην ίδια ευθεία και την τρίτη κορυφή στην άλλη ευθεία, αλλά και στη μεσοκάθετη της αρχικής πλευράς. Εδώ πρέπει η απόσταση των δύο πρώτων σημείων να είναι άρτιος ακέραιος, έτσι ώστε .να υπάρχει η μεσοκάθετη πάνω στην οποία βρίσκεται η τρίτη κορυφή . Χωρίζουμε τις στήλες σε δύο υποσύνολα, περιττές με αριθμό 1 ,3 ,5 , . . . , 49 και άρτιες με αριθμό 2,4,6, . . . 50. Από καθένα από τα παραπάνω υποσύνολα των στηλών, θεωρώντας τη βάση κατ' αρχή στη μία πλευρά του πλέγματος, μπορούμε να έχουμε ( 2;) + ( 2;) = 2 { 2;) = 2 25/4

= 600 βάσεtς ισοσκελών τριγώνων, άρα και 600 ισοσιcελ�

τρίγωνα. Έτσι στην κατηγορία αυτή έχουμε συνολικά 1 200 τέτοια ισοσκελή τρίγωνα. Επομένως ο συνολικός αριθμός των ισοσκελών τριγώνων που ζητάμε είναι 1 396.

lXI I><I><Ld1Γ\bJ Ι Ι · Ι ΙΧΙ 1 2 3 4 5 6 7 8 9 10 1 1 48 4 9 50

Στο παραπάνω σχήμα φαίνονται τα δύο τρίγωνα που αντιστοιχούν στις στήλες με αριθμό 1 και 50, τα τέσσερα τρίγωνα που αντιστοιχούν στις στήλες με αριθμό 2,3, . . . 49 και δύο μόνο από τα τρίγωνα της δεύτερης περίπτωσης που αντιστοιχούν στη στήλη 7 .

ΑσκΊ1σεις για λύση Ν 1 2 . Σε δεδομένο θετικό ακέραιο Ν γράφουμε δεξιά του δύο διαφορετικά μη μηδενικά ψηφία . . Ο αριθμός που προκύπτει διαιρείται με τον Ν . Να προσδιορίσετε τη μέγιστη δυνατή τιμή του Ν . Ν 1 3 . Να προσδιορίσετε όλους του ακέραιους n που είναι λύσεις της εξίσωσης

( n - 1 ) ( n - 3) ( n - 5 )· ·· ( n - 20 1 1 ) = n ( n + 2 ) ( n + 4) · ·· ( n + 20 10 ) Α Ή 7 . Ν α αποδείξετε ότι υπάρχουν άπειρα τέλεια τετράγωνα θετικών ακέραιων τα οποία μπορούν να παρασταθούν ως αθροίσματα της μορφής 2n + γ , όπου n, m διαφορετικοί θετικοί ακέραιοι. Π Ζ . Έστω ΑΒCD εγγράψιμο τετράπλευρο και M, L, N και Κ τα μέσα των πλευρών του AB.BC, CD και DA , αντίστοιχα. Δίνεται ότι ΒΜΝ = AINC . Να αποδείξετε ότι: { u) ΠkL = ciκ . (β) το τετράπλευρο ABCD έχει ένα ζευγάρι απέναντι πλευρές παράλληλες.

ΕΥΚΛΕΙΔΗΣ Α' 88 τ.4/43

Page 46: Ευκλειδης Α 88

Επιμέλεια: Συντακτική Επιτροπή

Αwό του� οuναδέλφοu� Γαρεφαλάκη Γιώργο και Σ ιούλα Γιάννη λά�αμε την εξαιρετική

εργασiα «ΜΑΘΗΜΑΠΚΑ ΚΑΙ . . . eτΥΜΟλΟΓΙΑ>>

Ένα μικρό δείγμα δημοσιεύουμε σε αυτό το τεύχος και η εργασία θα δημοσιευθεί σε επόμενο τεύχος

Αριθμός-Αριθμητική : Προέρχεται από το αρχαίο ρήμα αραρίσκω =συγκεντρώνω, συνάπτω, λογαριάζω.

Η Αριθμητική πρωτοαπαντάται στην Πολιτεία του Πλάτωνα, ενώ ο όρος Μαθηματικά

πρωτοχρησιμοποιείται στα Ηθικά Νικομάχεια του Αριστοτέλη . Η Αριθμητική (εκ του αριθμός) είχε και έχει πάντα περισσότερο πρακτικό-εφαρμοσμένο χαρακτήρα σε αντίθεση προς τα Μαθηματικά ,που καλύπτουν τον ευρύτερο χώρο της μαθηματικής επιστήμης και σκέψης.

Συνεργασία με το Μαθηματι κό Όμιλο των εκπαιδευτηρίων

" Εκπαιδευτική Αναγέννηση"

Οι μαθητές του Μαθηματικού Ομίλου της Εκπαιδευτικής Αναγέννησης, στο πλαίσιο των δραστηριοτήτων των Ομίλων, που λειτουργούν σε όλες τις βαθμίδες από το 2003 που ιδρύθηκε το Σχολείο, επισκέφθηκαν τα γραφεία της Μαθηματικής Εισιαtδευπιcή Ανιιyέιινηοq Εταιρείας. Κατά την επίσκεψη αυτή, η οποία έγινε

ΕΚΠΑΙΔΕΠΗΡΙΑ A�TQNOfiOYΛOY μετά από παράκληση των ίδιων των παιδιών, οι Ν-�;. - ιι,_,.σ - ιc,nσ-ι - ΛΗαο μαθητές πρότειναν στον κύριο Βαρόπουλο να

αναλάβουν την παρουσίαση των επαναληπτικών ασκήσεων για τις τάξεις β' και γ' Γυμνασίου, στο τελευταίο τεύχος του περιοδικού "Ευκλείδης", προκειμένου όχι μόνο να καλλιεργήσουν τα ίδια την έμφυτη κλίση τους και αγάπη για τα Μαθηματικά, αλλά και να δείξουν στους συμμαθητές τους την αξία και τη γοητεία της Μαθηματικής Λογικής. Στην συγγραφή των ασκήσεων συμμετείχαν οι μαθητές: Γ' Γυμνασίου : Αραπάκη Χριστίνα, Διαμαντή Κλεοπάτρα, Λαγωνίκα Ειρήνη - Άννα, Λαζαρίδης Ιάσων, Μακρής Αντώνης, Παναγιωτόπουλος Φοίβος, Παπαγεωργίου Νικόλ, Τσιριγώτη Μαρία Αγγελική, Φασίτσας Ιωάννης Β ' Λυκείου: Κάτρης Δημοσθένης, Κουτσούκου Μαρία, Οικονόμου Θανάσης. Γ' Λυκείου : Γουζούασης Βασίλης Με την καθοδήγηση των συναδέλφων μαθηματικών του σχολείου κ. Λάττα Κων/νου και κ. Σχίζα Κων/νου. Θερμά συγχαρητήρια στα παιδιά για την εξαιρετική εργασία τους. Επίσης ευχαριστούμε και συγχαίρουμε τους συναδέλφους κ. Λάττα Κων/νου , κ. Σχίζα Κων/νου και την κ. Διευθύντρια των Εκπαιδευτηρίων κ. Μαντά Έρι για τις πρωτοβουλίες τους και το εκπαιδευτικό τους έργο.

ΕΥΚΛΕΙΔΗΣ Α' 88 τ.4/44

Page 47: Ευκλειδης Α 88

Θεωρία Πα ιyνίων ========================= Γιώργος Τσομίδης

ι ανθρώπινες σχέσεις χαρακτηρίζονται από καθημερινές αλληλεπιδράσεις, συμβιβα­σμούς, υποχωρήσεις, συγκρούσεις, συμμαχίες, aνταγωνισμούς. Είναι εύλογο ότι κάθε άτομο δρα σύμφωνα με το προσωπικό του συμφέρον, ενώ θεωρείται θεμιτό και αποδε­

κτό να ενεργεί ως το σημείο εκείνο που δεν προσβάλλει, με οποιοδήποτε τρόπο, τα υπόλοιπα μέλη της κοινωνίας. Το συμφέρον του ατόμου εθεωρείτο ανέκαθεν κινητήριος μοχλός της συ­

μπεριφοράς του, καθώς στην ουσία αποτελεί φυσική ανάγκη τού κάθε ζωντανού οργανισμού να επιβιώσει στο περιβάλλον και να διαιωνίσει τα γονίδια του. Αυτός είναι και ο βασικός μηχανισμός της λειτουργίας της Φύσης, σύμφωνα με τους κανόνες της οποίας ο καλύτερα προσαρμοσμένος οργανισμός αναπτύσσεται και αναπαράγεται. Στη προσπάθεια του ο άνθρωπος να κατανοήσει αυτές τις αρχέγονες και αέναες συμπεριφορές χρησιμοποιεί Μαθηματικά εργαλεία, και επομένως το βασικό πλαίσιο (ή σημείο αναφοράς) μελέτης της συμπεριφοράς του ατόμου αναπτύσσεται και αναλύεται μέσα από τη Θεωρία Παιγνίων .

Η Θεωρία Παιγνίων άπτεται διαφόρων επιστημονικών κλάδων όπως είναι τα οικονομικά, η πολιτική, τα ηλεκτρονικά δίκτυα κ.ά. , διατηρώντας την

ουσία της αμετάβλητη, όπως θα επεσήμανε και ο Πλάτωνας. Ποια είναι όμως η ουσία της; Η στρατηγική αλληλεπίδραση των παικτών και τα αποτελέσματα που αυτή η «σύγκρουση» απο­φέρει. Οι αρχαίοι Έλληνες Φιλόσοφοι, είχαν επιτύχει μια πρώτη θεωρητική-περιγραφική προ­σέγγιση της Θεωρίας Παιγνίων μέσω της πολιτικής και κοινωνικής ανάλυσης της εποχής (Πολι­τεία Πλάτωνος, Ηθικά και Πολιτικά του Αριστοτέλη). Τι είναι όμως η Θεωρία Παιγνίων; Στο ακόλουθο άρθρο θα προσπαθήσουμε να προσεγγίσουμε και να κατανοήσουμε αυτόν τον συναρ­παστικό κόσμο ξεκινώντας με δύο πολύ κλασικά παραδείγματα, το πρώτο είναι γνωστό ως Δί­λημμα του Φυλακισμένου και το δεύτερο ως Η μάχη των δύο Φύλων.

Δίλημμα του Φυλακισμένου Ας φανταστούμε ότι υπάρχουν δύο σεσημασμένοι κακοποιοί, οι οποίοι είναι φίλοι, με πλού­

σιο ποινικό μητρώο ο καθένας στους. Έπειτα από αλλεπάλληλες σύλλήψεις και καταδίκες απο­φασίζουν να αφήσουν τη ζωή του κλέφτη και να ακολουθήσουν την οδό της νομιμοφροσύνης. Αφού λοιπόν βρίσκουν νόμιμα επαγγέλματα για να απασχολούνται και όλα βαίνουν ήρεμα στη ζωή τους, μαθαίνει ο ένας για ένα όχι και τόσο καλά φυλασσόμενο κοσμηματοπωλείο σε μία κοντινή γειτονιά. Αρχικά δεν αναφέρει στο φίλο του τίποτα και προσπαθεί να διώξει τις «κακές» σκέψεις από το μυαλό του. Περνώντας οι μέρες, η σκέψη του εύκολου κέρδους είχε για τα καλά ριζώσει μέσα στο μυαλό του και μην μπορώντας άλλο να κρατηθεί, τα λέει όλα στο φίλο του. Επειδή στη προκειμένη περίπτωση, είχε κυλήσει ο τέντζερης και είχε βρει το καπάκι του, απο­φασίζουν να ληστέψουν το κοσμηματοπωλείο.

Καταστρώνουν λοιπόν το σχέδιο δράσης και ένα ήσυχο βράδυ εξορμούν. Για κακή τους τύχη δύο αστυνομικοί με συνοδεία σκύλου τυχαίνει να περνούν έξω από το κοσμηματοπωλείο ενώ οι φίλοι μας βρίσκονται ήδη μέσα και γεμίζουν τις τσάντες τους με χρυσαφικά. Το σκυλί αρχίζει μανιασμένα να γαβγίζει, οι αστυνομικοί αντιλαμβάνονται ότι κάτι τρέχει, οι κλέφτες πανικοβάλ­λονται και πετιούνται απότομα έξω τρέχοντας και μετά από ένα σύντομο κυνηγητό οι αστυνομι­κοί τους συλλαμβάνουν.

ΕΥΚΛΕΙΔΗΣ Α' 88 τ.4/45

Page 48: Ευκλειδης Α 88

--------------- Θεωρία Παιγνίων ---------------

Στο αστυνομικό τμήμα τους βάζουν σε δύο ξεχωριστά δωμάτια ανάκρισης. Ο διοικητής, δια­βάζει τους φακέλους με το ιστορικό τους και αποφασίζει να τους «στριμώξει» προσφέροντάς την ίδια συμφωνία στον καθένα ξεχωριστά λέγοντας τα εξής:

Β/Α

Σιωπά

Ομολογεί

Αν δεν ομολογήσεις, θα μπεις φυλακή για ένα χρόνο. Αν προδώσεις το φίλο σου, και αυτός σιωπήσει εσύ θα τη γλιτώσεις και αυτός θα φάει τρία χρόνια. Αν όμως ο ένας προδώσει τον άλλο, τότε θα μπείτε και οι δύο μέσα για δύο χρόνια.

Όπως παρατηρείται ο διοικητής είναι έξυπνος και οι δύο φίλοι (Α και Β) ζητάνε από ένα κομμάτι χαρτί για να γράψουν σε ένα πινακάκι αυτά που τους προσφέρει ο Αστυνόμος. Έχουμε λοιπόν τον Πίνακα του Παιγνίου :

Σιωπά Ομολογεί Α: 1 χρόνο Α: ελεύθερος Β: 1 χρόνο Β : 3 χρόνια Α: 3 χρόνια Α: 2 χρόνια Β : ελεύθερος Β: 2χρόνια

Ας δούμε ποία θα είναι η έκβαση του παιγνίου ξεκινώντας από τον φυλακισμένο Α. τι θα επιλέξει1 ;

• Α ν ο Α δεν μιλήσει τότε γνωρίζει ότι υπάρχουν δύο περιπτώσεις: είτε ο Β να τον προδώ­σει είτε όχι. Αλλά από την άλλη μεριά αν ο Β σκεφτεί ότι Α δεν θα μιλήσει τότε συμφέ­ρει τον Β να προδώσει τον Α και να φύγει ελεύθερος.

• Α ν ο Β δεν μιλήσει τότε γνωρίζει ότι υπάρχουν δύο περιπτώσεις: είτε ο Α να τον προδώ­σει είτε όχι. Αλλά από την άλλη μεριά αν ο Α σκεφτεί ότι Β δεν θα μιλήσει τότε συμφέ­ρει τον Α να προδώσει τον Β και να φύγει ελεύθερος.

• Α ν ο Α ομολογήσει τότε γνωρίζει ότι υπάρχουν δύο περιπτώσεις: είτε ο Β να τον προδώ­σει είτε όχι. Αλλά από την άλλη μεριά αν ο Β σκεφτεί ότι Α θα ομολογήσει τότε συμφέ­ρει τον Β να προδώσει τον Α και να μπούνε και οι δύο μέσα για 2 χρόνια.

• Αν ο Β ομολογήσει τότε γνωρίζει ότι υπάρχουν δύο περιπτώσεις: είτε ο Α να τον προδώ­σει είτε όχι. Αλλά από την άλλη μεριά αν ο Α σκεφτεί ότι Β θα ομολογήσει τότε συμφέ­ρει τον Α να προδώσει τον Β και να μπούνε και οι δύο μέσα για 2 χρόνια.

Άρα η έκβαση του παιγνίου (δηλ. η ισορροπία του) είναι και οι δυο να ομολογήσουν [ομολογεί, ομολογεί] και να μπούνε για δύο χρόνια φυλακή . Παράλογο; Όπως μας διδάσκει η Θεωρία Παιγνίων και πιο συγκεκριμένα ο κύριος John Nash, με τη διάσημη ισορροπία Nash, μάλλον όχι. Ο John Nash ( 1 928) υπήρξε αναμφίβολα ένας από τους

σπουδαιότερους Μαθηματικούς-Οικονομολόγους του περασμένου αιώνα. Όχι μόνο για την οξυδέρκειά του αλλά και για τον τρόπο με τον οποίο προσέγγιζε τα διάφορα προβλήματα που απασχόλησαν τον επιστημονικό τον βίο. Ο Nash -ήρωας της χολιγουντιανής ταινίας «Ένα υπέροχος άνθρωπος»- ξεχώρισε όχι μόνο για την συμβολή του στην επιστήμη αλλά και λόγω της ιδιόρρυθμης

"'"'"''""'"'Ό,,.,, ,..,.,.,.....�""''" του, η οποία κλονιζόταν από τη βαριά σχιζοφρένεια και το σύνδρομο καταδίω­ξης; τα οποία τον ταλαιπωρούσαν ήδη από την ηλικία των 29 ετών. Χαρακτηριστικό της διανοί­ας του Nash είνqι η μοναδική φράση που είχε γράψει ο καθηγητής και μέντοράς του R.J. Duffin στη συνοδευτικη συστατική επιστολή για τις διδακτορικές του σπουδές (αρχικά στο Harvard και έπειτα στο Princeton) : «Α υτ6ς ο άν0 ι)(uπος είναι ιδιοφυία». Ο Nash υπήρξε <<αυθάδης» όσο αφορά τη Θεωρία Παιγνίων και ουσιαστικά τη πήρε από το χέρι

ΕΥΚΛΕΙΔΗΣ Α' 88 τ.4/46

Page 49: Ευκλειδης Α 88

--------------- Θεωρία Παιγνίων --------------­και την οδήγησε σε ένα ανώτερο επίπεδο με μία και μόνο απλή σκέψη . Αρχικά, ο Nash α­ναγνώρισε ότι σε ένα παίγνιο το αποτέλεσμα, εξ ορισμού, δεν εξαρτάται μόνο από την επιλογή ενός παίκτη, αλλά και από το σύνολο των επιλογών των υπολοίπων παικτών. Πιο συγκεκριμένα, δεν μπορείς να συμβουλέψεις κάποιον πώς πρέπει να συμπεριφερθεί σε ένα παίγνιο αγνοώντας τι θα κάνουν οι άλλοι. Άρα, δεν είναι δυνατόν να κάνεις εκείνη την επιλογή που εξυπηρετεί το συμφέρον σου καλύτερα, εφόσον δεν γνωρίζεις τις επιλογές των άλλων. Όμως, και οι υπόλοιποι παίκτες βρίσκονται στην ίδια θέση . Δηλαδή, είναι όλοι τους «παγιδευμένοι» στην ίδια κατάσταση ; Χαρακτηρίζεται, λοιπόν, η παραπάνω κατάσταση από aπροσδιοριστία; Αυτό το μπερδεμένο κουβάρι έλυσε ο Nash εισάγοντας τη περίφημη ισορροπία Nash.

Η ισορροπία Nash (ΝΕ) περιγράφει μια κατάσταση, η οποία έχει φτάσει σε εκείνο το σημείο, όπου κανένα παίχτη δεν συμφέρει να απομακρυνθεί, δεδομένων των επιλογών των αντιπάλων του, ακόμη και αν αυτή δεν συμφέ­ρει κανέναν. Με άλλα λόγια, όταν υπάρχει στρατηγική αλληλεπίδραση μεταξύ ατόμων, επιχει­ρήσεων κ.α. , μπορεί η ισορροπία του παιγνίου να καταλήγει σε μία κατάσταση στην οποία ενώ κανένας δεν είναι ευχαριστημένος (δεν έχει κέρδος-όφελος), είναι προτιμότερο για αυτόν/ους να μην απομακρυνθεί διότι τότε θα βγει εντελώς χαμένος. Όπως στο παραπάνω παράδειγμα με τους φίλους μας τους διαρρήκτες. Ας συνεχίσουμε τα παραδείγματα, με ένα αξιολάτρευτο αλλά όχι αχώριστο ζευγάρι και ακολούθως με μία ιδιότροπη παρέα.

Η μάχη των δύο Φύλων Ο Παρμενίωνας και η Ιοκάστη είναι πολλά χρόνια ζευγάρι. Λόγω του ότι εργάζονται πολλές

ώρες την ημέρα, προσπαθούν τα βράδια να βρίσκονται και να τα περνάνε μαζί κάνοντας βόλτες, πηγαίνοντας κινηματογράφο ή ότι άλλο τους κάνει κέφι. Ένα χαρακτηριστικό γνώρισμά τους είναι το ότι ενώ δεν έχουν τις ίδιες προτιμήσεις (γούστα), θέλουν να είναι αχώριστοι. Έτσι προσπαθούν να κάνουν ποικίλες εξόδους ούτως ώστε να παραμένουν και οι δύο ευχαριστημένοι και να μην αδικείται κανένας.

Ο Παρμενίωνας το τελευταίο διάστημα έχει υψηλό φόρτο εργασίας και είναι λιγάκι κουρασμένος και αφηρημένος. Το μόνο που σκέφτεται τη σημερινή μέρα είναι η βραδινή του έξοδος με την Ιοκάστη . Λόγω κούρασης όμως θα προτιμούσε να μην πάει τη σημε­ρινή βραδιά στην αγαπημένη όπερα της Ιοκάστης, αλλά να παρακολουθήσει τον ποδοσφαιρικό αγώνα της αγαπημένης του ομάδας από το σπίτι. Από την άλλη η Ιοκάστη γνωρίζει ότι ο σύ­ντροφός της είναι κουρασμένος, θέλει να του κάνει το χατίρι, αλλά δεν έχει όρεξη για ποδό­σφαιρο και θέλει διακαώς να παρακολουθήσει την όπερα2.

Λόγω αφηρημάδας, ο Παρμενίωνας έχει ξεχάσει το κινητό του αφόρτιστο και είναι μέσα στο αυτοκίνητο καθοδόν για το σπίτι. Αυτό που τριβελίζει το μυαλό του είναι αν θα καταφέρουν να συνεννοηθούν με την Ιοκάστη και να μην μαλώσουν. Το ίδιο αναρωτιέται και η Ιοκάστη πε­ριμένοντας στο σπίτι. Όπως και να έχει, θέλουν να είναι μαζί.

Οι σκέψεις (και οι α�συχίες) και των δύο μπορούν να aποτυπωθούν στον παρακάτω πίνα­κα με κλί ακα από το 0- 1 Ο των π οτι ' σεών του :

Πα Ιοκάσ Όπε α Π οδόσ Π:5 Π:Ο 1 : 1 0 1 :0

Ποδόσφαιρο Π:Ο Π: 1 Ο 1 :0 Ι :5

Ποία είναι η έκβαση του παιγνίου; Θα παραμείνει το ζευγάρι αχώριστο ή θα μαλώσουν και ο

ΕΥΚΛΕΙΔΗΣ Α' 88 τ.4/47

Page 50: Ευκλειδης Α 88

--------------- θεωρία Παιγνίων --------------­καθένας θα περάσει τη βραδιά μόνος/η ; Για να εξετάσουμε τη συλλογιστική του παιγνίου.

• Α ν ο Π επιλέξει όπερα, γνωρίζοντας ότι η Ι προτιμάει όπερα, τότε η Ι θα επιλέξει όπερα. Από την άλλη, αν η Ι επιλέξει όπερα τότε ο Π θα επιλέξει όπερα για να είναι μαζί.

• Αν η Ι επιλέξει όπερα τότε ο Π θα επιλέξει όπερα για να είναι μαζί. Από την άλλη, αν ο Π επιλέξει όπερα τότε η Ι θα ακολουθήσει την επιλογή του Π αφού τη συμφέρει.

• Α ν ο Π επιλέξει ποδόσφαιρο τότε η Ι θα επιλέξει ποδόσφαιρο για να είναι μαζί. Από την άλλη, αν η Ι επιλέξει ποδόσφαιρο τότε ο Π θα ακολουθήσει την επιλογή της Ι αφού τον συμφέρει

• Αν η Ι επιλέξει ποδόσφαιρο, γνωρίζοντας ότι ο Π προτιμάει ποδόσφαιρο, τότε ο Π θα ε­πιλέξει ποδόσφαιρο. Από την άλλη, αν ο Π επιλέξει ποδόσφαιρο τότε η Ι θα επιλέξει πο­δόσφαιρο για να είναι μαζί.

Όπως παρατηρείται το ζευγάρι βάζει τη σχέση του πάνω απ' όλα και έτσι έχουμε δύο ισορ­ροπίες Nash: [όπερα, όπερα] , [ποδόσφαιρο, ποδόσφαιρο] , που όπως θα λέγαμε πιο επιστημονικά έχουν ίση πιθανότητα πραγματοποίησης. Ο καταλυτικός παράγοντας αυτού του παιγνίου, το ο­ποίο έχει τις δύο παραπάνω ισορροπίες, είναι το γεγονός ότι και οι δύο παίκτες συμφωνούν εκ των προτέρων στο ότι θέλουν να είναι αχώριστοι. Οι εκ των προτέρων συμφωνίες στα παίγνια και πως αυτές επηρεάζουν το αποτέλεσμα ενός παιγνίου, αποτελούν ένα τεράστιο και ιδιαίτερα γόνιμο πεδίο επιστημονικής έρευνας.

Συνοψίζοντας, μπορούμε να πούμε ότι η Θεωρία Παιγνίων επιχειρεί μία Μαθηματική προ­σέγγιση των κοινωνικών, πολιτικών και οικονομικών αλληλεπιδράσεων μεταξύ φυσικών ή μη προσώπων με αρκετά μεγάλη επιτυχία. Η Μαθηματική θεμελίωσή της πατάει γερά πάνω στη Μαθηματική Λογική, όπου οι παίκτες θεωρείται ότι συμπεριφέρονται ορθολογικά.

Γενικότερα, στον κόσμο των οικονομικών και κοινωνικών επιστημών, τα άτομα-μονάδες τα οποία διαμορφώνουν καταστάσεις χαρακτηρίζονται ως ορθολογικοί παράγοντες-παίκτες.

Πιο απλά, κάθε άτομο μπορεί να αντιληφθεί τι είναι το καλύτερο, ορθότερο, σημαντικότερο, ουσιαστικότερο κ.τ.λ. για τον εαυτό του, προτού πάρει μια απόφαση για το πώς πρέπει να συμπεριφερθεί. Απαιτείται, λοιπόν, από τα άτομα να συμπεριφέρονται με τον καλύτερο τρόπο σε οποιαδήποτε κατάσταση, ούτως ώστε να ικανοποιείται η θεωρία. Αυτό, όμως, συμβαίνει στην πραγματική ζωή ; Στην καθημερινότητα όπου κάθε πολίτης βρίσκεται αντιμέτωπος με το απρόβλεπτο της ζωής; Σε αυτό

το ερώτημα προσπάθησε να δώσει απάντηση ο Nash και ως ένα μεγάλο βαθμό τα κατάφερε. Όπως είδαμε, η ιδιοφυία του Nash έγκειται στο γεγονός ότι συμπεριέλαβε στη θεωρία του

την κοινή παραδοχή ότι δεν μπορούν όλοι οι άνθρωποι να κάνουν ορθολογικές επιλογές και το αποτέλεσμα, όπου συνήθως είναι το μη αναμενόμενο, συνεχίζει ακόμη και σήμερα να εκπλήσ­σει. Θα μπορούσαμε να πούμε ότι ο Nash ήτανε ένας άνθρωπος ο οποίος δεν δίσταζε να χρησι­μοποιεί τη φαντασία του πάνω στης επιστήμη . Οι πρo-Nash αναλύσεις στη Θεωρία Παιγνίων ενώ ήτανε λογικές, ήτανε αρκετά aποστειρωμένες. Ο λόγος είναι ότι οι λύσεις που προτείνονταν συμβούλευαν τους παίκτες πώς να συμπεριφερθούν, σε ένα οποιοδήποτε παίγνιο, ανεξάρτητα από τις υποκειμενικές προσδοκίες για το τι θα κάνουν οι αντίπαλοί τους.

Με άλλα λόγια: κάνε αυτό που σε συμφέρει καλύτερα ελαχιστοποιώντας τις ζημίες, αδιαφο­ρώντας για τις επιλογές των άλλων, έστω και αν αυτές σε επηρεάζουν. Στη καθημερινότητα ό­μως όταν κάποιος κάνει μία επιλογή, η επιλογή του αυτή δεν αφορά μόνο τον ίδιο αλλά και υπό­λοιπα μέλη της κοινωνίας και αντίστροφα. Όπως όταν κάποιος αποφασίζει για το αν θα επι­σκευάσει το χαλασμένο κινητήρα του αυτοκινήτου του. Η επιλογή που θα κάνει δεν αφορά μόνο τον ίδιο αλλά και το κοινωνικό σύνολο διότι ένας χαλασμένος κινητήρας έχει σοβαρές περιβαλ­λοντικές επιπτώσεις που επηρεάζουν τη δημόσια υγεία. Εσείς τί θα επιλέγατε;

ΕΥΚΛΕΙΔΗΣ Α. 88 τ.4/48

Page 51: Ευκλειδης Α 88

==========================Σπύρος Γεωργίου

ΜΔΙ . Τα ορθογώνια Μπορείτε να βρείτε πόσα είναι τα ορθογώ­

νια παραλληλόγραμμα, οποιουδήποτε μεγέ­θους, που υπάρχουν στο παρακάτω σχήμα;

- , ... - -

1"0

ΜΔ2. Ο επταψήφιος αριθμός Βρείτε έναν επταψήφιο παλινδρομικό α­

ριθμό (διαβάζεται το ίδιο και ανάποδα) για τον οποίο ισχύουν τα παρακάτω:

1 . Κανένα ψηφίο δεν χρησιμοποιείται περισ­σότερο από 2 φορές.

2 . Το τελευταίο ψηφίο είναι τριπλάσιο από το δεύτερο.

3 . Το μηδέν δεν χρησιμοποιείται. 4. Η διαφορά του έκτου από το τρίτο ψηφίο

είναι ίση με το μεσαίο ψηφίο. 5 . Το άθροισμα όλων των ψηφίων είναι 4 1 .

ΜΔ3. Το ζητούμενο ψηφίο Τα γράμματα Α, Β, Γ, και Δ παρακάτω α­

ντιπροσωπεύουν όλα διαφορετικά ψηφία. Α ν

Α Β + Γ Α

Δ Α

Α Β - Γ Α

Α ποιο ψηφίο αντιπροσωπεύει το γράμμα Δ;

ΜΔ4. Ο διαιρέτης Στην παρακάτω ισότητα κάθε γράμμα είναι και ένα διαφορετικό ψηφίο από 1 έως 9 .

Α · Β · Γ Ε =

Δ Ποιες είναι οι δυνατές τιμές του διαιρέτη Δ;

ΜΔS. Τα 6 αστέρια Τοποθετήστε 6 αστέρια στο παρακάτω πλέγ­μα, έτσι ώστε σε κάθε γραμμή, κάθε στήλη και κάθε μία περιοχή να υπάρχει ακριβώς ένα α­στέρι. Τα αστέρια δεν πρέπει να τοποθετού­νται σε διαδοχικά τετράγωνα οριζοντίως, κα­θέτως, ούτε καν διαγωνίως.

·-·---1---- -·-··· --lf---1-·---·-··-----·-

;

I I

ΕΥΚΛΕΙΔΗΣ Α' 88 τ.4/49

Page 52: Ευκλειδης Α 88

------------ Τα Μαθηματικά μας διασκεδάζουν ---------------------------------------------

Απαντήσεις προηγούμενου τεύχους

ΜΔ 7. Ο εξαψήφιος αριΟμός Έστω χ είναι το τρίτο ψηφίο του αριθμού. Τό­

τε χρησιμοποιώντας προσε-κτικά τα δεδομένα

έχουμε ότι:

Το πρώτο ψηφίο είναι χ+ 1 Το πέμπτο ψηφίο είναι χ- 1 Το δεύτερο ψηφίο είναι 1 (χ+ 1 - χ) Το τέταρτο ψηφίο είναι 2 και τέλος το έκτο ψηφίο είναι 3 . Επειδή το άθροισμα όλων είναι 30, έχουμε:

(χ+ 1 )+ 1 +χ+2+(χ- 1 )+3=30, από όπου προ­

κύπτει ότι χ=8, δηλαδή ο ζητούμενος αριθμός είναι ο 9 1 8273 . ΜΔ8. Τα επτά ψηφία Για να είναι διαφορετικά όλα τα ψηφία, θα

πρέπει στα ψηφία των εκατοντάδων και των

χιλιάδων να υπάρχει κρατούμενο.

0 [2] + 0 0 0 0

0 0 0 0 Άρα το 9 και το Ο τοποθετούνται στις παραπά­νω θέσεις. Επίσης τα ψηφία των χιλιάδων

στον προσθετέο και το άθροισμα πρέπει να δι­αφέρουν κατά μία μονάδα.

0 [2] + [] 0 0 0

0 0 0 0 Άρα το 2 και το 3 τοποθετούνται σε αυτές τις

θέσεις.

Τέλος οι μόνες θέσεις που μπορεί να βρίσκο­

νται τα ψηφία 1 , 5 και 8 είναι όπως φαίνεται

στη λύση παρακάτω:

+

0 [2]

0 0 0 ω

0 0 [2] 0 ΜΔ9. Τα βάρη Καθένα από τα 5 άτομα ανέβηκε στη ζυγαριά

τέσσερεις φορές (μία με κάθε ένα από το υπό­

λοιπα 4 άτομα). Άρα το άθροισμα όλων των

μετρήσεων ισούται με το τετραπλάσιο του α­

θροίσματος των βαρών των 5 ατόμων.

Επομένως αφού το άθροισμα των μετρήσεων

είναι 1 504, το άθροισμα των βαρών των 5 α­τόμων είναι 1 504:4=376Kg. Η μικρότερη τώ­

ρα από τις μετρήσεις αντιστοιχεί στο άθροι­

σμα των βαρών των 2 "ελαφρότερων" ατό­

μων, ενώ η μεγαλύτερη από τις μετρήσεις στο

άθροισμα των βαρών των 2 "βαρύτερων" ατό­μων. Αφαιρώντας αυτές τις δυο τιμές από το άθροι­

σμα των βαρών όλων, έχουμε το βάρος του

μεσαίου ατόμου, που είναι ίσο δηλαδή με 376-1 24- 1 80=72Kg.

ΜΔ10. Το τετράγωνο

MΔl l . Πρώτοι αριθμοί Θεωρούμε πρώτους αριθμούς, έτσι ώστε κάθε

ψηφίο τους να είναι 2, 3, 5, ή 7 . Ο μικρότερος διψήφιος αριθμός που μπορεί να σχηματιστεί

με αυτά τα ψηφία είναι ο 22, αλλά δεν είναι

πρώτος. Συνεπώς το 23 είναι ο μικρότερος

πρώτος αριθμός του οποίου τα ψηφία του είναι και αυτά πρώτοι. Οι μεγαλύτεροι διψήφιοι α­

ριθμοί που μπορεί να σχη ματίζονται με αυτά

ψηφία είναι οι 77 και 75, αλλά δεν είναι πρώ­

τοι. Ο μεγαλύτερος αριθμός που πληροί τις

προϋποθέσεις είναι ο 73 . Άρα το ζητούμενο άθροισμα είναι: 23+ 73=96

ΕΥΚΛΕΙΔΗΣ Α ' 88 τ.4/50